*NURSING > HESI > NURS 203 HESI QUESTIONS 2020/2021 over 700 MCQs questions with Correct answers and Rationale (All)

NURS 203 HESI QUESTIONS 2020/2021 over 700 MCQs questions with Correct answers and Rationale

Document Content and Description Below

NURS 203 700 HESI with Rationale 1. Following discharge teaching, a male client with duodenal ulcer tells the nurse the he will drink plenty of dairy products, such as milk, to help coat and protec... t his ulcer. What is the best follow-up action by the nurse? a- Remind the client that it is also important to switch to decaffeinated coffee and tea. b- Suggest that the client also plan to eat frequent small meals to reduce discomfort c- Review with the client the need to avoid foods that are rich in milk and cream. d- Reinforce this teaching by asking the client to list a dairy food that he might select. Rationale: Diets rich in milk and cream stimulate gastric acid secretion and should be avoided. 2. A male client with hypertension, who received new antihypertensive prescriptions at his last visit returns to the clinic two weeks later to evaluate his blood pressure (BP). His BP is 158/106 and he admits that he has not been taking the prescribed medication because the drugs make him “feel bad”. In explaining the need for hypertension control, the nurse should stress that an elevated BP places the client at risk for which pathophysiological condition? a- Blindness secondary to cataracts b- Acute kidney injury due to glomerular damage c- Stroke secondary to hemorrhage d- Heart block due to myocardial damage Rationale: Stroke related to cerebral hemorrhage is major risk for uncontrolled hypertension. 3. The nurse observes an unlicensed assistive personnel (UAP) positioning a newly admitted client who has a seizure disorder. The client is supine and the UAP is placing soft pillows along the side rails. What action should the nurse implement? a- Ensure that the UAP has placed the pillows effectively to protect the client. b- Instruct the UAP to obtain soft blankets to secure to the side rails instead of pillows. a- Assume responsibility for placing the pillows while the UAP completes another task. b- Ask the UAP to use some of the pillows to prop the client in a side lying position. Rationale: The nurse should instruct the UAP to pad the side rails with soft blankest because the use of pillows could result in suffocation and would need to be removed at the onset of the seizure. The nurse can delegate paddling the side rails to the UAP 4. An adolescent with major depressive disorder has been taking duloxetine (Cymbalta) for the past 12 days. Which assessment finding requires immediate follow-up? a- Describes life without purpose b- Complains of nausea and loss of appetite c- States is often fatigued and drowsy d- Exhibits an increase in sweating. Rationale: Cymbalta is a selective serotonin and norepinephrine reuptake inhibitor that is known to increase the risk of suicidal thinking in adolescents and young adults with major depressive disorder. B, C and D are side effects 5. A 60-year-old female client with a positive family history of ovarian cancer has developed an abdominal mass and is being evaluated for possible ovarian cancer. Her Papanicolau (Pap) smear results are negative. What information should the nurse include in the client’s teaching plan? a- Further evaluation involving surgery may be needed b- A pelvic exam is also needed before cancer is ruled out c- Pap smear evaluation should be continued every six month d- One additional negative pap smear in six months is needed. Rationale: An abdominal mass in a client with a family history for ovarian cancer should be evaluated carefully 6. A client who recently underwear a tracheostomy is being prepared for discharge to home. Which instructions is most important for the nurse to include in the discharge plan? a- Explain how to use communication tools. b- Teach tracheal suctioning techniques c- Encourage self-care and independence. d- Demonstrate how to clean tracheostomy site. Rationale: Suctioning helps to clear secretions and maintain an open airway, which is critical. 7. In assessing an adult client with a partial rebreather mask, the nurse notes that the oxygen reservoir bag does not deflate completely during inspiration and the client’s respiratory rate is 14 breaths / minute. What action should the nurse implement? a- Encourage the client to take deep breaths b- Remove the mask to deflate the bag c- Increase the liter flow of oxygen d- Document the assessment data Rational: reservoir bag should not deflate completely during inspiration and the client’s respiratory rate is within normal limits. 8. During a home visit, the nurse observed an elderly client with diabetes slip and fall. What action should the nurse take first? a- Give the client 4 ounces of orange juice b- Call 911 to summon emergency assistance c- Check the client for lacerations or fractures d- Asses clients blood sugar level Rationale: After the client falls, the nurse should immediately assess for the possibility of injuries and provide first aid as needed 9. At 0600 while admitting a woman for a schedule repeat cesarean section (C-Section), the client tells the nurse that she drank a cup a coffee at 0400 because she wanted to avoid getting a headache. Which action should the nurse take first? a- Ensure preoperative lab results are available b- Start prescribed IV with lactated Ringer’s c- Inform the anesthesia care provider d- Contact the client’s obstetrician. Rationale: Surgical preoperative instruction includes NPO after midnight the day of surgery to decrease the risk of aspiration should vomiting occur during anesthesia. While it is possible the C-section will be done on schedule or rescheduled for later in the day, the anesthesia provider should be notified first. 10. After placing a stethoscope as seen in the picture, the nurse auscultates S1 and S2 heart sounds. To determine if an S3 heart sound is present, what action should the nurse take first? a- Side the stethoscope across the sternum. b- Move the stethoscope to the mitral site c- Listen with the bell at the same location d- Observe the cardiac telemetry monitor Rationale: The nurse uses the bell of the stethoscope to hear low-pitched sounds such as S3 and S4. The nurse listens at the same site using the diaphragm the diaphragm and bell before moving systematically to the next sites. 11. A 66-year-old woman is retiring and will no longer have a health insurance through her place of employment. Which agency should the client be referred to by the employee health nurse for health insurance needs? a- Woman, Infant, and Children program b- Medicaid c- Medicare d- Consolidated Omnibus Budget Reconciliation Act provision. Rationale: Title XVII of the social security Act of 1965 created Medicare Program to provide medical insurance for person more than 65 years or older, disable or with permeant kidney failure, WIC provides supplemental nutrition to meet the needs of pregnant of breastfeeding woman, infants and children up to age of 6. Medicaid provides financial assistance to pay for medical services for poor older adults, blind, disable and families with dependent children. COBRA(D) health benefit provisions is a limited insurance plan for those who has been laid off or become unemployed. 12. A client who is taking an oral dose of a tetracycline complains of gastrointestinal upset. What snack should the nurse instruct the client to take with the tetracycline? a- Fruit-flavored yogurt. b- Cheese and crackers. c- Cold cereal with skim milk. d- Toasted wheat bread and jelly Rationale: Dairy products decrease the effect of tetracycline, so the nurse instructs the client to eat a snack such as toast, which contains no dairy products and may decrease GI symptoms. 13. Following a lumbar puncture, a client voices several complaints. What complaint indicated to the nurse that the client is experiencing a complication? a- “I am having pain in my lower back when I move my legs” b- “My throat hurts when I swallow” c- “I feel sick to my stomach and am going to throw up” d- I have a headache that gets worse when I sit up” Rationale: A post-lumbar puncture headache, ranging from mild to severe, may occur as a result of leakage of cerebrospinal fluid at the puncture site. This complication is usually managed by bedrest, analgesic, and hydration. 14. An elderly client seems confused and reports the onset of nausea, dysuria, and urgency with incontinence. Which action should the nurse implement? a- Auscultate for renal bruits b- Obtain a clean catch mid-stream specimen c- Use a dipstick to measure for urinary ketone d- Begin to strain the client’s urine. Rationale: This elderly is experiencing symptoms of urinary tract infection. The nurse should obtain a clean catch mid-stream specimen to determine the causative agent so an anti-infective agent can be prescribed. 15. The nurse is assisting the mother of a child with phenylketonuria (PKU) to select foods that are in keeping with the child’s dietary restrictions. Which foods are contraindicated for this child? a- Wheat products b- Foods sweetened with aspartame. c- High fat foods d- High calories foods. Rationale: Aspartame should not be consumed by a child with PKU because ut is converted to phenylalanine in the body. Additionally, milk and milk products are contraindicated for children with PKU. 16. Before preparing a client for the first surgical case of the day, a part-time scrub nurse asks the circulating nurse if a 3-minute surgical hand scrub is adequate preparation for this client. Which response should the circulating nurse provide? a- Ask a more experience nurse to perform that scrub since it is the first time of the day b- Validate the nurse is implementing the OR policy for surgical hand scrub c- Inform the nurse that hand scrubs should be 3 minutes between cases. d- Direct the nurse to continue the surgical hand scrub for a 5-minute duration. Rationale: The surgical hand scrub should last for 5 to 10 mints, so the nurse should be directed to continue the vigorous scrub using a reliable agent for the total duration of 5 mints. It is not necessary to reassign staff (A). The length of the hand scrub and subsequent scrubs during the day require the same process for the same amount of time, (B and C) 17. Which breakfast selection indicates that the client understands the nurse’s instructions about the dietary management of osteoporosis? a- Egg whites, toast and coffee. b- Bran muffin, mixed fruits, and orange juice. c- Granola and grapefruit juice d- Bagel with jelly and skim milk. Rationale: D includes dairy products which contain calcium and does not include any foods that inhibit calcium absorption. The primary dietary implication of osteoporosis is the need for increased calcium and reduction in foods that decrease calcium absorption, such as caffeine and excessive fiber. 18. The charge nurse of a critical care unit is informed at the beginning of the shift that less than the optimal number of registered nurses will be working that shift. In planning assignments, which client should receive the most care hours by a registered nurse (RN)? a- A 34-year -old admitted today after an emergency appendendectomy who has a peripheral intravenous catheter and a Foley catheter. b- A 48-year-old marathon runner with a central venous catheter who is experiencing nausea and vomiting due to electrolyte disturbance following a race. c- A 63-year-old chain smoker admitted with chronic bronchitis who is receiving oxygen via nasal cannula and has a saline-locked peripheral intravenous catheter. d- An 82-year-old client with Alzheimer’s disease newly-fractures femur who has a Foley catheter and soft wrist restrains applied Rationale: (D) describe the client at the most risk for injury and complications because of the factor listed. (A) has complete the recovery period form anesthesia but requires critical care because of the invasive lines and new abdominal incision. (B) is likely to be in excellent physical condition and has one invasive line needed for rehydration. (C) is essentially stable, despite having a chronic condition. 19. Z a- Cleanse the foot with soap and water and apply an antibiotic ointment b- Provide teaching about the need for a tetanus booster within the next 72 hours. c- have the mother check the child's temperature q4h for the next 24 hours d- transfer the child to the emergency department to receive a gamma globulin injection Rationale: The nurse should cleanse the wound first and implement B next. 20. The mother of an adolescent tells the clinic nurse, “My son has athlete’s foot, I have been applying triple antibiotic ointment for two days, but there has been no improvement.” What instruction should the nurse provide? a- Antibiotics take two weeks to become effective against infections such as athlete’s foot. b- Continue using the ointment for a full week, even after the symptoms disappear. c- Applying too much ointment can deter its effectiveness. Apply a thin layer to prevent maceration. d- Stop using the ointment and encourage complete drying of the feet and wearing clean socks. Rationale: Athlete’s foot (tinea pedi) is a fungal infection that afflicts the feet and causes scaliness and cracking of the skin between the toes and on the soles of the feet. The feet should be ventilated, dried well after bathing, and clean socks should be placed on the feet after bathing. Antifungal ointments may be prescribed, but antibiotic ointments are not useful. 21. A 26-year-old female client is admitted to the hospital for treatment of a simple goiter, and levothyroxine sodium (Synthroid) is prescribed. Which symptoms indicate to the nurse that the prescribed dosage is too high for this client? The client experiences a- Palpitations and shortness of breath b- Bradycardia and constipation c- Lethargy and lack of appetite d- Muscle cramping and dry, flushed skin Rationale: An overdose of thyroid preparation generally manifests symptoms of an agitated state such as tremors, palpitations, shortness of breath, tachycardia, increased appetite, agitation, sweating and diarrhea. 22. A client with a history of heart failure presents to the clinic with a nausea, vomiting, yellow vision and palpitations. Which finding is most important for the nurse to assess to the client? a- Determine the client’s level of orientation and cognition b- Assess distal pulses and signs of peripheral edema c- Obtain a list of medications taken for cardiac history. d- Ask the client about exposure to environmental heat. Rationale: The client is presenting with signs of digitalis toxicity. A list of medication, which is likely to include digoxin (Lanoxin) for heart failure, can direct further assessment in validating digitalis toxicity with serum labels greater than 2 mg/ml that is contributing to client’s presenting clinical picture. 23. The healthcare provider prescribes an IV solution of isoproterenol (Isuprel) 1 mg in 250 ml of D5W at 300 mcg/hour. The nurse should program the infusion pump to deliver how many ml/hour? (Enter numeric value only.) a- 75 Rationale: Convert mg to mcg and use the formula D/H x Q. 300 mcg/hour / 1,000 mcg x 250 ml = 3/1 x 25 = 75 ml/hour 24. The pathophysiological mechanisms are responsible for ascites related to liver failure? (Select all that apply) a- Bleeding that results from a decreased production of the body’s clotting factors b- Fluid shifts from intravascular to interstitial area due to decreased serum protein c- Increased hydrostatic pressure in portal circulation increases fluid shifts into abdomen d- Increased circulating aldosterone levels that increase sodium and water retention e- Decreased absorption of fatty acids in the duodenum leading to abdominal distention. Rationale: When liver fail production of albumin is reduced. Since albumin is the primary serum protein creating intravascular osmotic pressure, decreased serum protein allows a fluids shift into the interstitial space. Pressure increases in the portal circulation © when venous return from the upper GI tract cannot flow freely into sclerosed liver, which cause a pressure gradient to further Increase fluid shifts into the abdomen. A failing liver ineffectively inactivates steroidal hormones, such as aldosterone resulting in sodium and water retention. 25. The nurse is auscultating a client’s heart sounds. Which description should the nurse use to document this sound? (Please listen to the audio first to select the option that applies) a- S1 S2 b- S1 S2 S3 c- Murmur d- Pericardial friction rub. Rationale: A murmur is auscultated as a swishing sound that is associated with the blood turbulence created by the heart or valvular defect. B is associate with Heart Failure. 26. The healthcare provider prescribes celtazidime (Fortax) 35 mg every 8 hours IM for an infant. The 500 mg vial is labeled with the instruction to add 5.3 ml diluent to provide a concentration of 100 mg/ml. How many ml should the nurse administered for each dose? (Enter numeric value only. If rounding is required, round to the nearest tenth) a- 0.4 Rationale: 35mg/100mg x 1 = 0.35 = 0.4 ml 27. The nurse notes that a client has been receiving hydromorphone (Dilaudid) every six hours for four days. What assessment is most important for the nurse to complete? a- Auscultate the client's bowel sounds b- Observe for edema around the ankles c- Measure the client’s capillary glucose level d- Count the apical and radial pulses simultaneously Rationale: hydromorphone is a potent opioid analgesic that slows peristalsis and frequently causes constipation, so it is most important to Auscultate the client's bowel sounds 28. A female client is admitted with end stage pulmonary disease is alert, oriented, and complaining of shortness of breath. The client tells the nurse that she wants “no heroic measures” taken if she stops breathing, and she asks the nurse to document this in her medical record. What action should the nurse implement? a- Ask the client to discuss “do not resuscitate” with her healthcare provider 29. A client is receiving a full strength continuous enteral tube feeding at 50 ml/hour and has developed diarrhea. The client has a new prescription to change the feeding to half strength. What intervention should the nurse implement? a- Add equal amounts of water and feeding to a feeding bag and infuse at 50ml/hour b- Continue the full strength feeding after decreasing the rate of infusion to 25 ml/hr. c- Maintain the present feeding until diarrhea subsides and the begin the next new prescription. d- Withhold any further feeding until clarifying the prescription with healthcare provides. Rationale: Diluting the formula can help alleviate the diarrhea. Diarrhea can occur as a complication of enteral tube feeding and can be due to a variety of causes including hyperosmolar formula. 30. A female client reports that her hair is becoming coarse and breaking off, that the outer part of her eyebrows have disappeared, and that her eyes are all puffy. Which follow-up question is best for the nurse to ask? a- “Is there a history of female baldness in your family?” b- “Are you under any unusual stress at home or work?” c- “Do you work with hazardous chemicals?” d- “Have you noticed any changes in your fingernails?” Rationale: The pattern of reported manifestations is suggestive of hypothyroidism. A question about the fingernails adds data to the clinical picture. 31. After a third hospitalization 6 months ago, a client is admitted to the hospital with ascites and malnutrition. The client is drowsy but responding to verbal stimuli and reports recently spitting up blood. What assessment finding warrants immediate intervention by the nurse? a- Bruises on arms and legs b- Round and tight abdomen c- Pitting edema in lower legs d- Capillary refill of 8 seconds Rationale: The client is bleeding and hypovolemia is likely. Capillary refill is greater than 3 to 5 seconds indicates poor perfusion and requires immediate attention 32. After the nurse witnesses a preoperative client sign the surgical consent form, the nurse signs the form as a witness. What are the legal implications of the nurse’s signature on the client’s surgical consent form? (Select all that apply) a- The client voluntarily grants permission for the procedure to be done b- The surgeon has explained to the client why the surgery is necessary. c- The client is competent to sign the consent without impairment of judgment d- The client understands the risks and benefits associated with the procedure e- After considering alternatives to surgery, the client elects to have the procedure. Rationale: Inform consent is required for any invasive procedure. The nurse’s signature as a witness to the client’s signature on surgical consent indicates that the client voluntary gives consent for the scheduled procedure. C is competent to give consent, and D and understand the risk and benefits of the procedure. 33. Following surgery, a male client with antisocial personality disorder frequently requests that a specific nurse be assigned to his care and is belligerent when another nurse is assigned. What action should the charge nurse implement? a- Ask the client to explain why he constantly request the nurse b- Encourage the client to verbalize his feelings about the nurse c- Reassure the client that his request will be met whenever possible. d- Advise the client that assignments are not based on client requests Rationale: Those with antisocial personality disorders are manipulative in order to meet their own needs. The charge nurse must set limits on this behavior. The client’s superficial charm and emotional maturity prevent effective therapeutic communication and (A and B) will be used to the client’s advantage. C encourage further manipulative behavior. 34. A client with cervical cancer is hospitalized for insertion of a sealed internal cervical radiation implant. While providing care, the nurse finds the radiation implant in the bed. What action should the nurse take? a- Call the radiology department b- Reinsert the implant into the vagina c- Apply double gloves to retrieve the implant for disposal. d- Place the implant in a lead container using long-handled forceps Rationale: Solid or sealed radiation sources, such as Cesium which is removed after treatment, are inserted into an applicator or cervical implant to emit continuous, low energy radiation for adjacent tumor tissues. If the radiation source or the applicator become dislodged long-handled forceps should be used to retrieve the radiation implant to prevent injury due to direct handling. The applicator is then placed in the lead container. 35. The client with which type of wound is most likely to need immediate intervention by the nurse? a- Laceration b- Abrasion c- Contusion d- Ulceration Rationale: A laceration is an open wound whose edges are often jagged and whose tissue are torn apart that is produced by the tearing of soft body tissue, placing the client at risk for bleeding, so this type of wound is likely to require the most immediate nursing intervention. A laceration wound is often contaminated with bacteria and debris from whatever object caused the cut. 36. The nurse is planning care for a client admitted with a diagnosis of pheochromocytoma. Which intervention has the highest priority for inclusion in this client’s plan of care? a- Record urine output every hour b- Monitor blood pressure frequently c- Evaluate neurological status d- Maintain seizure precautions Rationale: A pheochromocytoma is a rare, catecholamine-secreting tumor that may precipitate life-threatening hypertension. The tumor is malignant in 10% of cases but may be cured completely by surgical removal. Although pheochromocytoma has classically been associated with 3 syndromes—von Hippel-Lindau (VHL) syndrome, multiple endocrine neoplasia type 2 (MEN 2), and neurofibromatosis type 1 (NF1)—there are now 10 genes that have been identified as sites of mutations leading to pheochromocytoma. 37. When caring for a client who has acute respiratory distress syndrome (ARDS), the nurse elevates the head of the bed 30 degrees. What is the reason for this intervention? a- To reduce abdominal pressure on the diaphragm b- to promote retraction of the intercostal accessory muscle of respiration c- to promote bronchodilation and effective airway clearance d- to decrease pressure on the medullary center which stimulates breathing Rationale: a semi-sitting position is the best position for matching ventilation and perfusion and for decreasing abdominal pressure on the diaphragm, so that the client can maximize breathing. 38. When assessing a mildly obese 35-year-old female client, the nurse is unable to locate the gallbladder when palpating below the liver margin at the lateral border of the rectus abdominal muscle. What is the most likely explanation for failure to locate the gallbladder by palpation? a- The client is too obese b- Palpating in the wrong abdominal quadrant c- The gallbladder is normal d- Deeper palpation technique is needed Rationale: A normal healthy gallbladder is not palpable. 39. A woman with an anxiety disorder calls her obstetrician’s office and tells the nurse of increased anxiety since the normal vaginal delivery of her son three weeks ago. Since she is breastfeeding, she stopped taking her antianxiety medications, but thinks she may need to start taking them again because of her increased anxiety. What response is best for the nurse to provide this woman? a- Describe the transmission of drugs to the infant through breast milk b- Encourage her to use stress relieving alternatives, such as deep breathing exercises c- Inform her that some antianxiety medications are safe to take while breastfeeding d- Explain that anxiety is a normal response for the mother of a 3-week-old. Rationale: There are several antianxiety medications that are not contraindicated for breastfeeding mothers. The woman is apparently aware that drugs can be transmitted through breast milk, so A is not helpful. C might be helpful, but the client’s history suggest that nonpharmacological methods of anxiety management do not produce the best outcome. (D) the mother’s history places her at risk for severe anxiety. 40. An older male client with a history of type 1 diabetes has not felt well the past few days and arrives at the clinic with abdominal cramping and vomiting. He is lethargic, moderately, confused, and cannot remember when he took his last dose of insulin or ate last. What action should the nurse implement first? a- obtain a serum potassium level b- administer the client's usual dose of insulin c- assess pupillary response to light d- Start an intravenous (IV) infusion of normal saline Rationale: The nurse should first start an intravenous infusion of normal saline to replace fluids and electrolytes because the client has been vomiting, and it is unclear when he last ate or took insulin. The symptoms of confusion, lethargy, vomiting, and abdominal cramping are all suggestive of hyperglycemia, which also contributes to diuresis and fluid electrolyte imbalance. 41. A client who received multiple antihypertensive medications experiences syncope due to a drop-in blood pressure to 70/40. What is the rationale for the nurse’s decision to hold the client’s scheduled antihypertensive medication? a- Increased urinary clearance of the multiple medications has produced diuresis and lowered the blood pressure b- The antagonistic interaction among the various blood pressure medications has reduced their effectiveness c- The additive effect of multiple medications has caused the blood pressure to drop too low. d- The synergistic effect of the multiple medications has resulted in drug toxicity and resulting hypotension. Rationale: When medication with a similar action are administered, an additive effect occurs that is the sum of the effects of each of the medication. In this case, several medications that all lower the blood pressure, when administer together, resulted in hypotension. 42. Which client is at the greatest risk for developing delirium? a- An adult client who cannot sleep due to constant pain. b- an older client who attempted 1 month ago c- a young adult who takes antipsychotic medications twice a day d- a middle-aged woman who uses a tank for supplemental oxygen Rationale: Client who are in constant pain ad have difficulty sleeping or resting are at high risk for delirium. Supplemental oxygen may cause confusion. B is taking medication so is not at high risk of delirium. 43. Which intervention should the nurse include in a long-term plan of care for a client with Chronic Obstructive Pulmonary Disease (COPD)? a- Reduce risks factors for infection b- Administer high flow oxygen during sleep c- Limit fluid intake to reduce secretions d- Use diaphragmatic breathing to achieve better exhalation Rationale: Interventions aimed at reducing the risk factors of infections should be included in the plan of care COPD client are at particular risk for respiratory infection. Prevention and early detection of infections are necessary. 44. Which location should the nurse choose as the best for beginning a screening program for hypothyroidism? a- A business and professional women's group. b- An African-American senior citizens center c- A daycare center in a Hispanic neighborhood d- An after-school center for Native-American teens Rationale: The population at highest risk is A so this is the group that would benefit the most for a screening program of hypothyroidism and occurs between 35 and 60 years of age and is most common in females. 45. A female client has been taking a high dose of prednisone, a corticosteroid, for several months. After stopping the medication abruptly, the client reports feeling “very tired”. Which nursing intervention is most important for the nurse to implement? a- Measure vital signs b- Auscultate breath sounds c- Palpate the abdomen d- Observe the skin for bruising Rationale: Abrupt withdrawal of an exogenous corticosteroids can precipitate adrenal insufficiency and hypoglycemia, hypokalemia, and circulatory collapse can occur. Is most important for the nurse to assess vital sign to impending shock. 46. A male client reports the onset of numbness and tingling in his fingers and around his mouth. Which lab is important for the nurse to review before contacting the health care provider? a- capillary glucose b- urine specific gravity c- Serum calcium d- white blood cell count Rationale: Numbness and tingling of the fingers and around the mouth, along with muscle cramps are signs of hypocalcemia 47. What explanation is best for the nurse to provide a client who asks the purpose of using the log-rolling technique for turning? a- working together can decrease the risk for back injury b- The technique is intended to maintain straight spinal alignment. c- Using two or three people increases client safety. d- turning instead of pulling reduces the likelihood of skin damage Rationale: The main rationale for use of the long-rolling technique is to maintain the client’s spine straight alignment. 48. A client receiving chemotherapy has severe neutropenia. Which snack is best for the nurse to recommend to the client? a- Plain yogurt with sweetened with raw honey b- Peanuts in the shell, roasted or un-roasted. c- Aged farmer’s cheese with celery sticks d- Baked apples topped with dried raisins Rationale: A patient with chemotherapy-induced severe neutropenia is at high risk for infection. A low bacteria diet is required D is a healthy snack for a client receiving chemotherapy. A, B and C have a high bacterial count and should be avoided. 49. Which action should the school nurse take first when conducting a screening for scoliosis? a- Compare dorsal measurement of trunk b- Extend arms over head for visualization c- Inspect for symmetrical shoulder height. d- Observe weight-bearing on each leg. Rationale: Children between 9 and 15 years old should be screening for scoliosis, which is exhibited…. Vertebral column. Screening for scoliosis should begin with inspection of shoulder height. 50. An unlicensed assistive personnel (UAP) assigned to obtain client vital signs reports to the charge nurse that a client has a weak pulse with a rate of 44 beat/ minutes. What action should the charge nurse implement? a- Instruct the UAP to count the client apical pulse rate for sixty seconds b- Determine if the UAP also measured the client’s capillary refill time. c- Assign a practical nurse (LPN) to determine if an apical radial deficit is present. d- Notify the health care provider of the abnormal pulse rate and pulse volume. Rationale: Since the client has bradycardia and week pulse volume, further assessment should be performed by an LPN… of assessment skills (C) than the UAP. While follow-up assessment is outside the scope of the UAP 51. After a sudden loss of consciousness, a female client is taken to the ED and initial assessment indicate that her blood glucose level is critically low. Once her glucose level is stabilized, the client reports that was recently diagnosed with anorexia nervosa and is being treated at an outpatient clinic. Which intervention is more important to include in this client’s discharge plan? a- Describe the signs and symptoms of hypoglycemia. b- Encourage a low-carbohydrate and high-protein diet c- Reinforce the need to continue outpatient treatment d- Suggest wearing a medical alert bracelet at all time. Rationale: A client with anorexia nervosa with long term starvation or who self-restrict intake can sign…. Reserves. Providing the client with dietary selections such as low-carbohydrate, high protein…. Hypoglycemic episodes, which can become life-threating. 53- A client with a peripherally inserted central catheter (PICC) line has a fever. What client assessment is most important for the nurse to perform? a- Observe the antecubital fossa for inflammation. 54- The nurse administers an antibiotic to a client with respiratory tract infection. To evaluate the medication’s effectiveness, which laboratory values should the nurse monitor? Select all that apply a- White blood cell (WBC) count b- Sputum culture and sensitivity 55- A client is admitted to isolation with the diagnosis of active tuberculosis. Which infection control measures should the nurse implement? a- Negative pressure environment b- contact precautions c- droplet precautions d- protective environment 56- A school nurse is called to the soccer field because a child has a nose bleed (epistaxis). In what position should the nurse place the child? a- Sitting up and leaning forward 57- A young adult who is hit with a baseball bat on the temporal area of the left skull is conscious when admitted to the ED and is transferred to the Neurological Unit to be monitored for signs of closed head injury. Which assessment finding is indicative of a developing epidural hematoma? a- Altered consciousness within the first 24 hours after injury. 58- A female client with breast cancer who completed her first chemotherapy treatment today at an out-patient center is preparing for discharge. Which behavior indicates that the client understands her care needs a- Rented movies and borrowed books to use while passing time at home 59- Which instruction should the nurse provide a pregnant client who is complaining of heartburn? a- Limit fluids between meals to avoid over distension of the stomach b- Take an antacid at bedtime and whenever symptoms worsen c- Maintain a sitting position for two hours after eating. d- Eat small meal throughout the day to avoid a full stomach. Rationale: Eating small frequent meals throughout the day decreases stomach fullness and helps decrease heartburn. Fluids should not be consumed with foods because they further distend the stomach, but fluids not be limited between meals (A) because this puts the client at risk for dehydration. (B) is not recommended during pregnancy unless prescribed by the health provider because they place the client at risk for electrolyte imbalance (sodium), constipation (aluminum, or diarrhea (magnesium) (C) is less effective than (D) preventing heartburn. 60- A client is admitted to the intensive care unit with diabetes insipidus due to a pituitary gland tumor. Which potential complication should the nurse monitor closely? a- Hypokalemia b- Ketonuria. c- Peripheral edema d- Elevated blood pressure Rational: pituitary tumors that suppress antidiuretic hormone (ADH) result in diabetes insipidus, which causes massive polyuria and serum electrolyte imbalances, including hypokalemia, which can lead to lethal arrhythmias. 61- A female client reports she has not had a bowel movement for 3 days, but now is defecating frequent small amount of liquid stool. Which action should the nurse implement? a- Digitally check the client for a fecal impaction 62- After changing to a new brand of laundry detergent, an adult male report that he has a fine itchy rash. Which assessment finding warrants immediate intervention by the nurse? a- Bilateral Wheezing. 63- The nurse should teach the parents of a 6 year-old recently diagnosed with asthma that the symptom of acute episode of asthma are due to which physiological response? a- Inflammation of the mucous membrane & bronchospasm 64- A 10-year-old who has terminal brain cancer asks the nurse, "What will happen to my body when I die?" How should the nurse respond? a- "The heart will stop beating & you will stop breathing." 65- The nurse is assessing a 3-month-old infant who had a pylorotomy yesterday. This child should be medicated for pain based on which findings? Select all that apply: a- Restlessness b- Clenched Fist (puño cerrado) c- Increased pulse rate d- Increased respiratory rate. e- Increased temperature f- Peripheral pallor of the skin 66- The nurse is preparing to administer an oral antibiotic to a client with unilateral weakness, ptosis, mouth drooping and, aspiration pneumonia. What is the priority nursing assessment that should be done before administering this medication? a- Ask the client about soft foods preferences b- Auscultate the client’s breath sounds c- Obtain and record the client’s vital signs d- Determine which side of the body is weak. 67- The nurse who is working on a surgical unit receives change of shift report on a group of clients for the upcoming shift. A client with which condition requires the most immediate attention by the nurse? a- Gunshot wound three hours ago with dark drainage of 2 cm noted on the dressing. b- Mastectomy 2 days ago with 50 ml bloody drainage noted in the Jackson-pratt drain. c- Collapsed lung after a fall 8h ago with 100 ml blood in the chest tube collection container d- Abdominal-perineal resection 2 days ago with no drainage on dressing who has fever and chills. Rationale: the client with an abdominal- perineal resection is at risk for peritonitis and needs to be immediately assessed for other signs and symptoms for sepsis. 68- The nurse is caring for a client who had gastric bypass surgery yesterday. Which intervention is most important for the nurse to implement during the first 24 postoperative hours? a- Insert an indwelling urinary catheter b- Monitor for the appearance of an incisional hernia c- Instruct the client to eat small frequent meals d- Measure hourly urinary output. Rationale: A serious early complications of gastric bypass surgery is an anastomoses leak, often resulting in death. Early detection is critical, so the nurse should monitor for signs and symptom, which include unexplained tachycardia, oliguria and pain in the back, left shoulder, abdomen. 69- When preparing to discharge a male client who has been hospitalized for an adrenal crisis, the client expresses concern about having another crisis. He tells the nurse that he wants to stay in the hospital a few more days. Which intervention should the nurse implement? a- Administer anti=anxiety medication prior to providing discharge instructions b- Schedule an appointment for an out-patient psychosocial assessment. c- Obtain a blood cortisol level after last dose of synthetic ACTH d- Encourage the healthcare provider to delay the client’s discharge. Rationale: Emotional stress can precipitate another adrenal crisis and should be monitored with periodic psychosocial assessments. A may be indicated but does not address the problem after discharge. A blood cortisol level is to diagnosis not to monitor the ongoing disease process. Canceling the discharge only delays the resolution of the problem. 70- An adult female client tells the nurse that though she is afraid her abusive boyfriend might one-day kill her, she keeps hoping that he will change. What action should the nurse take first? a- Report the finding to the police department b- Discuss treatment options for abusive partners c- Determine the frequency and type of client’s abuse d- Explore client’s readiness to discuss the situation. Rationale: By assessing the client’s level of readiness to discuss her situation the nurse can begin to stablish trust so that further action can be taken to protect her. The nurse needs the client’s permission to report the abuse to the police department, which may be obtained after trust is established. B might be an option during the discussion it is most important that the client has a safe refuge even if the abusive partner does not commit seeking help. 71- In caring for a client with Cushing syndrome, which serum laboratory value is most important for the nurse to monitor? a- Lactate b- Glucose c- Hemoglobin d- Creatinine Rationale: Cushing syndrome, caused by excess corticosteroids causes hyperglycemia and the client’s serum glucose level should be monitor for this side effect. 72- Azithromycin is prescribed for an adolescent female who has lower lobe pneumonia and recurrent chlamydia. What information is most important for the nurse to provide to this client? a- Have partner screened for human immunodeficiency virus b- Report a sudden onset arthralgia to the healthcare provider c- Decrease intake of high-fat-foods, caffeine, and alcohol d- Use two forms of contraception while taking this drug. Rationale: Antibiotic, especially broad-spectrum drugs, like azithromycin, decrease the effectiveness of oral contraceptives and some spermicides, so the adolescent should be encouraging to use at least two forms of contraception to prevent pregnancy. 73- A client in the emergency center demonstrates rapid speech, flight of ideas, and reports sleeping only three hours during the past 48h. Based on these finding, it is most important for the nurse to review the laboratory value for which medication? a- Olanzapine b- Divalproex. c- Lorazepam d- Fluoxetine Rationale: divalproex is the first line of treatment for bipolar disorder BPD because it has a high therapeutic index, few side effects, and a rapid onset in controlling symptoms and preventing recurrent episodes of mania and depression. The serum value of divalproex should be determined since the client is exhibiting symptoms of mania, which may indicate non-compliance with the medication regimen. 74- A male client who is admitted to the mental health unit for treatment of bipolar disorder has a slightly slurred speech pattern and an unsteady gait. Which assessment finding is most important for the nurse to report to the healthcare provider? a- Blood alcohol level of 0.09% b- Serum lithium level of 1.6 mEq/L or mmol/l (SI) c- Six hours of sleep in the past three days. d- Weight loss of 10 pounds (4.5 kg) in past month. Rationale: The therapeutic level of Serum lithium is 0.6 to 1.2 mEq/L or mmol/l (SI). Slurred speech and ataxia are sign of lithium toxicity. 75- A client was admitted to the cardiac observation unit 2 hours ago complaining of chest pain. On admission, the client’s EKG showed bradycardia, ST depression, but no ventricular ectopy. The client suddenly reports a sharp increase in pain, telling the nurse, “I feel like an elephant just stepped on my chest” The EKG now shows Q waves and ST segment elevations in the anterior leads. What intervention should the nurse perform? a- Increase the peripheral IV flow rate to 175 ml/hr to prevent hypotension and shock b- Administer prescribed morphine sulfate IV and provide oxygen at 2 L/min per nasal cannula. c- Obtain a stat 12 lead EKG and perform a venipuncture to check cardiac enzymes levels. d- Notify the healthcare provider of the client’s increase chest pain a call for the defibrillator crash cart. Rationale: Administer morphine sulfate can increase oxygen supply are the priority intervention for symptoms of acute MI and should be supplemented with nitroglycerin and aspirin administration. A may result in overload that the impaired myocardium cannot handdle effectively. C and D are helpful but after. B 76- The nurse is developing a teaching program for the community. What population characteristic is most influential when choosing strategies for implementing a teaching plan? a- Literacy level b- Prevalent learning style c- Median age d- Percent with internet access. Rationale: Reading ability, or literacy level is the most important population characteristic in choosing strategies for implementing teaching plan. If the population cannot read it would be useless to reinforce teaching with written material. 77- A client is being discharged with a prescription for warfarin (Coumadin). What instruction should the nurse provide this client regarding diet? a- Eat approximated the same amount of leafy green vegetables daily so the amount of vitamin K consumed is consistent. b- Avoid eating all foods that contain any vitamin K because it is an antagonist of Coumadin. c- Increase the intake of dark green leafy vegetables while taking Coumadin d- Eat two servings of raw dark green leafy vegetables daily and continue for 30 days after Coumadin therapy is completed. Rationale: The Coumadin dose is prescribed and adjusted based on the client’s normal consumption of foods containing vitamin K (an essential clotting factor that counteracts the effects of Coumadin), so the client should eat a consistent amount of vitamin K food sources (A). (B and C) alter the effectiveness of the already set dosage of Coumadin. (D) provides a consistent amount of vitamin K but does not take into consideration how much is already being consumed by the client. 78- A client who had a small bowel resection acquired methicillin resistant staphylococcus aureus (MRSA) while hospitalized. He treated and released but is readmitted today because of diarrhea and dehydration. It is most important for the nurse to implement which intervention. a- Maintain contact transmission precaution b- Review white blood cell (WBC) count daily c- Instruct visitors to gown and wash hands d- Collect serial stool specimens for culture Rationale” The client may have residual postoperative MRSA infection, a resistant and highly contagious healthcare-associated infection (HAI), that requires strict contact precautions (A), as recommend by the Center for Disease Control (CDC). 79- A postoperative female client has a prescription for morphine sulfate 10 mg IV q3 hours for pain. One dose of morphine was administered when the client was admitted to the post anesthesia care unit (PACU) and 3 hours later, the client is again complaining of pain. Her current respiratory rate is 8 breaths/minute. What action should the nurse take? a- Provide oxygen 100% via facemask b- Check peripheral tendon reflexes c- Give another IV dose of morphine d- Administer Naloxone IV Rationale: naloxone, the antidote for morphine, is indicated for respiratory depression below 10 breath/minute A is indicative for oxygen saturation is low. B is not indicated at this time. Another dose of morphine within the prescribed time interval © can cause a further decline in the respiratory rate, which could be life-threatening. 80- Which intervention is most important for the nurse to include in the plan of care for an older woman with osteoporosis? a- Evaluate the client’s orientation to time and place b- Place the client on fall precautions c- Encourage the client to drink milk with meals d- Assess the client’s breath sounds daily. Rationale: Osteoporosis causes bone to become brittle, fragile and less dense with age, which increases an older client’s risk for falls and fractures which increases their risk for another pathology. 81- Based on the information provided in this client’s medical record during labor, which should the nurse implement? (Click on each chart tab for additional information. Please be sure to scroll to the bottom right corner of each tab to view all information contained in the client’s medical record) a- Apply oxygen 10 l/mask b- Stop the oxytocin infusion c- Turn the client to the right lateral position. d- Continue to monitor the progress of labor. Rationale: Early deceleration are indicative of head compression as the fetus descends in the birth canal, which is a normal patter during active labor, so labor progression should continue to be monitored 82- An unlicensed assistive personnel UAP leaves the unit without notifying the staff. In what order should the unit manager implement this intervention to address the UAPs behavior? (Place the action in order from first on top to last on bottom.) 1. Note date and time of the behavior. 2. Discuss the issue privately with the UAP. 3. Plan for scheduled break times. 4. Evaluate the UAP for signs of improvement. Rationale: Noting the date and time of the behavior is the first action that is important in providing factual information. The unit manager should discuss the behavior with the UAP and describe the problems the behaviors causes for the staff, when a problem is identified, it is important to plan and implement solutions, such as scheduled break times during the shift. These interventions should be evaluated based on the UAP’s signs of improvements. 83- A client with intestinal obstructions has a nasogastric tube to low intermittent suction and is receiving an IV of lactated Ringer’s at 100 ml/H. which finding is most important for the nurse to report to the healthcare provider? a- Gastric output of 900 mL in the last 24 hours b- Serum potassium level of 3.1 mEq/L or mmol/L (SI) c- Increased blood urea nitrogen (BUN) d- 24-hour intake at the current infusion rate. Rationale: The normal potassium level in the blood is 3.5-5.0 mill Equivalents per liter (mEq/L). 84- Which type of Leukocyte is involved with allergic responses and the destruction of parasitic worms? a- Neutrophils b- Lymphocytes c- Eosinophils d- Monocytes Rationale: Eosinophils are involved in allergic responses and destruction of parasitic worms. 85- Several months after a foot injury, and adult woman is diagnosed with neuropathic pain. The client describes the pain as severe and burning and is unable to put weight on her foot. She asks the nurse when the pain will “finally go away.” How should the nurse respond? a- Explain the healing from injury can take many months b- Assist the client in developing a goal of managing the pain. c- Encourage the client to verbalize her fears about the pain d- Complete an assessment of the client’s functional ability. Rationale: Neuropathic pain is chronic pain and the nurse should first help the client understand the need to learn to manage the pain. 86- One day following an open reduction and internal fixation of a compound fracture of the leg, a male client complains of “a tingly sensation” in his left foot. The nurse determines the client’s left pedal pulses are diminished. Based on these finding, what is the client’s greatest risk? a- Reduce pulmonary ventilation and oxygenation related to fat embolism. b- Neurovascular and circulation compromise related to compartment syndrome. c- Wound infection and delayed healing to fractured bone protrusion. d- Venous stasis and thrombophlebitis related to postoperative immobility. Rationale: Inflammation from the traumatic injury produces swelling and edema inside the closed space under the skin that produces pressure, which decreases blood flow to capillaries and nerves, causing altered perfusion related to compartment syndrome 87- The nurse is completing a head to be assessment for a client admitted for observation after falling out of a tree. Which finding warrants immediate intervention by the nurse? a- Sluggish pupillary response to light b- Clear fluid leaking from the nose. c- Complaint of severe headache d- Periorbital ecchymosis of right eye. Rationale: Clear fluid from nose or ear may be cerebrospinal fluid related to a basilar skull fracture and require immediate intervention. 88- A client with multiple sclerosis (MS) has decreased motor function after taking a hot bath (Uhthoff’s sign). Which pathophysiological mechanism supports this response? a- Arterial Constriction b- Temporary vasodilation c- Poor temperature control d- Severe dehydration. Rationale: Uhthoff’s sign results from temporary vasodilation 89- While assessing a radial artery catheter, the client complains of numbness and pain distal to the insertion site. What interventions should the nurse implement? a- Determine of aspirin was given prior to radial artery catheter insertion. b- Promptly remove the arterial catheter from the radial artery. c- Irrigate the arterial line using a syringe with sterile saline d- Administer a PRN analgesic and assess numbness in 30 mints Rationale: The client is manifesting evidence of sensory dysfunction and ischemia distal to the arterial catheter insertion site, so the arterial catheter should promptly be removed to minimize tissue necrosis. 90- A client is admitted with an epidural hematoma that resulted from a skateboarding accident. To differentiate the vascular source of the intracranial bleeding, which finding should the nurse monitor? a- Slow increasing intracranial pressure (ICP) b- Decerebrate posturing c- Rapid onset of decreased level of consciousness. d- Coup contrecoup signs Rationale: Epidural hematomas results from arterial bleeds that cause a rapid increase in ICP, which initially manifested by an early and rapid onset of decreased consciousness. Slowly increasing ICP is more likely to occur with a venous subdural hematoma. 91- The nurse finds a client at 33 weeks’ gestation in cardiac arrest. What adaptation to cardiopulmonary resuscitation (CPR) should the nurse implement? a- Apply oxygen by mask after opening the airway b- Position a firm wedge to support pelvis and thorax at 30-degree tilt. c- Give continuous compression with a ventilation ratio at 20:3 d- Apply less compression force to reduce aspiration Rationale: To relieve aortocaval compression caused by the gravid uterus, left lateral uterine displacement (LUD) should be maximized using a firm wedge to support the pelvis and thorax at 30- degree tilt to optimize maternal hemodynamic during CPR. Maternal modification should include ventilation with 100% oxygen, not A. Pregnant adults should be resuscitated using a compression-ventilation ration of 30:2 not C without interruption of continuous compressions. Effective chest compression should be forceful rhythmic application of pressure (fast and hard) at 100 compressions/minutes at the depth of 2 inches (5cm) to generate myocardial and cerebral blood flow. 92- When preparing a client for discharge from the hospital following a cystectomy and a urinary diversion to treat bladder cancer, which instruction is most important for the nurse to include in the client’s discharge teaching plan? a- Report any signs of cloudy urine output. b- Seek counseling for body image concerns c- Follow instruction for self-care toileting d- Frequently empty bladder to avoid distension. Rationale Infection can be life-threatening and cloudy urine output is a sign of urinary tract infection, which should be reported immediately. 93- For the past 24 hours, an antidiarrheal agent, diphenoxylate, has been administered to a bedridden, older client with infectious gastroenteritis. Which finding requires the nurse to take further action? a- Loss of appetite b- Serum K 4.0 mEq/or mmol/dl (SI) c- Loose, runny stool d- Tented skin turgor. Rationale: D indicate dehydration, a serious complication following prolonged diarrhea that requires further interventions by the nurse. 94- After repositioning an immobile client, the nurse observes an area of hyperemia. To assess for blanching, what action should the nurse take? a- Note the skin color around the area b- Measure the degree of… c- Apply light pressure over the area. d- Palpate the temperature of the area. Rationale: To assess for blanching the nurse should apply pressure to the area of hyperemia with one finger and when the finger is removed evaluate for return of erythema (blanching hyperemia) 95- The nurse enters a client’s room and observes the client’s wrist restraint secured as seen in the picture. What action should the nurse take? a- Use a full knot to secure the restrain tie. b- Reposition the restraint tie onto the bedframe. c- Raise the button side rail of the client’s bed d- Document that the restrain is secured. Rationale: Restraints should be secured to the bedframe, which is more stable than the side rails. A is difficult to release quickly. The restrain should be removed from the side rail before the position of the side rail is changed. 96- A female client with acute respiratory distress syndrome (ARDS) is chemically paralyzed and sedated while she is on as assist-control ventilator using 50% FIO2. Which assessment finding warrants immediate intervention by the nurse? a- Premature atrial contractions (PAC) b- Hemoccult-positive nasogastric fluid c- Diminished left lower lobe sounds. d- Increasing endotracheal secretions. Rationale: Diminished lobe sounds indicate collapsed alveoli or tension pneumothorax, which required immediate chest tube insertion to re-inflate the lung. A and B are common complications that result from increased stress in all body systems but are not immediately life threatening. 97- The development of atherosclerosis is a process of sequential events. Arrange the pathophysiological events in orders of occurrence. (Place the first event on top and the last on the bottom) 1. Arterial endothelium injury causes inflammation 2. Macrophages consume low density lipoprotein (LDL), creating foam cells 3. Foam cells release growth factors for smooth muscle cells 4. Smooth muscle grows over fatty streaks creating fibrous plaques 5. Vessel narrowing results in ischemia. Rationale: Atherosclerosis is triggered by injury to artery endothelium causing inflammation that attracts macrophages, which consume LDL, creating foam cells…. 98- Following a motor vehicle collision, an adult female with a ruptured spleen and a blood pressure of 70/44, had an emergency splenectomy. Twelve hours after the surgery, her urine output is 25 ml/hour for the last two hours. What pathophysiological reason supports the nurse’s decision to report this finding to the healthcare provider? a- This output is not sufficient to cleat nitrogenous waste b- Oliguria signals tubular necrosis related to hypoperfusion c- Low urine output puts the client at risk for fluid overload d- An increased urine output is expected after splenectomy Rationale: Prolonged low blood pressure leads to renal ischemia, which is the common etiology of acute tubular necrosis(ATN) Decreasing urine output is an early indicator of ATN. 99- A nurse-manager is preparing the curricula for a class for charge nurses. A staffing formula based on what data ensures quality client care and is most cost-effective? a- Client geographic location and age b- Number of staff and number of clients c- Weekend and weekday staff availability d- Skills of staff and client acuity Rationale: D considers both client need and staff competence. 100- When performing postural drainage on a client with Chronic Obstructive Pulmonary Disease (COPD), which approach should the nurse use? a- Perform the drainage immediately after meals b- Instruct the client to breath shallow and fast c- Obtain arterial blood gases (ABG’s) prior to procedure d- Explain that the client may be placed in five positions Rationale: Frequently, the client is placed in five positions (head down, prone, right and left lateral, and sitting upright) to aid in drainage of each of the five lobes of the lungs (D). Postural drainage should be performed before meals to prevent nausea, vomiting and aspiration(A). The client should breath slow and exhale through pursed lips to help keep airway open so that secretions can be drained while assuming the various positions. C is not required 101- A client presents in the emergency room with right-sided facial asymmetry. The nurse asks the client to perform a series of movements that require use of the facial muscles. What symptoms suggest that the client has most likely experience a Bell’s palsy rather than a stroke? a- Slow onset of facial drooping associated with headache b- Inability to close the affected eye, raise brow, or smile c- A flat nasolabial fold on the right resulting in facial asymmetry. d- Drooling is present on right side of the mouth, but not on the left. Rationale: Because the motor function controlling eye closure, brow movement and smiling are all carried on the 7th cranial (facial) nerve, the combination of symptoms directly relating to an impairment of all branches of the facial nerve indicate that Bell’s palsy has occurred. 102- The nurse is teaching a client how to perform colostomy irrigations. When observing the client’s return demonstration, which action indicated that the client understood the teaching? a- Turns to left the side to instill the irrigating solution into the stoma b- Keeps the irrigating container less than 18 inches above the stoma c- Instills 1,200 ml of irrigating solution to stimulate bowel evacuation d- Inserts irrigating catheter deeper into stoma when cramping occurs Rationale: Keeping the irrigating container less than 18 inches above the stoma permits the solution to flow slowly with little excessive peristalsis does not cause immediate release of stool. 103- The nurse should teach the client to observe which precaution while taking dronedarone? a- Stay out of direct sunlight b- Avoid grapefruits and its juice c- Reduce the use of herbal supplements d- Minimize sodium intake. Rationale: Grapefruit increase the effect of dronedarone thereby increasing the possibility of serious side effects. A does not cause a serious effect. C may potentiate lethal arrhythmias and should be avoided. D does not directly affect those taking dronedarone. 104- A client who sustained a head injury following an automobile collision is admitted to the hospital. The nurse includes the client’s risk for developing increased intracranial pressure (ICP) in the plan of care. Which signs indicate to the nurse that ICP has increased? a- Increased Glasgow coma scale score. b- Nuchal rigidity and papilledema. c- Confusion and papilledema d- Periorbital ecchymosis. Rationale: papilledema is always an indicator of increased ICP, and confusion is usually the first sign of increased ICP. Other options do not necessarily reflect increased ICP. 105- The nurse is caring for a client receiving continuous IV fluids through a single lumen central venous catheter (CVC). Based on the CVC care bundle, which action should be completed daily to reduce the risk for infection? a- Remind staff to follow protective environment precautions b- Gently flush the catheter lumen with sterile saline solution c- Cleanse the site and change the transparent dressing. d- Confirm the necessity for continued use of the CVC. Rationale: Increase the length of use increase the risk for infection. The CVC care bundle includes the review of the need for continued use of the CVC. Effective hand hygiene and standard precautions should be maintained but protective environment precautions are not needed. B is not needed if continuous IV fluid are infused, ad may introduce contaminants. Use of a transparent dressing allows the site to be visualized for any signs of infection but changing the dressing daily increases the risk for infection © 106- During an annual physical examination, an older woman’s fasting blood sugar (FBS) is determined to be 140 mg/dl or 7.8 mmol/L (SI). Which additional finding obtained during a follow-up visit 2 weeks later is most indicative that the client has diabetes mellitus (DM)? a- An increased thirst with frequent urination b- Blood glucose range during past two weeks was 110 to 125 mg/dl or 6.1 to 7.0 mmol/L(SI) c- Two-hour postprandial glucose tolerance test (GTT) is 160 mg/dL or 8.9 mmol/L (SI) d- Repeated fasting blood sugar (FBS) is 132 mg/dl or 7.4 mmol/L (SI). Rationale: FBS grater that 126 mg/dL or 7.0 mmol/L (SI) glucose (normal FBS range 70-110 mg/dL or 3.9 to 6.1 mmol/L (SI)) on at least two occasions is most diagnostic for DM. Classic symptoms of polyphagia, polydipsia, and polyuria may not be present in type…. 107- A new mother tells the nurse that she is unsure if she will be able to transition into parenthood. What action should the nurse take? a- Provide reassurance to the client that these feeling are normal after delivery b- Discuss delaying the client’s discharge from the hospital for another 24 hrs. c- Determine if she can ask for support from family, friend, or the baby’s father. d- Explain the differences between postpartum blues and postpartum depression. Rationale: Emotional support of significant family and friends can help a new mother cope with anxiety about transitioning to parenthood. The nurse should ask the client who is available to support her. 108- A client who was admitted yesterday with severe dehydration is complaining of pain a 24-gauge IV with normal saline is infusing at a rate of 150 ml/hour. Which intervention should the nurse implement first? a- Establish the second IV site b- Asses the IV for blood return c- Stop the normal saline infusion. d- Discontinue the 24-gauge IV Rationale: If the IV has infiltrated or become dislodges, the fluid is infusing into surrounding tissue and not into the vein. Stopping the infusion C is the priority action. Establishing another IV site is necessary for fluid resuscitation after the infiltrated infusion is discontinuing the IV (D) is necessary due to the pain, and a large gauge needle is preferable. 109- An elderly female is admitted because of a change in her level of sensorium. During the evening shift, the client attempts to get out bed and falls, breaking her left hip. Buck’s skin traction is applied to the left leg while waiting for surgery. Which intervention is most important for the nurse to include in this client’s plan care? a- Evaluate her response to narcotic analgesia b- Asses the skin under the traction moleskin c- Place a pillow under the involved lower left leg d- Ensure proper alignment of the leg in traction. Rationale: A fractured hip results in external rotation and shortening of the affected extremity. With the application of Buck’s skin traction proper alignment ensures the transaction S pull is exerted to align the fracture hip with the distal leg, immobilize the fractured bone, and minimize muscle spasms and surrounding tissue injury related to the fracture. A should be implement but improper pull of traction can increase pain and soft tissue damage. B and C should be implemented but the greatest risk is improper alignment of the traction. 110- An Unna boot is applied to a client with a venous stasis ulcer. One week later, when the Unna boot is removed during a follow-up appointment, the nurse observes that the ulcer site contains bright red tissue. What action should the nurse take in response to this finding? a- Immediately apply a pressure dressing b- Document the ongoing wound healing. c- Irrigate the wound with sterile saline d- Obtain a capillary INR, measurement Rationale: Appearance of granulation tissue is the best indicator of increased venous retuns and ongoing wound healing 111- At the end of a preoperative teaching session on pain management techniques, a client starts to cry and states, “I just know I can’t handle all the pain.” What is the priority nursing diagnosis for this client? a- Knowledge deficit b- Anxiety c- Anticipatory grieving d- Pain (acute) Rationale: The client is demonstrating only anxiety. There is no indication that the client is presenting signs of A, C or D 112- The nurse notes a visible prolapse of the umbilical cord after a client experiences spontaneous rupture of the membranes during labor. What intervention should the nurse implement immediately? a- Administer oxygen by face mask at 6L/mint b- Transport the client for a cesarean delivery c- Elevate the presenting part off the cord. d- Place the client to a knee-chest position. Rationale: The nurse should immediately elevate the presenting part off the cord because when the cord prolapses, the presenting part applies pressure to the cord, especially during each contraction, and reduces perfusion to the fetus. A can be delayed until pressure is removed from the cord. B and D are important but do not have priority. 113- A client who had a right hip replacement 3 day ago is pale has diminished breath sound over the left lower lung fields, a temperature of 100.2 F, and an oxygen saturation rate of 90%. The client is scheduled to be transferred to a skilled nursing facility (SNF) tomorrow for rehabilitative critical pathway. Based on the client’s symptoms, what recommendation should the nurse give the healthcare provider? a- Reassess readiness for SNF transfer. b- Obtain specimens for culture analysis c- Confer with family about home care plans d- Arrange physical therapy for strengthening. Rationale: Based on the client’s symptoms, reassessing the client’s readiness for rehabilitation in the SNF is critical 114- A client who is newly diagnosed with type 2 diabetes mellitus (DM) receives a prescription for metformin (Glucophage) 500 mg PO twice daily. What information should the nurse include in this client’s teaching plan? (Select all that apply.) a- Take an additional dose for signs of hyperglycemia b- Recognize signs and symptoms of hypoglycemia. c- Report persist polyuria to the healthcare provider. d- Use sliding scale insulin for finger stick glucose elevation. e- Take Glucophage with the morning and evening meal. Rationale: Glucophage, an antidiabetic agent, acts by inhibiting hepatic glucose production and increases peripheral tissue sensitivity to insulin. The client and family should be taught to recognize signs and symptoms of hypoglycemia. If the dose of Glucophage is inadequate, signs of hypoglycemia, such as polydipsia and polyuria, should be reported to the healthcare provider. Glucophage should be taken with meals to reduce GI upset and increase absorption (E). 115- The nurse is developing an educational program for older clients who are being discharged with new antihypertensive medications. The nurse should ensure that the educational materials include which characteristics? Select all that apply a- Written at a twelfth-grade reading level b- Contains a list with definitions of unfamiliar terms c- Uses common words with few Syllables d- Printed using a 12-point type font e- Uses pictures to help illustrate complex ideas Rationale: During the aging process older clients often experience sensory or cognitive changes, such as decreased visual or hearing acuity, slower thought or reasoning processes, and shorter attention span. Materials for this age group should include at least of terms, such as a medical terminology that incline may not know and use common words that expresses information clearly and simply. Simple, attractive pictures help hold the learner’s attention. The reading level of material should be at the 4th to 5th grade level. Materials should be printed using large font (18-point or higher), not the standard 12-point font. 116- During the admission assessment, the nurse auscultates heart sounds for a client with no history of cardiovascular disease. Where should the nurse listen when assessing the client’s point of maximal impulse (PMI) (Click the chosen location. To change, click on a new location) a- 117- An older male adult resident of long-term care facility is hospitalized for a cardiac catheterization that occurred yesterday. Since the procedure was conducted, the client has become increasingly disoriented. The night shift nurse reports that he attempted to remove the sandbag from his femoral artery multiple times during the night. What actions should the nurse take? (Select all that apply.) a- Recommend a 24-hour caregiver on discharge to the long-term facility. b- Notify the healthcare provider of the client’s change in mental status. c- Include q2 hour’s reorientation in the client’s plan of care. d- Request immediate evaluation by Rapid Response Team e- Apply soft wrist restraints so that the operative site is protected. Rationale: The client’s condition reflects mental changes that could be related to post procedure stress, sundowner’s syndrome, or cerebral complications, the nurses should inform the healthcare provider of the client’s change in mental status for the client’s safety, q2 hour orientation evaluations and reorientation should be included in the plan of care. 118- An older male comes to the clinic with a family member. When the nurse attempts to take the client’s health history, he does not respond to questions in a clear manner. What action should the nurse implement first? a- Ask the family member to answer the questions. b- Provide a printed health care assessment form c- Assess the surroundings for noise and distractions. d- Defer the health history until the client is less anxious. Rationale: Older client may be distracted from hearing and understanding by surrounding noise and the nurse should first confirm that noise and distraction have been minimized…. 119- The nurse caring for a client with acute renal fluid (ARF) has noted that the client has voided 800 ml of urine in 4 hours. Based on this assessment, what should the nurse anticipate that client will need? a- Treatment for acute uremic symptoms within 24 hours b- Change to a regular diet c- Large amounts of fluid and electrolyte replacement. d- Unrestricted sodium intake Rationale: This client, whose output is significantly high will need fluids and electrolyte replacement. The diuretic stage of ARF begins when the client has greater than 500 ml of urine in 24 hrs. A is associated with the oliguric and anuric stage of ARF. B and D should not occur until the client’s BUN and electrolytes indicate a significant improvement that will allow for such changes. 120- Which intervention should the nurse include in the plan of care for a child with tetanus? a- Open window shades to provide natural light b- Reposition side to side every hour. c- Minimize the number of stimuli in the room. d- Encourage coughing and deep breathing Rationale: Tetanus is an acute, preventable, and often fatal disease caused by an exotoxin produces by the anaerobic spore forming gram positive bacillus clostridium tetani, which affect neuromuscular junction and causes painful muscular rigidity. In planning caring for a child with tetanus, any environmental stimulation should be minimized. 121- Suicide precautions are initiated for a child admitted to the mental health unit following an intentional narcotic overdose. After a visitor leaves, the nurse finds a package of cigarettes in the client’s room. Which intervention is most important for the nurse to implement? a- Assign a sitter for constant observation b- Screen future visitors for contraband c- Document suicide monitoring frequently d- Remove cigarettes for the client’s room. Rationale: Safety is the priority, and any items that might cause self-harm, such as cigarettes should be removed immediately to create a safe environment. 122- A family member of a frail elderly adult asks the nurse about eligibility requirements for hospice care. What information should the nurse provide? (Select all that apply.) a- All family must agree about the need for hospice care. b- Hospice services are covered under Medicare Part B. c- A client must be willing to accept palliative care, not curative care. d- The healthcare provider must project that the client has 6 months or less to live. e- All medications except pain treatment will be stopped during hospice care. Rationale: The eligibility criteria for Medicare coverage requires that the client is willing to accept palliative care, not curative care (C). The healthcare provider should provide an expected prognosis of 6 months or less to live (D) which can be extended by the healthcare provider. It is not necessary for all family members to agree with the need for hospice. 123- A client with atrial fibrillation receives a new prescription for dabigatran. What instruction should the nurse include in this client’s teaching plan? a- Keep an antidote available in the event of hemorrhage b- Continue obtaining scheduled laboratory bleeding test c- Eliminate spinach and other green vegetable in the diet. d- Avoid use of nonsteroidal ant-inflammatory drugs (NSAID). Rationale: Dabigatran, a directed reversible thrombin inhibitor, is prescribe to reduce the risk of stroke in client with atrial fibrillation. The risk of bleeding and GI event can be significant and the concomitant use of NSAID and other anticoagulants should be avoided. 124- A nurse with 10 years’ experience working in the emergency room is reassigned to the perinatal unit to work an 8-hour shift. Which client is best to assign to this nurse? a- A client who is leaking clear fluid b- A mother who just delivered a 9 pounds boy c- A mother with an infected episiotomy. d- A client at 28- weeks’ gestation in pre-term labor. Rationale: An infected episiotomy is essentially an infected surgical incision, and an experienced emergency room nurse is likely be able to care for such a client. A, B and D required specialized maternity nursing care. 125- An infant who is admitted for surgical repair of a ventricular septal defect (VSD) is irritable and diaphoretic with jugular vein distention. Which prescription should the nurse administer first? a- Spironolactone b- Potassium c- Ampicillin sodium parental d- Digoxin. Rationale: This infant is demonstrating early signs of heart failure due to an increase right ventricular workload caused by a left to right shunt through the VSD, son an inotropic, such as digoxin should be administered first to improve the efficiency of myocardial contractility. Next a high ceiling diuretic to reduce fluid volume and workload of the heart. If hypokalemia occurs as result of potassium-wasting diuretic, should be given to reduce the risk of digoxin toxicity. 126- The nursing staff on a medical unit includes a registered nurse (RN), practical nurse (PN), and an unlicensed assistive personnel (UAP). Which task should the charge nurse assign to the RN? a- Supervise a newly hired graduate nurse during an admission assessment. b- Transport a client who is receiving IV fluids to the radiology department. c- Administer PRN oral analgesics to a client with a history of chronic pain d- Complete ongoing focused assessments of a client with wrist restrain. Rationale: The admission assessment of a client should be completed by a professional nurse. A graduate nurse should be supervised by the RN to ensure that the graduate nurse understand and performs within the expected scope of practice. The UAP transport a stable client. (B) The PN can complete C and D 127- While teaching a young male adult to use an inhaler for his newly diagnosed asthma, the client stares into the distance and appears to be concentrating on something other than the lesson the nurse is presenting. What action should the nurse take? a- Remind the client that a rescue inhaler might save his life b- Leave the client alone so that he can grieve his illness c- Ask the client what he is thinking about at his time. d- Gently touch the client then continue with teaching. Rationale: Asking the client what is thinking about right now provides the nurse an opportunity to assess him for denial, frustration, grief, or disinterest in learning about self-care of his asthma and use of the inhaler…. 128- After several hours of non-productive coughing, a client presents to the emergency room complaining of chest tightness and shortness of breath. History includes end stage chronic obstructive pulmonary disease (COPD) and diabetes mellitus. While completing the pulmonary assessment, the nurse hears wheezing and poor air movement bilaterally. Which actions should the nurse implement? (Select all that apply.) a- Apply oxygen via nasal cannula b- Administer PRN nebulizer treatment. c- Obtain 12 lead electrocardiogram. d- Monitor continuous oxygen saturation. e- Give PRN dose of regular insulin Rationale: A nebulizer treatment may improve the wheezing. Chest tightness is most likely to coughing, but a 12-lead electrocardiogram is needed to assess for cardiac ischemia. Oxygen saturation monitors for adequate oxygenation. 129- The nurse caring for a 3-month-old boy one day after a pylorotomy notices that the infant is restless, is exhibiting facial grimaces, and is drawing his knees to his chest. What action should the nurse take? a- Administer a prescribed analgesia for pain. b- Increase IV infusion rate for rehydration c- Provide additional blankets to increase body temperature d- Feed one ounce of formula to correct hypoglycemia. Rationale: Since this child is exhibiting signs of pain, the prescribed analgesic should be administered. The behavioral signs of pain in an infant are facial grimaces, restlessness or agitation, and guarding the area of pain, in this case by pulling the knees to the chest 130- Two days after admission a male client remembers that he is allergic to eggs, and informs the nurse of the allergy. Which actions should the nurse implement? (Select all that apply) a- Notify the food services department of the allergy. b- Enter the allergy information in the client’s record. c- Document the statement in the nurse’s notes d- Note the allergy on the diet intake flow sheet e- Add egg allergy to the client’s allergy arm band. Rationale: The dietary department needs to screen menu selections for foods that are prepared with eggs. The client’s chart should be clearly marked but the statement does not need to be documented in the nurse’s note or included in the intake record. Allergy identification on the arm band is a universal location where allergies are noted while client is hospitalized. 131- The rapid response teams detect return of spontaneous circulation (ROSC) after 2 min of continuous chest compressions. The client has a weak, fast pulse and no respiratory effort, so the healthcare provider performs a successful oral, intubation. What action should the nurse implement? a- Perform bilateral chest auscultation. b- Resume compression for 2 minutes c- Administer a dose of epinephrine d- Program the monitor for cardioversion. Rationale: With the ROSC and no respiratory effort intubation is indicated, and as soon as the procedure is completed, the position of the intubation tube should be assessed for proper placement. Auscultating for breath sounds is the first and quickest method to use to check for proper placement of the intubation tube and can be confirmed by a chest x ray. 132- After administering an antipyretic medication. Which intervention should the nurse implement? a- Encouraging liberal fluid intake 133- A client with hyperthyroidism is being treated with radioactive iodine (I-131). Which explanation should be included in preparing this client for this treatment? a- Explain the need for using lead shields for 2 to 3 weeks after the treatment b- Describe the signs of goiter because this is a common side effect of radioactive iodine c- Explain that relief of the signs/symptoms of hyperthyroidism will occur immediately d- Describe radioactive iodine as a tasteless, colorless medication administered by the healthcare provider Rationale: A single dose of tasteless, colorless radioactive iodine is administered by mouth and the client is observed for signs of thyroid storm. 85% of clients are cure by one dose. The dose of radioactivity is not enough to warrant (A). B is indicated for a client receiving iodine or iodine compound medications in the treatment of hyperthyroidism. It takes 3 to 4 weeks for sings of hyperthyroidism to subside. 134- After a colon resection for colon cancer, a male client is moaning while being transferred to the Post anesthesia Care Unit (PACU). Which intervention should the nurse implement first? a- Assess the client’s dressing for bleeding b- Determine client’s pulse, blood pressure, and respirations c- Administer a PRN dose of IV Morphine d- Check the client’s orientation to time and place. Rationale: Colon resection, a major abdominal surgical procedure, causes severe pain in the immediate postoperative period and requires administration of IV morphine regularly to maintain analgesic serum level. Before administering a central nervous system depressing analgesia, the client’s vital signs should be assessed to determine the client’s current level of CNS depression. In the immediate postoperative period, during administration to PACU (A, C and D) should be evaluated. 135- The nurse is caring for a group of clients with the help of a licensed practical nurse (LPN) and an experienced unlicensed assistive personnel (UAP). Which procedures can the nurse delegate to the UAP? (Select all that apply) a- Change a saturated surgical dressing for a client who had an abdominal hysterectomy. b- Take postoperative vital signs for a client who has an epidural following knee arthroplasty c- Start a blood transfusion for client who had a below-the knee amputation. d- Collect a sputum specimen for a client with a fever of unknown origin e- Ambulate a client who had a femoral-popliteal bypass graft yesterday Rationale: Measuring vital signs, collecting specimens, and ambulating a mobile client are within the scope of practice for a UAP 136- A male client with cirrhosis has ascites and reports feeling short of breath. The client is in semi Fowler position with his arms at his side. What action should the nurse implement? a- Reposition the client in a side-lying position and support his abdomen with pillows. b- Elevate the client’s feet on a pillow while keeping the head of the bed elevated. c- Raise the head of the bed to a Fowler’s position and support his arms with a pillow d- Place the client in a shock position and monitor his vital signs at frequent intervals. Rationale: The Ascites is the accumulation of fluid in the peritoneal or abdominal cavity, and this fluid pushes on the diaphragm, limiting the client’s lung expansion and causing dyspnea. To relieve pressure, the head of the bed should be elevated with the arms supported for comfort. 137- A client with a chronic health problem has difficulty ambulating short distance due to generalized weakness but can bear weight on both legs. To assist with ambulation and provide the greatest stability, what assistive device is best for this client? a- A quad cane b- Crutches with 2-point gait. c- Crutches with 3-point gait. d- Crutches with 4-point gait. Rationale: Crutches using a 4-point gait provide stability and require weight bearing on both legs, which this client should be able to provide. A is used when is partial or complete leg paralysis or some hemiplegia. B requires at least partial weight bearing on each foot but does not provide the stability of D. C is useful when the client must bear all the weight on one foot and this is not the problem experienced by this client. 138- The nurse uses the parkland formula (4ml x kg x total body surface area = 24 hours’ fluid replacement) to calculate the 24-hours IV fluid replacement for a client with 40% burns who weighs 76kg. How many ml should the client receive? (Enter numeric value only.) a- Answer: 12160 Rationale: 4ml x 76kg x 40 (bsa) =12,160 ml 139- A client with leukemia undergoes a bone marrow biopsy. The client’s laboratory values indicate the client has thrombocytopenia. Based on this data, which nursing assessment is most important following the procedure? a- Assess body temperature b- Monitor skin elasticity c- Observe aspiration site. d- Measure urinary output 140- An 18-year-old female client is seen at the health department for treatment of condyloma acuminate (perineal warts) caused by the human papillomavirus (HPV). Which intervention should the nurse implement? a- Tell the client that the vaccine for HPV is not indicated b- Inform the client that warts do not return following cryotherapy c- Recommended the use of latex condoms to prevent HPV transmission. d- Reinforce the importance of annual papanicolaou (Pap) smears. Rationale: Because the HPV is associated with cervical cancer, close follow-up, including yearly Pap smears should be recommended. Although the HPV quadrivalent (types 6, 11, 16 and 18) vaccine, recombinant does not work as well for… 141- A client admitted to the psychiatric unit diagnosed with major depression wants to sleep during the day, refuses to take a bath, and refuses to eat. Which nursing intervention should the nurse implement first? a- Assess the client’s ability to communicate with the other staff members b- Arrange a meeting with the family to discuss the client’s situation c- Administer the client’s antidepressant medication as prescribed. d- Establish a structured routine for the client to follow. 142- A client with history of bilateral adrenalectomy is admitted with a week, irregular pulse, and hypotension. Which assessment finding warrants immediate intervention by the nurse? a- Decrease urinary output b- Low blood glucose level c- Profound weight gain d- Ventricular arrhythmias. Rationale: adrenal crisis, a potential complication of bilateral adrenalectomy, results in the loss of mineralocorticoids and sodium excretions that is characterized by hyponatremia, hyperkalemia, dehydration, and hypotension. Ventricular arrhythmias are life threatening and required immediate intervention to correct critical potassium levels. 143- The mother of a 7-month-old brings the infant to the clinic because the skin in the diaper area is excoriated and red, but there are no blisters or bleeding. The mother reports no evidence of watery stools. Which nursing intervention should the nurse implement? a- Instruct the mother to change the child’s diaper more often. b- Encourage the mother to apply lotion with each diaper charge c- Tell the mother to cleanse with soap and water at each diaper change d- Ask the mother to decrease the infant’s intake of fruits for 24 hours. Rationale: Changing the diaper more often helps to decrease the amount of time the skin comes in contact with wet soiled diapers and helps heal the irritation. 144- A resident of a long-term care facility, who has moderate dementia, is having difficulty eating in the dining room. The client becomes frustrated when dropping utensils on the floor and then refuses to eat. What action should the nurse implement? a- Allow client to choose foods from a menu b- Assign a staff member to feed the client c- Have meals brought to the client’s room d- Encourage the client to eat finger foods. Rationale: Eyes-hand coordination is often affected with dementia. Providing a way to eat without using utensils is likely to help the client maintain independence while obtaining adequate nutrition. A: increase frustration. 145- A client is receiving mesalamine 800 mg PO TID. Which assessment is most important for the nurse to perform to assess the effectiveness of the medication? a- Pupillary response b- Oxygen saturation c- Peripheral pulses d- Bowel patterns Rationale: the client should be assessed for a change in bowel patterns to evaluate the effectiveness of this medication because Mesalamine is used to treat ulcerative colitis (a condition which causes swelling and sores in the lining of the colon [large intestine] and rectum) and to maintain improvement of ulcerative colitis symptoms. Mesalamine is in a class of medications called anti-inflammatory agents. It works by stopping the body from producing a certain substance that may cause inflammation. 146- While in the medical records department, the nurse observes several old medical records with names visible in waste container. What action should the nurse implement? a- Place the records in a separate trash bag and tie the bag securely closed b- Point out the record to a worker in the medical records department c- Contact the medical records department supervisor. d- Immediately remove and shred the records. Rationale: Notify the department supervisor of a Privacy officer alerts the appropriate people to a possible internal procedural problem and provides an opportunity of education a prevention of recurrence. (C) 147- A 16-year-old adolescent with meningococcal meningitis is receiving a continuous IV infusion of penicillin G, which is prescribed as 20 million units in a total volume of 2 liters of normal saline every 24 hr. The pharmacy delivers 10 million units/ liters of normal saline. How many ml/hr should the nurse program the infusion pump? (Enter numeric value only. If rounding is required, round to the nearest whole number.) a- Answer 83 Rationale: 1000 ml-----12hr. Xml ---------1hr. 1000/12 = 83.33 = 83. 148- While visiting a female client who has heart failure (HF) and osteoarthritis, the home health nurse determines that the client is having more difficulty getting in and out of the bed than she did previously. Which action should the nurse implement first? a- Inquire about an electric bed for the client’s home use b- Submit a referral for an evaluation by a physical therapist. c- Explain the usual progression of osteoarthritis and HF d- Request social services to review the client’s resources. Rationale: To promote independence and safety in the home, the client’s decline in physical mobility and strength should be evaluated first by the physical therapist who is a member if the home health treatment team. 149- A client has an intravenous fluid infusing in the right forearm. To determine the client’s distal pulse rate most accurately, which action should the nurse implement? a- Elevate the client’s upper extremity before counting the pulse rate b- Auscultate directly below the IV site with a Doppler stethoscope c- Turn off the intravenous fluids that are infusing while counting the pulse. d- Palpate at the radial pulse site with the pads of two or three fingers. Rationale: The radial pulse is easily accessible and palpable unless an IV is placed at the client wrist. A may make the pulse more difficult to palpate B places the stethoscope over a vein rather than an artery and is unlikely to provide an accurate pulse rate. The pulse rate can be accurately counted without implementing. 150- A child is admitted to the pediatric unit diagnosed with sickle cell crisis. When the nurse walks into the room, the unlicensed assistive personnel (UAP) is encouraging the child to stay in bed in the supine position. Which action should the nurse implement? a- Reposition the client with the head of the bed elevated. b- Commend the UAP for implementing the proper position c- Tell the UAP that this position is harmful to the client d- Encourage the child to ambulate in the room Rationale: Since children is sickle cell crisis often have shallow breathing due to acute chest syndrome, raising the head of the bed (A) will facilitate chest expansion by decreasing pressure of the diaphragm (B and C) are not be commended, nor should he UAP be corrected in front of the child. D is contraindicated because bed rest is warranted to conserve energy and promote oxygenation. 151- A preschool-aged boy is admitted to the pediatric unit following successful resuscitation from a near-drowning incident. While providing care to child, the nurse begins talking with his preadolescent brother who rescued the child from the swimming pool and initiated resuscitation. The nurse notices the older boy becomes withdrawn when asked about what happened. What action should the nurse take? a- Ask the older brother how he felt during the incident. b- Commend the older brother for his heroic actions c- Tell the older brother that he seems depressed d- Develop a water safety teaching plan for the family. Rationale: The brother’s change in demeanor may indicate that he is experiencing post-traumatic stress that warrants further investigation, so the nurse should address the older brother’s feeling. 152- After six days on a mechanical ventilator, a male client is extubated and place on 40% oxygen via face mask. He is awake and cooperative but complaining of a severe sore throat. While sipping water to swallow a medication, the client begins coughing, as if strangled. What intervention is most important for the nurse to implement? a- Administer PRN medication b- Titrate the oxygen to keep saturation above 92% c- Hold oral intake until swallow evaluation is done. d- Elevate the head of his bed at least 45 degrees. Rationale: After oral intubation, the client is at high risk for swallowing difficulties. A swallowing evaluation should be done to determine what consistency of liquids the client can tolerate without aspirating. A, B and D helps but does not have the priority. 153- The nurse is interacting with a female client who is diagnosed with postpartum depression. Which finding should the nurse document as objective signs of depression? (Select all that apply) a- Report feeling sad b- Interacts with a flat affect. c- Avoids eye contact. d- Has a disheveled appearance. e- Express suicidal thoughts. Rationale: Observed finding are objective and include the client’s appearance, such as flat affect, lack of eyes contact, and disheveled appearance. A and E are subject only the client can express verbally. 154- A client in the post anesthesia care unit (PACU) has an eight (8) on the Aldrete post anesthesia scoring system. What intervention should nurse implement? a- The client should be kept in the recovery room b- Assess the client’s respiratory status immediately c- Notify the client’s surgeon immediately d- Transfer the client to the surgical floor. Rationale: A score of 7 to 8 is normal and indicates that the client can be discharge from PACU. The PACU assessment form includes 5 mints areas of assessment: muscle activity, circulation, consciousness level, and oxygen saturation. Each of these 5 areas receives two points for normal. A, B, C are interventions that are not indicated for a score of 8. 155- In caring for the body of a client who just died, which tasks can be delegate to the unlicensed assistive personnel (UAP)? (Select all that apply.) a- Place personal religious artifacts on the body. b- Confirm the client’s wishes for tissue donation c- Observe consent for autopsy signature by family. d- Attach identifying name tags to the body. e- Follow cultural beliefs in preparing the body. Rationale: With adequate supervision and instruction (A, D and E) can be delegate to the UAP. 156- An adult male report the last time he received penicillin he developed a severe maculopapular rash all over his chest. What information should the nurse provide the client about future antibiotic prescriptions? a- Be alert for possible cross-sensitivity to cephalosporin agents. b- Monitor peak ad trough levels whenever taking any antibiotic c- Watch daily urine output and weight gain while taking antibiotics d- Wear sun block and protective clothing to avoid exposure to sun. Rationale: Cross-sensitivity with cephalosporin can occur in those who are allergic to penicillin, so the nurse should provide this warning. 157- A client with a prescription for “do not resuscitate” (DNR) begins to manifest signs of impending death. After notifying the family of the client’s status, what priority action should the nurse implement? a- The impending signs of death should be documented b- The client’s need for pain medication should be determined. c- The nurse manager should be updated on the client’s status d- The client’s status should be conveyed to the chaplain Rationale: Palliative care includes nursing interventions that provide relief for the dying client’s suffering by assessment and treatment of pain and other problems that are physical, psychosocial and spiritual. After the family is notified for the client’s impending death, the client’s need for pain medication should be assessed. 158- A client with cirrhosis of the liver is admitted with complications related to end stage liver disease. Which intervention should the nurse implement? (Select all that apply.) a- Monitor abdominal girth. b- Increase oral fluid intake to 1500 ml daily. c- Report serum albumin and globulin levels. d- Provide diet low in phosphorous. e- Note signs of swelling and edema. Rational: monitoring for increasing abdominal girth and generalized tissue edema and swelling are focused assessments that provide data about the progression of disease related complications. In advanced cirrhosis, liver function failure results in low serum albumin and serum protein levels, which caused third spacing that results in generalized fluid retention and ascites. Other options are not indicated in end stage liver disease. 159- During discharge teaching, the nurse discusses the parameters for weight monitoring with a client who was recently diagnosed with heart failure (HF). Which information is most important for the client to acknowledge? a- Keep a daily weight record b- Obtain weight at the same time every day c- Limit intake of dietary salt. d- Report weight gain of 2 pounds (0.9kg) in 24 hours 160- Which problem, noted in the client’s history, is important for the nurse to be aware of prior to administration of a newly prescribed selective serotonin reuptake inhibitor (SSRI)? a- Bulimia nervosa b- Obsessive compulsive disorder c- Aural migraine headaches. d- Erectile dysfunction. Rationale: A client with a history of migraine headache requires further assessment to determine if medications from 5-HT1 receptor agonist (Triptans), such as sumatriptan, are currently be taken. Triptans and SSRI combination could potentially cause the fatal drug-drug interaction serotonin syndrome. SSRI are specific for treatment of A and B, although erectile dysfunction is a side effect of SSRI’s. 161- When implementing a disaster intervention plan, which intervention should the nurse implement first? a- Initiate the discharge of stable clients from hospital units b- Identify a command center where activities are coordinated c- Assess community safety needs impacted by the disaster d- Instruct all essential off-duty personnel to report to the facility. Rationale: First, the command center should be identified so activities can be coordinated. 162- The nurse is evaluating a client’s symptoms, and formulates the nursing diagnosis, “high risk for injury due to possible urinary tract infection.” Which symptoms indicate the need for this diagnosis? a- Hematuria and proteinuria b- Azotemia and anorexia c- Fever and dysuria. d- Straining on urination and nocturia 163- A client is admitted with metastatic carcinoma of the liver, ascites, and bilateral 4+ pitting edema of both lower extremities. When the client complains that the antiembolic stocking are too constricting, which intervention should the nurse implement? a- Maintain both lower extremities elevated on pillows. b- Remove the contracting antiembolic stocking c- Administer diuretics in the morning hours d- Restrict PO fluid intake to 500 ml per shift Rationale: Hepatocellular failure and hypertension contribute to third spacing of fluids. The clients complain best addresses by maintaining both extremities in an elevated position on pillows, which uses gravity to facilitate venous return and decrease peripheral edema. Stockings should be reapplied evenly to relieve constriction, but no removed. 164- A client with muscular dystrophy is concerned about becoming totally dependent and is reluctant to call the nurse to assist with activities of daily living (ADLs). To achieve maximum mobility and independence, which intervention is most important for the nurse to include in the client’s plan of care? a- Elevate lower extremities while out of bed b- Teach family proper range of motion exercises. c- Maintain proper body alignment when in bed d- Encourage diaphragmatic breathing exercises. Rationale: Performing proper range of motion exercised helps maintain maximum mobility by preventing excessive muscle atrophy and joint contractures. Elevating lower extremities decreases the amount of peripheral edema. Proper body alignment reduces strain on joints, tendons, ligaments and muscles and minimizes contractures in an abnormal position. Diaphragmatic breathing exercises may decrease the risk of pulmonary complications. 165- The nurse is teaching a postmenopausal client about osteoporosis prevention. The client reports that she smokes 2 packs of cigarettes a day and takes 750 mg calcium supplements daily. What information should the nurse include when teaching this client about osteoporosis prevention? a- Family history is more important than calcium intake in determining the occurrence of osteoporosis b- Calcium should be taken once a day, preferable at the same time of day c- Smoking cessation is more important than calcium intake in preventing osteoporosis. d- Postmenopausal women need an intake of at least 1,500 mg of calcium daily. Rationale: A postmenopausal woman needs a minimum of 1,500 mg calcium daily, which is best taken in divided doses not just once a day. A and C are important factors in determining the occurrence of osteoporosis, they are not more important than calcium intake. 166- When evaluating a client’s rectal bleeding, which findings should the nurse document? a- Number of blood clots expelled with each stool. b- Unique odor noted with GI bleeding c- Evidence of internal hemorrhoids. d- Color characteristics of each stool. Rationale: Color characteristics indicate if blood is coming from high in the GI tract, which would be black and tarry, or from lower area near rectum, which would be bright red blood. 167- The nurse is auscultating a client’s lung sounds. Which description should the nurse use to document this sound? a- High pitched or fine crackles. b- Rhonchi c- High pitched wheeze d- Stridor 168- An adult male is admitted to the emergency department after falling from a ladder. While waiting to have a computed tomography (CT) scan, he requests something for a severe headache. When the nurse offers him a prescribed dose of acetaminophen, he asks for something stronger. Which intervention should the nurse implement? a- Request that the CT scan be done immediately b- Review the client’s history for use of illicit drugs c- Assess client’s pupils for their reaction to light. d- Explain the reason for using only non-narcotics. Rationale: The client needs to understand that any pain medication that can mask declining neurological symptoms, such as narcotics should be avoid. There is no indication that the CT scan needs to be done immediately. In the absent of additional information B is presumptive. Regular neurological assessment is necessary, but they do not address the client’s pain. 169- The nurse is managing the care of a client with Cushing’s syndrome. Which interventions should the nurse delegate to the unlicensed assistive personnel (UAP)? (Select all that apply) a- Evaluate the client for sleep disturbances b- Weigh the client and report any weight gain. c- Report any client complaint of pain or discomfort. d- Assess the client for weakness and fatigue e- Note and report the client’s food and liquid intake during meals and snacks. Rationale: B, C and E are functions within the scope of practice for the UAO include reporting client complaints of pain. 170- Ten years after a female client was diagnosed with multiple sclerosis (MS), she is admitted to a community palliative care unit. Which intervention is most important for the nurse to include in the client’s plan of care? a- Allow the family to visit whenever they wish b- Medicate as needed for pain and anxiety. c- Allow client to participate in care provided d- Maintain quiet, low lighting environment Rationale: Neuropathic pain in MS is related to damage to peripheral nerves or structures in the CNS can be sudden, intense or lingering, and shooting, electric shock-like sensations that results for paroxysmal firing of injured nerves. Once the client enters palliative care, the primary goal is comfort. 171- An increased number of elderly persons are electing to undergo a new surgical procedure which cures glaucoma. What effect is the nurse likely to note because of this increases in glaucoma surgeries? a- Decrease morbidity in the elderly population b- Decrease prevalence of glaucoma in the population. c- Increase mortality in the elderly population d- Increased incidence of glaucoma in the population. Rationale: Prevalence describe the number of existing causes of glaucoma. Since glaucoma occurs mostly in the elderly population and the elderly are obtaining the curative surgery, a decrease prevalence of glaucoma in the population at large can be expected. 172- The nurse is caring for a client who is entering the second stage of labor. Which action should the nurse implement first? a- Convey to the client that birth is imminent. b- Prepare the client for spinal anesthesia c- Empty the client’s bladder using a straight catheter d- Prepare the coach to accompany the client to delivery Rationale: The second stage of labor occurs when the client is fully dilated, and the fetus is crowning, so completing preparations and informing the client that birth is imminent, so A is the first action. B is usually administered immediately prior to delivery. C is usually performed prior of after delivery D is not the priority action at this time. 173- To evaluate the effectiveness of male client’s new prescription for ezetimibe, which action should the clinic nurse implement? a- Remind the client to keep his appointments to have his cholesterol level checked. b- Teach the client to weigh himself weekly and keep a log of the measurements c- Assess the elasticity of the client’s skin at the next scheduled clinic appointment d- Encourage the client to keep a diary of his food intake until his next visit to the clinic. Rationale: Ezetimibe lowers cholesterol and LDL levels, so it is important for the nurse to remind the clients to keep his appointments at the laboratory. D may influence his serum levels, but A provide better indicator. 174- Diagnostic studies indicate that the elderly client has decreased bone density. In providing client teaching, which area of instruction is most important for the nurse to include? a- Application of joint splints b- Effective body mechanisms c- Fall prevention measures. d- Low fat, high protein diet. Rationale: Client instruction should include measures to prevent falls, because elderly clients with decrease bone density are at high risk for bone fractures and impaired bone healing in fracture should occur. 175- A young adult client is admitted to the emergency room following a motor vehicle collision. The client’s head hit the dashboard. Admission assessment include: Blood pressure 85/45 mm Hg, temperature 98.6 F, pulse 124 beat/minute and respirations 22 breath/minute. Based on these data, the nurse formulates the first portion of nursing diagnosis as” Risk of injury” What term best expresses the “related to” portion of nursing diagnosis? a- Infection b- Increase intracranial pressure c- Shock d- Head Injury. Rationale: This client has symptoms of shock. Two signs of shock are decreased BP, and increased (often weak and thread) pulse, this client has both symptoms. A temperature of 98.6 F is average normal. An increase of temperature. D is correct but is vague and is not specifically related to the assessment date describe, so it is not the best answer. 176- An older male client with history of diabetes mellitus, chronic gout, and osteoarthritis comes to the clinic with a bag of medication bottles. Which intervention should the nurse implement first? a- Obtain a medical history b- Record pain evaluation c- Assess blood glucose d- Identify pills in the bag. Rationale: Comorbidity places the client at risk for multiple drug interaction and side effects, and the client’s gout therapy may need to be modified. A review of the medication in the bag (D) is the most important way to analyze the client’s polypharmacy. And therapeutic response for comorbidities. Obtain a medical history (A), pain evaluation (B), and assessing blood glucose level (C) should be done in a timely manner. 177- A male client who was diagnosed with viral hepatitis A 4 weeks ago returns to the clinic complaining of weakness and fatigue. Which finding is most important for the nurse to report to the healthcare provider? a- Dark yellow-brown colored urine b- Nonspecific muscle and joint pain c- New onset of purple skin lesions. d- Weakness when getting up to walk. Rationale: During the convalescence period of hepatitis A, the client major complain is malaise and fatigability. Purple skin lesions may be indicative of the liver’s impaired ability to produce clothing elements and should be reported to the healthcare provider (C) for further analysis. Urine may become dark when excess bilirubin is excreted by the kidney, which is expected even when the client is not jaundice during the acute phase hepatitis (A). Myalgia and arthralgia (B) are intermittent complains with ongoing malaise, fatigue and weakness (D) during convalescence of hepatitis A. 178- In assessing a client twelve hour following transurethral resection of the prostate (TURP), the nurse observes that the urinary drainage tubing contains a large amount of clear pale pink urine and the continuous bladder irrigation is infusing slowly. What action should the nurse implement? a- Increase the rate of the continuous bladder irrigation b- Manually irrigate the catheter with sterile normal saline c- Clam the catheter above the drainage. d- Ensure that no dependent loops are present in the tubing. Rationale: The nurse should ensure that the tubing is not kinked, and adequate flow is maintained to prevent bladder distention. Clear pale pink urine is desirable following TURP and indicates the absence of clots or excessive hemorrhage. A is implemented if the flow is dark red to prevent clot formation, and B if clots is present, to prevent obstruction. C is not a useful action in this situation and causes bladder distention while the bladder irrigation is still infusing 179- The healthcare provider prescribes the antibiotic Cephradine (cephalosporin) 300mg PO every 12 h for a client with postoperative wound infections. Which feeds should the nurse encourage this client to eat? a- Yogurt and/or buttermilk. b- Avocados and cheese c- Green leafy vegetables d- Fresh fruits Rationale: A should be encouraging to help maintain intestinal flora and decrease diarrhea, which is a common side effect of antibiotic therapy, particularly cephradine. B and C are contraindicated because they can increase bowel elimination, thereby exacerbating diarrhea as a side effect. 180- The charge nurse is making assignment on a psychiatric unit for a practical nurse (PN) and newly license register nurse (RN). Which client should be assigned to the RN? a- An adult female who has been depress for the past several months and denies suicidal ideations. b- A middle-age male who is in depressive phase on bipolar disease and is receiving Lithium. c- A young male with schizophrenia who said voices is telling him to kill his psychiatric. d- An elderly male who tell the staff and other client that he is superman and he can fly. Rationale: The RN should deal with the client with command hallucinations and these can be very dangerous if the client’s acts on the commands, especially if the command is a homicidal in nature. Other client present low safety risk. 181- A client at 30-week gestation is admitted due to preterm labor. A prescription of terbutaline sulfate 8.35 mg is gives subcutaneously. Based on which finding should the nurse withhold the next dose of this drug? a- Maternal blood pressure of 90/60 b- Fetal heart rate of 170 beats per minute for 15 mints c- Maternal pulse rate of 162 beats per min d- Serum potassium of 2.3 mg/dl Rationale: The nurse checks the maternal pulse prior to administering the beta sympathomimetic drug terbutaline and notify the healthcare provider before administration of the drug if the pulse is over 140 beats in within normal limits because peripheral vasodilation accompanies pregnancy and causes the BP decrease. 182- In assessing an older female client with complication associated with chronic obstructive pulmonary disease (COPD), the nurse notices a change in the client’s appearance. Her face appears tense and she begs the nurse not to leave her alone. Her pulse rate is 100, and respirations are 26 per min. What is the primary nursing diagnosis? a- Impaired gas exchange related to narrowing of small airways b- Death anxiety related to concern about prognosis c- Anxiety related to fear of suffocation. d- Ineffective coping related to knowledge deficit about COPD Rationale: A common problem with clients who have COPD is anxiety. These clients cannot aerate their bodies, so they feel a perpetual state of suffocation which is worse during exacerbation of their COPD. A classic descriptor of COPD id impaired gas exchange (A). Because the client has typically adapted to impaired gas exchange over a long period of time, and the nurse has assessed a change in her appearance (A) is not the primary diagnosis at this time. Based on the data presented (B and D) are not the best diagnoses in this situation. 183- A client with a cervical spinal cord injury (SCI) has Crutchfield tongs and skeletal traction applied as a method of closed reduction. Which intervention is most important for the nurse to include in the client’s a plan of care? a- Provide daily care of tong insertion sites using saline and antibiotic ointment b- Modify the client’s diet to prevent constipation c- Encourage active range of motion q2 to 4 hours. d- Instruct the client to report any symptoms of upper extremity paresthesia. Rationale: Crutchfield tongs, a skeletal traction device for cervical immobilization, requires daily care of the surgically inserted tongs to minimize the risk of infection of the insertion site and cranial bone. Daily cleansing with normal saline solution and antibiotic applications minimizes bacterial colonization and helps to prevent infection. 184- A client arrives on the surgical floor after major abdominal surgery. What intervention should the nurse perform first? a- Administer prescribed pain medication b- Assess surgical site c- Determine the client’s vital sign. d- Apply warmed blankets Rationale: The First priority must be to obtain baseline vital signs. A and B should also be accomplished soon, but not until the initial vital signs are determined. C is a nice thing to do. 185- A client is admitted to the emergency department with a respiratory rate of 34 breaths per minute and high pitched wheezing on inspiration and expiration, the medical diagnosis is severe exacerbation of asthma. Which assessment finding, obtained 10 min after the admission assessment, should the nurse report immediately to the emergency department healthcare provider? a- An apical pulse of 120 beats per minute b- Extreme agitation with staff and family c- Client report being anxious d- No wheezing upon auscultation of the chest. Rationale: No wheezing an auscultation indicates that the client is not exchanging air and is highly compromised immediate action is indicated A, B, and C are sign of hypoxia but no as critical as D 186- The nurse is planning a class for a group of clients with diabetes mellitus about blood glucose monitoring. In teaching the class as a whole, the nurse should emphasize the need to check glucose levels in which situation? a- Prior to exercising b- Immediately after meals c- Before going to bed d- During acute illness. Rationale: Client should be instructed to always check their blood glucose whenever they feel sick or nauseated. There is great variability in recommendations for frequency of blood glucose testing. When first diagnosed, clients are often advised to test before and after meals and at bedtime, then after meals and at bedtime for a short period. Once they are stable, clients may be advised to test as often as four times a day or as little as once each week, depending on the consistency of their diet and exercise. 187- A 350-bed acute care hospital declares an internal disaster because the emergency generators malfunctioned during a city-wide power failure. The UAPs working on a general medical unit ask the charge nurse what they should do first. What instruction should the charge nurse provide to these UAPs? a- Go to the emergency department and complete assigned tasks b- Shut all doors to client rooms on the unit in case a fire erupts c- Offer to assist the ICY with ventilator-dependent clients d- Tell all their assigned clients to stay in their rooms. Rationale: A power failure leaves a unit in total darkness except for battery operated lighting. The top priority should be ensuring client safety by having clients stay in their rooms, and UAP can implement this. A is a higher priority in external disaster. B would further compound the lighting problems and is not indicated unless file or smoke is visible. C contraindicated until client safety is ensured on the assigned unit. 188- The nurse is auscultating a client’s heart sounds. Which description should the nurse use to document this sound? (Please listen to the audio file to select the option that applies.) a- Murmur b- s1 s2 c- pericardial friction rub d- s1 s2 s3 189- The healthcare provider changes a client’s medication prescription from IV to PO administration and double the dose. The nurse notes in the drug guide that the prescribed medication, when given orally, has a high first-pass effect and reduce bioavailability. What action should the nurse implement? a- Continue to administer the medication via the IV route b- Give half the prescribed oral dose until the provider is consulted. c- Administer the medication via the oral route as prescribed. d- Consult with the pharmacist regarding the error in prescription. Rationale: Bioavailability refers to the percentage of drug available in the systemic circulation. An increase in dosage is necessary to provide a therapeutic effect for oral medications with significantly reduce bioavailability. 190- A client refuses to ambulate, reporting abdominal discomfort and bloating caused by “too much gas buildup” the client’s abdomen is distended. Which prescribed PRN medication should the nurse administer? a- Hydrocodone/Acetaminophen (Lortab) b- Simethicone (Mylicon) c- Promethazine (Phenergan) d- Nalbupine (Nubain) Rationale: Simethicone is an antiflatulent that is used to increase the client’s ability to expel flatus (B), which relieves the clients discomfort (A and D) are analgesic used to manage pain but do not alleviate the causes of the pain (C) is an antiemetic used to treat nauseas and does not relive excess flatus. 191- The public nurse health received funding to initiate primary prevention program in the community. Which program the best fits the nurse’s proposal? a- Case management and screening for clients with HIV. b- Regional relocation center for earthquake victims c- Vitamin supplements for high-risk pregnant women. d- Lead screening for children in low-income housing. Rational: Primary prevention activities focus on health promotions and disease preventions, so vitamin for high-risk pregnant women provide adequate vitamin and mineral for fetal developmental. 192- When assessing and adult male who presents as the community health clinic with a history of hypertension, the nurse note that he has 2+ pitting edema in both ankles. He also has a history of gastroesophageal reflex disease (GERD) and depression. Which intervention is the most important for the nurse to implement? a- Arrange to transport the client to the hospital b- Instruct the client to keep a food journal, including portions size. c- Review the client’s use of over the counter (OTC) medications. d- Reinforce the importance of keeping the feet elevated. Rationale: Sodium is used in several types of OTC medications. Including antacids, which the client may be using to treat his GERD. Further evaluation is need it to determine the need for hospitalization (A) A food journal (B) may help over, but dietary modifications are needed now since edema is present. (C) May relieve dependent edema, but not treat the underlying etiology. 193- An older client is admitted to the intensive care unit with severe abdominal pain, abdominal distention, and absent bowel sound. The client has a history of smoking 2 packs of cigarettes daily for 50 years and is currently restless and confused. Vital signs are: temperature 96`F, heart rate 122 beats/minute, respiratory rate 36 breaths/minute, mean arterial pressure(MAP) 64 mmHg and central venous pressure (CVP) 7 mmHg. Serum laboratory findings include: hemoglobin 6.5 grams/dl, platelets 6o, 000, and white blood cell count (WBC) 3,000/mm3. Based on these findings this client is at greatest risk for which pathophysiological condition? a- Multiple organ dysfunction syndrome (MODS) b- Disseminated intravascular coagulation (DIC) c- Chronic obstructive disease. d- Acquired immunodeficiency syndrome (AIDS) Rational: MODS are a progressive dysfunction of two or more major organs that requires medical intervention to maintain homeostasis. This client has evidence of several organ systems that require intervention, such as blood pressure, hemoglobin, WBC, and respiratory rate. DIC may develop as a result of MODS. The other options are not correct. 194- A man expresses concern to the nurse about the care his mother is receiving while hospitalized. He believes that her care is not based on any ethical standards and ask what type of care he should expect from a public hospital. What action should the nurse take? a- Provide the man and his mother with a copy of the Patient’s Bill of Rights b- Explain that the hospital adheres to all national accreditation standards c- Advise the man to discuss his concerns with his mother’s healthcare provider d- Determine if he would like to review the hospital’s manual of approved polices. Rationale: The Patient’s Bill of Rights is a universally used tool that describes the rights of clients in all healthcare settings and is essential in ensuring that clients care is provided in an ethical manner. (B) may be perceived as defensive and does not provide the man with specific information about expected standards of care. Concern about the quality of care should be addressed by the hospital staff rather than C. All the healthcare agencies are required to maintain policy and procedure manual for the purpose of standardizing delivery of care within the agencies, but the policy manual is unlikely to provide useful information for clients or family members. 195- A client experiencing withdrawal from the benzodiazepines alprazolam (Xanax) is demonstrating severe agitation and tremors. What is the best initial nursing action? a- Administer naloxone (Narcan) per PNR protocol b- Initiate seizure precautions c- Obtain a serum drug screen d- Instruct the family about withdrawal symptoms. Rationale: Withdrawal of CNS depressants, such as Xanax, results in rebound over-excitation of the CNS. Since the client exhibiting tremors, the nurse should anticipate seizure activity and protect the client. 196- The nurse is caring for a client who is taking a macrolide to treat a bacterial infection. Which finding should the nurse report to the healthcare provider before administering the next dose? a- Jaundice b- Nausea c- Fever d- Fatigue Rationale: Macrolides can cause hepatotoxicity, which is manifested by jaundice and should be reported to the healthcare provider before further doses of the medication are administered, B is a common side effect of macrolides. Fever and Fatigue are expected finding when a client has an infection. 197- A client with Alzheimer’s disease (AD) is receiving trazodone (Desyrel), a recently prescribed atypical antidepressant. The caregiver tells the home health nurse that the client’s mood and sleep patterns are improved, but there is no change in cognitive ability. How should the nurse respond to this information? a- Explain that it may take several weeks for the medication to be effective b- Confirm the desired effect of the medication has been achieved. c- Notify the health care provider than a change may be needed. d- Evaluate when and how the medication is being administered to the client. Rationale: Trazodone o Desyrel, an atypical antidepressant, is prescribed for client with AD to improve mood and sleep. 198- A client with diabetic peripheral neuropathy has been taking pregabalin (Lyrica) for 4 days. Which finding indicates to the nurse that the medication is effective? a- Reduced level of pain b- Full volume of pedal pulses c- Granulating tissue in foot ulcer d- Improved visual acuity. Rationale: Pregabalin is prescribed to decrease the pain associated with diabetic peripheral neuropathy. A, C and D are not expected outcomes of this medication’s effectiveness. 199- A group of nurse-managers is asked to engage in a needs assessment for a piece of equipment that will be expensed to the organization’s budget. Which question is most important to consider when analyzing the cost-benefit for this piece of equipment? a- How many departments can use this equipment? b- Will the equipment require annual repair? c- Is the cost of the equipment reasonable? d- Can the equipment be updated each year? Rationale: If the equipment can be used by more than one department it can be expenses to interdepartmental cost sharing… no only the cost but also the benefit of the equipment. 200- While receiving a male postoperative client’s staples de nurse observe that the client’s eyes are closed and his face and hands are clenched. The client states, “I just hate having staples removed”. After acknowledgement the client’s anxiety, what action should the nurse implement? a- Encourage the client to continue verbalize his anxiety b- Attempt to distract the client with general conversation c- Explain the procedure in detail while removing the staples d- Reassure the client that this is a simple nursing procedure. Rational: Distract is an effective strategy hen a client experience anxiety during an uncomfortable procedure. (A & D) increase the client’s anxiety. 201- A male client is admitted for the removal of an internal fixation that was inserted for the fracture ankle. During the admission history, he tells the nurse he recently received vancomycin (vancomycin) for a methicillin-resistant Staphylococcus aureus (MRSA) wound infection. Which action should the nurse take? (Select all that apply.) a- Collect multiple site screening culture for MRSA b- Call healthcare provider for a prescription for linezolid (Zyrovix) c- Place the client on contact transmission precautions d- Obtain sputum specimen for culture and sensitivity e- Continue to monitor for client sign of infection. Rationale: Until multi-site screening cultures come back negative (A), the client should be maintained on contact isolation(C) to minimize the risk for nosocomial infection. Linezolid (Zyvox), a broad spectrum anti-infecting, is not indicated, unless the client has an active skin structure infection cause by MRSA or multidrug- resistant strains (MDRSP) of Staphylococcus aureus. A sputum culture is not indicated D) based on the client’s history is a wound infection. 202- A vacuum-assistive closure (VAC) device is being used to provide wound care for a client who has stage III pressure ulcer on a below-the- knee (BKA) residual limb. Which intervention should the nurse implement to ensure maximum effectiveness of the device? a- Empty the device every 8 hours and change the dressing daily ensure sterility b- Extended the transparent film dressing only to edge of wound to prevent tension. c- Ensure the transparent dressing has no tears that might create vacuum leaks d- Use an adhesive remover when changing the dressing to promote comfort. Rationale: The nurse should ensure that the VAC transparent film is intact, without tears or loose edges C) because a break in the seal resulting in drying the wound and decreasing the vacuum. The vacuum-assisted closure (VAC) device uses an open sponge in the wound bed, sealed with a transparent film dressing and tube extrudes to a suction device that exert negative pressure to remove excess wound fluid, reduce the bacterial count and stimulate granulation. The VAC is changed every other day or third day, not (A) depending on the stage of wound healing and emptied when full or weekly. The transparent wound dressing should extend 3 to 5 cm beyond the wound edges, not (B) to ensure and airtight seal. Adhesive removers leave a reduce that binder transparent film adherence (D) 203- The nurse is developing the plan of care for a client with pneumonia and includes the nursing diagnosis of “Ineffective airway clearance related to thick pulmonary secretions.” Which intervention is most important for the nurse to include in the client’s plan of care? a- Increase fluid intake to 3,000 ml/daily b- Administer O2 at 5L/mint per nasal cannula c- Maintain the client in a semi Fowler’s position d- Provide frequent rest period. Rationale: The plan of care should include an increase in fluid intake (A) to liquefy and thin secretions for easier removal of thick pulmonary secretion which facilitates airway clearance. (B) should be implemented for signs of hypoxia (C) implemented to facilitate lung expansion, and (D) implemented for activity intolerance, but these interventions do not have the priority of (A) 204- The nurse plans to collect a 24- hour urine specimen for a creatinine clearance test. Which instruction should the nurse provide to the adult male client? a- Clearance around the meatus, discard first portion of voiding, and collect the rest in a sterile bottle b- Urinate at specific time, discard the urine, and collect all subsequent urine during the next 24 hours. c- For the next 24 hours, notify the nurse when the bladder is full, and the nurse will collect catheterized specimens. d- Urinate immediately into a urinal, and the lab will collect specimen every 6 hours, for the next 24 hours. Rationale: Urinate at specific time, discard the urine, and collect all subsequent urine during the next 24 hours is the correct procedure for collecting 24-hour urine specimen. Discarding even one voided specimen invalidate the test. 205- The nurse is preparing to administer a histamine 2-receptor antagonist to a client with peptic ulcer disease. What is the primary purpose of this drug classification? a- Neutralize hydrochloric (HCI) acid in the stomach b- Decreases the amount of HCL secretion by the parietal cells in the stomach c- Inhibit action of acetylcholine by blocking parasympathetic nerve endings. d- Destroys microorganisms causing stomach inflammation. Rationale: B correctly describe the action of histamine 2 receptor antagonist in helping to prevent peptic ulcer disease. 206- The healthcare provider prescribes acarbose (Precose), an alpha-glucosidase inhibitor, for a client with Type 2 diabetes mellitus. Which information provides the best indicator of the drug’s effectiveness? a- Body max index (BMI) between 20 and 24 b- Blood pressure reading less than 120/80 mm Hg c- Hemoglobin A1C (HbA1C) reading less than 7% d- Self-reported glucose levels of 120-150 mg/dl. Rationale: Acarbose (Precose) delays carbohydrate absorption in the GI tract and causes the blood glucose to rise slowly after a meal. The best indicator of acarbose effectiveness is a serum hemoglobin A1 no greater than 7%, an indication of glucose level over time. Acarbose has no effect on pain or blood pressure. Self-reported glucose levels of 120-150 reflect the blood sugar at the time taken and are not the best indicator of drug effectiveness. 207- The nurse assesses a client with new onset diarrhea. It is most important for the nurse to question the client about recent use of which type of medication? a- Antibiotics b- Anticoagulants c- Antihypertensive d- Anticholinergics Rationale: Antibiotic use may be altering the normal flora in the GI tract, resulting in the onset of diarrhea, and several classes of antibiotics result in the overgrowth of Clostridium difficile, resulting in severe diarrhea. 208- A neonate with a congenital heart defect (CHD) is demonstrating symptoms of heart failure (HF). Which interventions should the nurse include in the infant’s plan of care? a- Give O2 at 6 L/nasal canula for 3 repeated oximetry screens below 90% b- Administer diuretics via secondary infusion in the morning only c- Evaluate heart rate for effectiveness of cardio tonic medications d- Use high energy formula 30 calories/ounce at Q3 hours feeding via soft nipples e- Ensure Interrupted and frequent rest periods between procedures. Rationale: Pulse oximetry screening supports prescribed level of O2. HR provides an evaluative criterion for cardiac medications, which reduce heart rate, increase strength contractions (inotropic effects) and consequently affect systemic circulation and tissue oxygenation. Breast milk or basic formula provide 20 calories/ounce, so frequent feedings with high energy formula. D minimize fatigue is necessary. 209- The nurse is caring for a 4-year-old male child who becomes unresponsive as his heart rate decreases to 40 beats/minute. His blood pressure is 88/70 mmHg, and his oxygen saturation is 70% while receiving 100% oxygen by non-rebreather face mask. In what sequence, from first to last, should the nurse implement these actions? (Place the first action on top and last action on the bottom.) 1. Start chest compressions with assisted manual ventilations 2. Administer epinephrine 0.01 mg/kg intraosseous (IO) 3. Apply pads and prepare for transthoracic pacing 4. Review the possible underlying causes for bradycardia. Rationale: The American Heart Association guidelines recommend that the basic life support (BLS) algorithm should be initiated immediately in pediatric clients who are unresponsive or have a heart rate below 60 beats/minutes*** and exhibit signs of poor perfusion. This child is manifesting poor perfusion as evidenced by a low blood pressure and poor oxygenation, so chest compression and assisted manual ventilation should be provided first, followed by administration of drug therapy for persistent bradycardia. Preparation with pad placement for transthoracic pacing should be implemented next, followed by treatment indicated for the underlying cause of the child’s bradycardia. 210- An elderly male client is admitted to the mental health unit with a sudden onset of global disorientation and is continuously conversing with his mother, who died 50 years ago. The nurse reviews the multiple prescriptions he is currently taking and assesses his urine specimen, which is cloudy, dark yellow, and has foul odor. These findings suggest that his client is experiencing which condition? a- Delirium b- Depression c- Dementia d- Psychotic episode Rationale: The client’s clinical findings-polypharmia, urinary tract infection, and possible fluid imbalance are the most common causes of cognition and memory impairment, which is characteristic of delirium. 211- Following an esophagogastroduodenoscopy (EGD) a male client is drowsy and difficult to arouse, and his respiration are slow and shallow. Which action should the nurse implement? Select all that apply. a- Prepare medication reversal agent b- Check oxygen saturation level c- Apply oxygen via nasal cannula d- Initiate bag- valve mask ventilation. e- Begin cardiopulmonary resuscitation Rationale: Sedation, given during the procedure may need to be reverse if the client does not easily wake up. Oxygen saturation level should be asses, and oxygen applied to support respiratory effort and oxygenation. The client is still breathing so the bag- valve mask ventilation and CPR are not necessary. 212- The nurse is planning preoperative teaching plan of a 12-years old child who is scheduled for surgery. To help reduce the child anxiety, which action is the best for the nurse to implement? a- Give the child syringes or hospital mask to play it at home prior to hospitalization. b- Include the child in pay therapy with children who are hospitalized for similar surgery. c- Provide a family tour of the preoperative unit one week before the surgery is scheduled. d- Provide doll an equipment to re-enact feeling associated with painful procedures Rationale: School age children gain satisfaction from exploring and manipulating their environment, thinking about objectives, situations and events, and making judgments based on what they reason. A tour of the unit allows the child to see the hospital environment and reinforce explanation and conceptual thinking. 213- What intervention should the nurse implement during the administration of vesicant chemotherapeutic agent via an IV site in the client’s arm? a- Explain the temporary burning of the IV site may occur. b- Assess IV site frequently for signs of extravasation c- Apply a topical anesthetic of the infusion site for burning d- Monitor capillary refill distal to the infusion site. Rationale: Infiltration of a vesicant can cause severe tissue damage and necrosis, so the IV site should be assessed regularly for extravasation (B) of the chemotherapeutic agent. The client should be instructed to report any discomfort at the site (A). If pain and burning occur, the IV should be stopped and C is not indicated. Peripheral pulses, not D, provide the best assessment of perfusion distal to the infusion should the drug extravasate or infiltrate. 214- When development a teaching plan for a client newly diagnosed type 1 diabetes, the nurse should explain that an increase thirst is an early sing of diabetes ketoacidosis (DKA), which action should the nurse instruct the client to implement if this sign of DKA occur? a- Resume normal physical activity b- Drink electrolyte fluid replacement c- Give a dose of regular insulin per sliding scale d- Measure urinary output over 24 hours. Rationale: As hyperglycemia persist, ketone body become a fuel source, and the client manifest early signs of DKA that include excessive thirst, frequent urination, headache, nausea and vomiting. Which result in dehydration and loss of electrolyte. The client should determine fingerstick glucose level and self-administer a dose of regular insulin per sliding scale. 215- The nurse is teaching a group of clients with rheumatoid arthritis about the need to modify daily activities. Which goal should the nurse emphasize? a- Protect joint function b- Improve circulation c- Control tremors d- Increase weight bearing. Rationale: Primary goal in the management of rheumatoid arthritis is to protect and maintain joint function. 216- An adult client experiences a gasoline tank fire when riding a motorcycle and is admitted to the emergency department (ED) with full thickness burns to all surfaces of both lower extremities. What percentage of body surface area should the nurse document in the electronic medical record (EMR)? a- 9 % b- 18 % c- 36 % d- 45 % Rational: according to the rule of nines, the anterior and posterior surfaces of one lower extremity is designated as 18 %of total body surface area (TBSA), so both extremities equal 36% TBSA, other options are incorrect. 217- A client with hyperthyroidism is receiving propranolol (Inderal). Which finding indicates that the medication is having the desired effect? a- Decrease in serum T4 levels b- Increase in blood pressure c- Decrease in pulse rate d- Goiter no longer palpable. Rationale: Beta blockers such as propranolol help control the symptoms of hyperthyroidism, such as palpitations or tachycardia, but do not alter thyroid hormone levels, B is not a desired effect in hyperthyroidism. Beta blocker do not impact the presence of a goiter. 218- An older male client with type 2 diabetes mellitus reports that has experiences legs pain when walking short distances, and that the pain is relieved by rest. Which client behavior indicates an understanding of healthcare teaching to promote more effective arterial circulation? a- Consistently applies TED hose before getting dressed in the morning. b- Frequently elevated legs thorough the day. c- Inspect the leg frequently for any irritation or skin breakdown d- Completely stop cigarette/ cigar smoking. Rationale: Stopping cigarette smoking helps to decrease vasoconstriction and improve arterial circulation to the extremity. 219- A community health nurse is concerned about the spread of communicable diseases among migrant farm workers in a rural community. What action should the nurse take to promote the success of a healthcare program designed to address this problem? a- Conduct face to face prevention education group session is Spanish b- Offer low literacy material that explain respiratory hygiene and handwashing techniques c- Establish trust with community leaders and respect cultural and family values. d- Provide public services announcements advising those who aril o seek prompt medical attention. Rationale: Nurses must use respectful, targeted approaches and culturally-competent strategies based on a foundation of trust to work effectively with vulnerable populations such as migrant far, worker. These individuals are often transient and fearful of deportation, so building trust is fundamental to other strategies such as client education media. This client population is not likely to be solely comprised of Hispanic individuals, so A does not target all affected individuals Band D are not likely to address the problem until trust and respect are established with this population. 220- The nurse performs a prescribed neurological check at the beginning of the shift on a client who was admitted to the hospital with a subarachnoid brain attack (stroke). The client’s Glasgow Coma Scale (GCS) score is 9. What information is most important for the nurse to determine? a- When the client’s stroke symptoms started b- If the client is oriented to time c- The client’s previous GCS score. d- The client’s blood pressure and respiration rate. Rationale: The normal GCS is 15, and it is most important for the nurse determine if this abnormal score is a sign of improvement or deterioration in the client’s conditions. A is irrelevant. B is part of the GCS. The classic vital signs in late or sudden increasing ICP are Cushing’s triad (widening pulse pressure, bradycardia with full, bounding pulse, and irregular respirations) Additional vital signs and trending of values are needed to evaluate the current finding(D) and C is a more sensitive, consistent evaluation 221- The charge nurse in a critical care unit is reviewing clients’ conditions to determine who is stable enough to be transferred. Which client status report indicates readiness for transfer from the critical care unit to a medical unit? a- Pulmonary embolus with an intravenous heparin infusion and new onset hematuria b- Myocardial infarction with sinus bradycardia and multiple ectopic beats c- Adult respiratory distress syndrome with pulse oximetry of 85% saturation. d- Chronic liver failure with a hemoglobin of 10.1 and slight bilirubin elevation Rationale: A slight bilirubin elevation and anemia are expected finding in a stable client with chronic liver failure who should be transferred to a less-acute medical unit. 222- Based on principles of asepsis, the nurse should consider which circumstance to be sterile? a- One inch- border around the edge of the sterile field set up in the operating room b- A wrapped unopened, sterile 4x4 gauze placed on a damp table top. c- An open sterile Foley catheter kit set up on a table at the nurse waist level d- Sterile syringe is placed on sterile area as the nurse riches over the sterile field. Rationale: A sterile package at or above the waist level is considered sterile. The edge of sterile field is contaminated which include a 1-inch border (A). A sterile object become contaminated by capillary action when sterile objects become in contact with a wet contaminated surface. 223- An unlicensed assistive personnel (UAP) reports that a client’s right hand and fingers spasms when taking the blood pressure using the same arm. After confirming the presence of spams what action should the nurse take? a- Ask the UAP to take the blood pressure in the other arm b- Tell the UAP to use a different sphygmomanometer. c- Review the client’s serum calcium level d- Administer PRN antianxiety medication. Rationale: Trousseau’s sign is indicated by spasms in the distal portion of an extremity that is being used to measure blood pressure and is caused by hypocalcemia (normal level 9.0-10.5 mg/dl, so C should be implemented. 224- A 56-years-old man shares with the nurse that he is having difficulty making decision about terminating life support for his wife. What is the best initial action by the nurse? a- Provide an opportunity for him to clarify his values related to the decision b- Encourage him to share memories about his life with his wife and family c- Advise him to seek several opinions before making decision d- Offer to contact the hospital chaplain or social worker to offer support. Rationale: When a client is faced with a decisional conflict, the nurse should first provide opportunities for the client to clarify values important in the decision. The rest may also be beneficial once the client as clarified the values that are important to him in the decision-making process. 225- A client is being discharged home after being treated for heart failure (HF). What instruction should the nurse include in this client’s discharge teaching plan? a- Weigh every morning b- Eat a high protein diet c- Perform range of motion exercises d- Limit fluid intake to 1,500 ml daily Rationale: It is necessary to monitor weight gain because fluid retention can exacerbate HF. The client should be instructed to weight each morning before breakfast with approximately the same clothing. A is not specifically to HF and fluid retention. 226- A woman just learned that she was infected with Helicobacter pylori. Based on this finding, which health promotion practice should the nurse suggest? a- Schedule a colonoscopy within the next month. b- Encourage screening for a peptic ulcer. c- Screen all family member for hepatitis A d- Eat small, frequent meals thought the day. Rationale: Helicobacter pylori is a gram- negative organism than can colonize in the stomach and is associated with peptic ulcers formation. 227- A client who recently underwear a tracheostomy is being prepared for discharge to home. Which instructions is most important for the nurse to include in the discharge plan? a- Explain how to use communication tools. b- Teach tracheal suctioning techniques c- Encourage self-care and independence. d- Demonstrate how to clean tracheostomy site. Rationale: Suctioning helps to clear secretions and maintain an open airway, which is critical. 228- A child with heart failure is receiving the diuretic furosemide (Lasix) and has serum potassium level 3.0 mEq/L. Which assessment is most important for the nurse to obtain? a- Cardiac rhythm and heart rate. b- Daily intake of foods rich in potassium. c- Hourly urinary output d- Thirst ad skin turgor. Rationale: Hypokalemia is a side effect of potassium-wasting diuretics, such as Lasix, and manifest as muscle weakness, hypotension, tachycardia, and cardiac dysrhythmias, so changes in the child’s heart rate and cardiac rhythm should be reported to the healthcare provider. Although BCD can affect the serum potassium level, the most important finding is the effect of hypokalemia on the child’s cardiac rate and rhythm. 229- The nurse notes a depressed female client has been more withdrawn and non-communicative during the past two weeks. Which intervention is most important to include in the updated plan of care for this client? a- Encourage the client’s family to visit more often b- Schedule a daily conference with the social worker c- Encourage the client to participate in group activities d- Engage the client in a non-threatening conversation. Rationale: Consistent attempts to draw the client into conversations which focus on non-threatening subjects can be an effective means of eliciting a response, thereby decreasing isolation behaviors. There is not sufficient data to support the effectiveness of A as an intervention for this client. Although B may be indicated, nursing interventions can also be used to treat this client. C is too threatening to this client. 230- A client with rheumatoid arthritis (RA) starts a new prescription of etanercept (Enbrel) subcutaneously once weekly. The nurse should emphasize the importance of reporting problem to the healthcare provider? a- Headache b- Joint stiffness c- Persistent fever d- Increase hunger and thirst Rationale: Enbrel decrease immune and inflammatory responses, increasing the client’s risk of serious infection, so the client should be instructed to report a persistent fever, or other signs of infection to the healthcare provider. 231- The nurse is assessing an older adult with type 2 diabetes mellitus. Which assessment finding indicates that the client understands long- term control of diabetes? a- The fating blood sugar was 120 mg/dl this morning. b- Urine ketones have been negative for the past 6 months c- The hemoglobin A1C was 6.5g/100 ml last week d- No diabetic ketoacidosis has occurred in 6 months. Rationale: A hemoglobin A1C level reflects he average blood sugar the client had over the previous 2 to 3 month, and level of 6.5 g/100 ml suggest that the client understand long-term diabetes control. Normal value in a diabetic patient is up to 6.5 g/100 ml. 232- An older male client is admitted with the medical diagnosis of possible cerebral vascular accident (CVA). He has facial paralysis and cannot move his left side. When entering the room, the nurse finds the client’s wife tearful and trying unsuccessfully to give him a drink of water. What action should the nurse take? a- Give the wife a straw to help facilitate the client’s drinking. b- Assist the wife and carefully give the client small sips of water c- Obtain a thickening powder before providing any more fluids. d- Ask the wife to stop and assess the client’s swallowing reflex. Rationale: Until a swallowing reflex has been established, giving oral fluids can be dangerous, even life-threatening. The nurse should immediately stop the dangerous situation and assess the client. It is most important to determine if the client can swallow before giving him anything by mouth. 233- A 13 years-old client with non-union of a comminuted fracture of the tibia is admitted with osteomyelitis. The healthcare provider collects home aspirate specimens for culture and sensitivity and applies a cast to the adolescent’s lower leg. What action should the nurse implement next? a- Administer antiemetic agents b- Bivalve the cast for distal compromise c- Provide high- calorie, high-protein diet d- Begin parenteral antibiotic therapy Rationale: The standard of treatment for osteomyelitis is antibiotic therapy and immobilization. After bond and blood aspirate specimens are obtained for culture and sensitivity, the nurse should initiate parenteral antibiotics as prescribed. 234- The nurse is preparing a community education program on osteoporosis. Which instruction is helpful in preventing bone loss and promoting bone formation? a- Recommend weigh bearing physical activity b- Reduce intake of foods high in vitamin D c- Decrease intake of foods high in fat d- Minimize heavy lifting and bending. Rationale: Active weight-bearing exercise is a primary preventive measure for osteoporosis. C is indicated for client with cardiac and liver diseases. D may decrease injuries but is not directed toward slowing bone loss and promoting bone formation. 235- A client with a history of chronic pain requests a nonopioid analgesic. The client is alert but has difficulty describing the exact nature and location of the pain to the nurse. What action should the nurse implement next? a- Assess the client’s vital signs b- Observe the client’s pupils for dilation c- Document the client’s drug tolerance d- Administer the analgesic as requested Rationale: Chronic pain may be difficult to describe, but should be treated with analgesics as indicated 236- A male client receives a thrombolytic medication following a myocardial infarction. When the client has a bowel movement, what action should the nurse implement? a- Send stool sample to the lab for a guaiac test b- Observe stool for a day-colored appearance. c- Obtain specimen for culture and sensitivity analysis d- Asses for fatty yellow streaks in the client’s stool. Rationale: Thrombolytic drugs increase the tendency for bleeding. So, guaiac (occult blood test) test of the stool should be evaluated to detect bleeding in the intestinal tract. 237- The mother of a child with cerebral palsy (CP) ask the nurse if her child’s impaired movements will worsen as the child grows. Which response provides the best explanation? a- Brain damage with CP is not progressive but does have a variable course b- CP is one of the most common permanent physical disability in children c- Severe motor dysfunction determines the extent of successful habilitation d- Continued development of the brain lesion determines the child’s outcome. Rationale: CP is nonprogressive cerebral insult due to asphyxia, brain malformation, or toxicity, such as kernicterus. It is characterized by impair movement, posturing and may include perceptual, expressive and intellectual deficits, but the motor disabilities can vary as the child grows (A) and as interventions are implemented to prevent disuse complications. 238- During shift report, the central electrocardiogram (EKG) monitoring system alarms. Which client alarm should the nurse investigate first? a- Respiratory apnea of 30 seconds b- Oxygen saturation rate of 88% c- Eight premature ventricular beats every minute d- Disconnected monitor signal for the last 6 minutes. Rationale: The priority is the client whose alarm indicating respiratory apnea that should be assessed first. 239- In early septic shock states, what is the primary cause of hypotension? a- Peripheral vasoconstriction b- Peripheral vasodilation c- Cardiac failure d- A vagal response Rationale: Toxins released by bacteria in septic shock create massive peripheral vasodilation and increase microvascular permeability at the site of the bacterial invasion. 240- A client diagnosed with calcium kidney stones has a history of gout. A new prescription for aluminum hydroxide (Amphogel) is scheduled to begin at 0730. Which client medication should the nurse bring to the healthcare provider’s attention? a- Allopurinol (Zyloprim) b- Aspirin, low dose c- Furosemide (lasix) d- Enalapril (vasote) Rationale: The effectiveness of allopurinol is diminished when aluminum hydroxide is used leading to an increased chance for gout flare ups. The healthcare provider should be alerted about the allopurinol interaction so any changes in medication regimen may be considered. 241- A male client’s laboratory results include a platelet count of 105,000/ mm3 Based on this finding the nurse should include which action in the client’s plan of care? a- Cluster care to conserve energy b- Initiate contact isolation c- Encourage him to use an electric razor d- Asses him for adventitious lung sounds Rationale: This client is at risk for bleeding based on his platelet count (normal 150,000 to 400,000/ mm3). Safe practices, such as using an electric razor for shaving, should be encouraged to reduce the risk of bleeding. 242- A client is admitted to the hospital after experiencing a brain attack, commonly referred to as a stroke or cerebral vascular accident (CVA). The nurse should request a referral for speech therapy if the client exhibits which finding? a- Abnormal responses for cranial nerves I and II b- Persistent coughing while drinking c- Unilateral facial drooping d- Inappropriate or exaggerated mood swings Rationale: After a stroke, clients may experience dysphagia and an impaired gag reflex that is evaluated by a speech pathology team. Coughing while drinking results from impaired swallowing and gag reflex, so a referral to a speech therapist is indicated to evaluate the coordination of oral movements associated with speech and deglutination. Cranial nerves I and II are sensory nerves for taste and sight and do not require a referral to speech pathology. Unilateral facial drooping is associated with stroke but is not a focus of rehabilitation. D sre not addressed by speech therapy. 243- At 1615, prior to ambulating a postoperative client for the first time, the nurse reviews the client’s medical record. Based on date contained in the record, what action should the nurse take before assisting the client with ambulation: a- Remove sequential compression devices. b- Apply PRN oxygen per nasal cannula. c- Administer a PRN dose of an antipyretic. d- Reinforce the surgical wound dressing. Rationale: Sequential compression devices should be removed prior to ambulation and there is no indication that this action is contraindicated. The client’s oxygen saturation levels have been within normal limits for the previous four hours, so supplemental oxygen is not warranted. 244- Which assessment finding for a client who is experiencing pontine myelinolysis should the nurse report to the healthcare provider? a- Sudden dysphagia b- Blurred visual field c- Gradual weakness d- Profuse diarrhea Rationale: Osmotic demyelination, also known as pontine myelinolysis, results in destruction of the myelin sheath that covers nerve cell in the brainstem. This condition can be caused by rapid correction of hyponatremia and is often seen in those with syndrome of inappropriate antidiuretic hormone, Symptoms of pontine myelinolysis are sudden and can include dysphagia, para or quadriparesis and dysarthria. Due to the risk of aspiration the healthcare provider should be notified of the client’s sudden onset of difficulty swallowing dysphagia (A). Diplopia not blurred vision (B) may be experienced. Weakness occurs suddenly, rather than gradually (C). Constipation, not diarrhea (D), is common due to decreased motility. 245- A client is scheduled to receive an IV dose of ondansetron (Zofran) eight hours after receiving chemotherapy. The client has saline lock and is sleeping quietly without any restlessness. The nurse caring for the client is not certified in chemotherapy administration. What action should the nurse take? a- Ask a chemotherapy-certified nurse to administer the Zofran b- Administer the Zofran after flushing the saline lock with saline c- Hold the scheduled dose of Zofran until the client awakens d- Awaken the client to assess the need for administration of the Zofran. Rationale: Zofran is an antiemetic administered before and after chemotherapy to prevent vomiting. The nurse should administer the antiemetic using the accepter technique for IV administration via saline lock. Zofran is not a chemotherapy drug and does not need to be administered by a chemotherapy- certified nurse. 246- When providing diet teaching for a client with cholecystitis, which types of food choices the nurse recommend to the client? a- High protein b- Low fat c- Low sodium d- High carbohydrate. Rationale: A client with cholecystitis is at risk of gall stones that can be move into the biliary tract and cause pain or obstruction. Reducing dietary fat decrease stimulation of the gall bladder, so bile can be expelled, along with possible stones, into the biliary tract and small intestine. 247- A client with a history of cirrhosis and alcoholism is admitted with severe dyspnea and ascites. Which assessment finding warrants immediate intervention by the nurse? a- Jaundice skin tone b- Muffled heart sounds c- Pitting peripheral edema d- Bilateral scleral edema Rationale: Muffled heart sounds may indicative fluid build-up in the pericardium and is life- threatening. The other one is sign of end stage liver disease related to alcoholism but are not immediately life- threatening. 248- When entering a client’s room, the nurse discovers that the client is unresponsive and pulseless. The nurse initiate CPR and Calls for assistance. Which action should the nurse take next? a- Prepare to administer atropine 0.4 mg IVP b- Gather emergency tracheostomy equipment c- Prepare to administer lidocaine at 100 mg IVP d- Place cardiac monitor leads on the client’s chest. Rationale: Before further interventions can be done, the client’s heart rhythm must be determined. This can be done by connecting the client to the monitor. A or C are not a first line drug given for any of the life threatening, pulses dysrhythmias 249- A client with a history of dementia has become increasingly confused at night and is picking at an abdominal surgical dressing and the tape securing the intravenous (IV) line. The abdominal dressing is no longer occlusive, and the IV insertion site is pink. What intervention should the nurse implement? a- Replace the IV site with a smaller gauge. b- Redress the abdominal incision c- Leave the lights on in the room at night. d- Apply soft bilateral wrist restraints. Rationale: The abdominal incision should be redressed using aseptic-techniques. The IV site should be assessed to ensure that it has not been dislodged and a dressing reapplied, if need it. Leaving the light on at night may interfere with the client’s sleep and increase confusion. Restraints are not indicated and should only be used as a last resort to keep client from self-harm. 250- An adult male client is admitted to the emergency room following an automobile collision in which he sustained a head injury. What assessment data would provide the earliest that the client is experiencing increased intracranial pressure (ICP)? a- Lethargy b- Decorticate posturing c- Fixed dilated pupil d- Clear drainage from the ear. Rationale: Lethargy is the earliest sign of ICP along with slowing of speech and response to verbal commands. The most important indicator of increase ICP is the client’s level or responsiveness or consciousness. B and C are very late signs of ICP. 251- In preparing a diabetes education program, which goal should the nurse identify as the primary emphasis for a class on diabetes self-management? a- Prepare the client to independently treat their disease process b- Reduce healthcare costs related to diabetic complications c- Enable clients to become active participating in controlling the disease process d- Increase client’s knowledge of the diabetic disease process and treatment options. Rationale: The primary goal of diabetic self- management education is to enable the client to become an active participant in the care and control of disease process, matching levels of self- management to the abilities of the individual client. The goal is to place the client in a cooperative or collaborative role with healthcare professional rather than (A) 252- To reduce staff nurse role ambiguity, which strategy should the nurse manager implemented? a- Confirm that all the staff nurses are being assigned to equal number of clients. b- Review the staff nurse job description to ensure that it is clear, accurate, and recurrent. c- Assign each staff nurse a turn unit charge nurse on a regular, rotating basis. d- Analyze the amount of overtime needed by the nursing staff to complete assignments. Rationale: Role ambiguity occurs when there is inadequate explanation of job descriptions and assigned tasks, as well as the rapid technological changes that produce uncertainty and frustration. A and D may be implemented if the nurse manager is concerned about role overload, which is the inability to accomplish the tasks related to one’s role. C is not related to ambiguity. 253- The nurse is assisting a new mother with infant feeding. Which information should the nurse provide that is most likely to result in a decrease milk supply for the mother who is breastfeeding? a- Supplemental feedings with formula b- Maternal diet high in protein c- Maternal intake of increased oral fluid d- Breastfeeding every 2 or 3 hours. Rationale: Infant sucking at the breast increases prolactin release and proceeds a feedback mechanism for the production of milk, the nurse should explain that supplemental bottle formula feeding minimizes the infant’s time at the breast and decreases milk supply. B promotes milk production and healing after delivery. C support milk production. C is recommended routine for breast feeding that promote adequate milk supply. 254- Which assessment is more important for the nurse to include in the daily plan of care for a client with a burned extremity? a- Range of Motion b- Distal pulse intensity c- Extremity sensation d- Presence of exudate Rationale: Distal pulse intensity assesses the blood flow through the extremity and is the most important assessment because it provides information about adequate circulation to the extremity. Range of motions evaluates the possibility of long term contractures sensation. C evaluates neurological involvement, and exudate. D provides information about wound infection, but this assessment does not have the priority of determining perfusion to the extremity. 255- An elderly client with degenerative joint disease asks if she should use the rubber jar openers that are available. The nurse’s response should be based on which information about assistive devices? a- They can contribute to increased dependency b- They decrease the risk for joint trauma c- They promote muscle strength d- They diminish range of motion ability. Rationale: Assistive devices of this kind are very beneficial in reducing joint trauma(B) caused by excessive twisting. These devices promote independence, rather that increasing dependency 256- When assessing a 6-month old infant, the nurse determines that the anterior fontanel is bulging. In which situation would this finding be most significant? a- Crying b- Straining on stool c- Vomiting d- Sitting upright. Rationale: The anterior fontanel closes at 9 months of age and may bulge when venous return is reduced from the head, but a bulging anterior fontanel is most significant if the infant is sitting up and may indicated an increase in cerebrospinal fluid. Activities that reduce venous return from the head, such as crying, a Valsalva maneuver, vomiting or a dependent position of the head, cause a normal transient increase in intracranial pressure. 257- A client with angina pectoris is being discharge from the hospital. What instruction should the nurse plan to include in this discharge teaching? a- Engage in physical exercise immediately after eating to help decrease cholesterol levels. b- Walk briskly in cold weather to increase cardiac output c- Keep nitroglycerin in a light-colored plastic bottle and readily available. d- Avoid all isometric exercises but walk regularly. Rationale: Isometric exercise can raise blood pressure for the duration of the exercise, which may be dangerous for a client with cardiovascular disease, while walking provides aerobic conditioning that improves ling, blood vessel, and muscle function. Client with angina should refrain from physical exercise for 2 hours after meals, but exercising does not decrease cholesterol levels. Cold water cause vasoconstriction that may cause chest pain. Nitroglycerin should be readily available and stored in a dark-colored glass bottle not C, to ensure freshness of the medication. 258- What is the priority nursing action when initiating morphine therapy via an intravenous patient-controlled analgesia (PCA) pump? a- Assess the client’s ability to use a numeric pain scale b- Initiate the dosage lockout mechanism on the PCA pump c- Instruct the client to use the medication before the pain become severe d- Assess the abdomen for bowel sounds Rationale: Morphine depress respiration, so ensuring that the client cannot overdose on the medications 259- While undergoing hemodialysis, a male client suddenly complains of dizziness. He is alert and oriented, but his skin is cool and clammy. His vital signs are: heart rate 128 beats/minute, respirations 18 breaths/minute, and blood pressure 90/60. Which intervention should the nurse implement first? a- Raise the client’s legs and feet b- Administer 250 ml saline bolus c- Decrease blood flow from dialyzer d- Stop the hemodialysis procedure. Rationale: To raise the client’s blood pressure is the most immediate and easiest intervention for the nurse to implement. B and C should be done asap to add volume to the vascular space by ceasing to pull fluid from the client. If the blood pressure does not increase, then the procedure may be needing to be stopped. (D) 260- The nurse receives a newborn within the first minutes after a vaginal delivery and intervenes to establish adequate respirations. What priority issue should the nurse address to ensure the newborn’s survival? a- Hypoglycemia b- Fluid balance c- Heat loss d- Bleeding tendencies Rationale: Adequate thermoregulation is the nurse next priority. The newborn is at risk for significant heat loss due to a large surface area exposed to the environment, a thin layer of subcutaneous fat, and distribution of brow fat. Heat loss increases the neonate’s metabolic pathway’s utilization of oxygen and glucose. 261- The fire alarm goes off while the charge nurse is receiving the shift report. What action should the charge nurse implement first? a- Instruct the client’s family member to stay in the visitor waiting area until further notice b- Tell the staff to keep all clients and visitors in the client rooms with the doors closed. c- Direct the nursing staff to evacuate the clients using the stairs in a calm and orderly manner. d- Call the hospital operator to determine if the is indeed a real emergency or a fire drill. Rationale: The charge nurse should treat the alarm as an actual fire emergency and instruct all clients and visitors to stay in the clients’ room with doors closed until otherwise notified. A should be anxiety producing. Visitors should remain in the rooms with the clients. C is only necessary if the location and severity of the fire make the unit unsafe for inhabitants and would only be implemented after other measures to control de fire had failed. D should not be done until after measures are taken to protect clients and visitors. 262- A 60-year-old female client asks the nurse about hormones replacement therapy (HRT) as a means preventing osteoporosis. Which factor in the client’s history is a possible contraindication for the use of HRT? a- Her mother and sister have a history of breast cancer 263- A male client, who is 24 hours postoperative for an exploratory laparotomy, complains that he is “starving” because he has had no “real food” since before the surgery. Prior to advancing his diet, which intervention should the nurse implement? a- Discontinue intravenous therapy b- Obtain a prescription for a diet change c- Assess for abdominal distention and tenderness. d- Auscultate bowel sounds in all four quadrants Rationale: Prior to advancing the client, the nurse should ensure that the client has active bowel sound. 264- The nurse working in the psychiatric clinic has phone messages from several clients. Which call should the nurse return first? a- A young man with schizophrenia who wants to stop taking his medication b- The mother of a child who was involved in a physical fight at school today. c- A client diagnosed with depression who is experiencing sexual dysfunction. d- A family member of a client with dementia who has been missing for five hours Rationale: safety is always the priority concern and the family member of the missing client with dementia needs assistance with contacting authorities as well as psychological support during this time. 265- During change of shift, the nurse reports that a male client who had abdominal surgery yesterday increasingly confused and disoriented during the night. He wandered into other client’s rooms, saying that there are men in his room trying to hurt him. Because of continuing disorientation and the client’s multiple attempts to get of bed, soft restrains were applied at 0400. In what order should the nurse who is receiving report implement these interventions? (Arrange from first action on top to last on the bottom). 1. Assess the client’s skin and circulation for impairment related to the restrains 2. Evaluate the client’s mentation to determine need to continue the restrains 3. Assign unlicensed assistive personnel to remove restrains and remain with client 4. Contact the client’s surgeon and primary healthcare provider Rationale: The nurse who applied the restraints determined that the client was potentially a danger to himself and others, so retrain can be applied without a prescription. 1 the nurse should assess the client for possible complications related to the restraining, evaluate the c… 266- A mother brings her 3-year-old son to the emergency room and tells the nurse the he has had an upper respiratory infection for the past two days. Assessment of the child reveals a rectal temperature of 102 F. He is drooling and becoming increasingly more restless. What action should the nurse take first? a- Put a cold cloth on his head and administer acetaminophen. b- Listen to lung sounds and place him in a mist tent. c- Notify the healthcare provider and obtain a tracheostomy tray d- Assist the child to lie down and examine his throat. Rationale: This child exhibiting signs and symptoms of epiglottitis, a bacterial infection causing acute airway obstruction, so is the immediate action to take. 267- After receiving the first dose of penicillin, the client begins wheezing and has trouble breathing. The nurse notifies the healthcare provider immediately and received several prescriptions. Which medication prescription should the nurse administer first? a- Epinephrine Injection, USP IV b- Diphenhydramine IV c- Albuterol (Ventolin) inhaler d- Methylprednisolone IV Rationale: Epinephrine should be administered immediately to open the airway and raise the blood pressure by vasoconstricting the blood vessels. All other medications should be administered after the epinephrine is given. 268- Two clients ring their call bells simultaneously requesting pain medication. What action should the nurse implement first? a- Prepared both client’s medication and take to them at once b- Determine when each client last received pain medication. c- Evaluate both client’s pain using a standardized pain scale d- Provide non-pharmacologic pain management interventions. Rationale: Before administering pain medication, each client’ s level of pain should be evaluated using a standardizing scale to determine what type and how much pain medication the clients need. 269- A client receives a new prescription for simvastatin (Zocor) 5 mg PO daily at bedtime. What action should the nurse take? a- Provide a bedtime snack to be eaten before taking the medication. b- Administer the medication as prescribed with a glass of water c- Contact the prescriber about changing the time of administration. d- Check the client’s blood pressure prior to administering the med. Rationale: Simvastatin (Zocor), a HMG co-enzyme A reductase inhibitor, interferes with cholesterol synthesis pathway. Zocor can be taken at any time. 270- Which client should the nurse assess frequently because of the risk for overflow incontinence? A client a- Who is bedfast, with increased serum BUN and creatinine levels b- Who is confused and frequently forgets to go to the bathroom c- With hematuria and decreasing hemoglobin and hematocrit levels d- Who has a history of frequent urinary tract infections. Rationale: Overflow incontinence occurs when the bladder becomes overly distended, which is common in the confused client (B) who does not remember to empty his/her bladder. 271- While monitoring a client during a seizure, which interventions should the nurse implement? (Select all that apply) a- Move obstacle away from client b- Monitor physical movements c- Insert an oral padded tongue blade d- Observe for a patent airway e- Record the duration of the seizure f- Restrain extremity to avoid seizures Rationale: Moving this away from the client helps prevent to unnecessary injurie. Observing for the pt airway alert the nurse to provide airway assistance as soon as the seizure stop D and E provide the healthcare provider with an accurate description of the seizure activities. C inserting something on the mouth can obstruct may cause further airway obstruction and is contraindicated even if the client is biting the tongue. F may cause further injury and is contraindicated. 272- A male client with a long history of alcoholism is admitted because of mild confusion and fine motor tremors. He reports that he quit drinking alcohol and stopped smoking cigarettes one month ago after his brother died of lung cancer. Which intervention is most important for the nurses to include in the client’s plan of care? a- Determine client’s level current blood alcohol level. b- Observe for changes in level of consciousness. c- Involve the client’s family in healthcare decisions. d- Provide grief counseling for client and his family. Rationale: Based on the client’s history of drinking, he may be exhibiting sign of hepatic involvement and encephalopathy. Changes in the client’s level of consciousness should be monitored to determine if he able to maintain consciousness, so neurological assessment has the highest priority. 273- An older adult female admitted to the intensive care unit (ICU) with a possible stroke is intubated with ventilator setting of tidal volume 600, PlO2 40%, and respiratory rate of 12 breaths/minute. The arterial blood gas (ABG) results after intubation are PH 7.31. PaCO2 60, PaO2 104, SPO2 98%, HCO3 23. To normalize the client’s ABG finding, which action is required? a- Report the results to the healthcare provider. b- Increase ventilator rate. c- Administer a dose of sodium carbonate. d- Decrease the flow rate of oxygen. Rationale: This client is experience respiratory acidosis. Increasing the ventilator rate depletes CO2 a, which returns the PH toward normal. Report findings is important but only after increasing ventilator rate. 274- The mother of the 12- month-old with cystic fibrosis reports that her child is experiencing increasing congestion despite the use of chest physical therapy (CPT) twice a day and has also experiences a loss of appetite. What instruction should the nurse provide? a- Perform CPT after meals to increase appetite and improve food intake. b- CPT should be performed more frequently, but at least an hour before meals. c- Stop using CPT during the daytime until the child has regained an appetite. d- Perform CPT only in the morning, but increase frequency when appetite improves. Rationale: CPT with inhalation therapy should be performed several times a day to loosen the secretions and move them from the peripheral airway into the central airways where they can be expectorated. CPT should be done at least one hour before meals or two hours after meals. 275- The nurse is evaluating the diet teaching of a client with hypertension. What dinner selection indicates that the client understands the dietary recommendation for hypertension? a- Tomato soup, grilled cheese sandwich, pickles, skim milk, and lemon meringue pie. b- Baked pork chop, applesauce, corn on the cob, 2% milk, and key-lime pie. c- Grilled steak, baked potato with sour cream, green beans, coffee and raisin cream pie. d- Beed stir fry, fried rice, egg drop soup, diet coke and pumpkin pie. Rationale: B is limited in sodium, is high in fiber, and no additional fat is added through cooking, so it is the best choice for an antihypertensive meal. A high in sodium and cholesterol, which should be avoid. C is high in fat and caffeine which can elevate the BP D is high in sodium and cholesterol and includes caffeine. 276- A client with type 2 diabetes mellitus is admitted for frequent hyperglycemic episodes and a glycosylated hemoglobin (HbA1c) of 10%. Insulin glargine 10 units subcutaneously once a day at bedtime and a sliding scale with insulin Aspart q6h are prescribed. What action should the nurse include in this client’s plan of care? a- Fingerstick glucose assessment q6h with meals b- Mix bedtime dose of insulin glargine with insulin aspart sliding scale dose c- Review with the client proper foot care and prevention of injury d- Do not contaminate the insulin aspart so that it is available for iv use e- Coordinate carbohydrate controlled meals at consistent times and intervals f- Teach subcutaneous injection technique, site rotation and insulin management 277- Which problem reported by a client taking lovastatin requires the most immediate follow up by the nurse? a- Diarrhea and flatulence b- Abdominal cramps c- Muscle pain d- Altered taste Rationale: statins can cause rhabdomyolysis, a potentially fatal disease of skeletal muscle characterized by myoglobinuria and manifested with muscle pain, so this symptom should immediately be reported to the HCP. 278- While assessing a client’s chest tube (CT), the nurse discovers bubbling in the water seal chamber of the chest tube collection device. The client’s vital signs are: blood pressure of 80/40 mmHg, heart rate 120 beats/minutes, respiratory rate 32 breaths/minutes, oxygen saturation 88%. Which interventions should the nurse implement? a- Provide supplemental oxygen b- Auscultate bilateral lung fields c- Administer a nebulizer treatment d- Reinforce occlusive CT dressing e- Give PRN dose of pain medication Rationale: The air bubbles indicate an air leak from the lungs, the chest tube site, or the chest tube collection system. Providing oxygen improves the oxygen saturation until the leak has been resolved. Auscultating the lung fields helps to identify absent or decrease lung sound due to collapsing lung. 279- Before leaving the room of a confused client, the nurse notes that a half bow knot was used to attach the client's wrist restraints to the movable portion of the client's bed frame. What action should the nurse take before leaving the room? a- Ensure that the knot can be quickly released. b- Tie the knot with a double turn or square knot. c- Move the ties so the restraints are secured to the side rails. d- Ensure that the restraints are snug against the client's wrist. 280- Oral antibiotics are prescribed for an 18-month-old toddler with severe otitis media. An antipyrine and benzocaine-otic also prescribed for pain and inflammation. What instruction should the nurse emphasize concerning the installation of the antipyrine/benzocaine otic solution? a- Place the dropper on the upper outer ear canal and instill the medication slowly. b- Warm the medication in the microwave for 10 seconds before instilling. c- Keep the medication refrigerated between administrations. d- Have the child lie with the ear up for one to two minutes after installation. 281- An older adult male is admitted with complications related to chronic obstructive pulmonary disease (COPD). He reports progressive dyspnea that worsens on exertion and his weakness has increased over the past month. The nurse notes that he has dependent edema in both lower legs. Based on these assessment findings, which dietary instruction should the nurse provide? a- Limit the intake of high calorie foods. b- Eat meals at the same time daily. c- Maintain a low protein diet. d- Restrict daily fluid intake. Rationale: the client is exhibiting signs of Cor pulmonale, a complication of COPD that causes the right side of the heart to fail. Restricting fluid intake to 1000 to 2000 ml/day, eating a high-calorie diet at small frequent meals with foods that are high in protein and low in sodium can help relieve the edema and decrease workload on the right-side of the heart. 282- The nurse inserts an indwelling urinary catheter as seen in the video what action should the nurse take next? a- Remove the catheter and insert into urethral opening b- Observe for urine flow and then inflate the balloon. c- Insert the catheter further and observe for discomfort. d- Leave the catheter in place and obtain a sterile catheter. Rationale: the catheter is in the vaginal opening. 283- A client with coronary artery disease who is experiencing syncopal episodes is admitted for an electrophysiology study (EPS) and possible cardiac ablation therapy. Which intervention should the nurse delegate to the unlicensed assistive personnel (UAP)? a- Prepare the skin for procedure. b- Identify client's pulse points c- Witness consent for procedure d- Check telemetry monitoring 284- Following an outbreak of measles involving 5 students in an elementary school, which action is most important for the school nurse to take? a- Review the immunization records of all children in the elementary school b- Report the measles outbreak to all community health organizations c- Schedule a mobile public health vehicle to offer measles inoculations to unvaccinated children. d- Restrict unvaccinated children from attending school until measles outbreak is resolved. 285- A preeclamptic client who delivered 24h ago remains in the labor and delivery recovery room. She continues to receive magnesium sulfate at 2 grams per hour. Her total input is limited to 125 ml per hour, and her urinary output for the last hour was 850 ml. What intervention should the nurse implement? a- discontinue the magnesium sulfate immediately b- Decrease the client's iv rate to 50 ml per hour c- Continue with the plan of care for this client d- Change the client's to NPO status Rationale: continue with the plan. Diuresis in 24 to 48h after birth is a sign of improvement in the preeclamptic client. As relaxation of arteriolar spasms occurs, kidney perfusion increases. With improvement perfusion, fluid is drawn into the intravascular bed from the interstitial tissue and then cleared by the kidneys 286- The nurse is planning care for a client who admits having suicidal thoughts. Which client behavior indicates the highest risk for the client acting on these suicidal thoughts? a- Express feelings of sadness and loneliness b- Neglects personal hygiene and has no appetite c- Lacks interest in the activity of the family and friends d- Begin to show signs of improvement in affect Rationale: when a depressed client begins to show signs of improvement, it can be because the client has "figured out" how to be successful in committing suicide. Depressed clients, particularly those who have shown signs of potentially becoming suicidal, should be watched with care for an impending suicide attempt might be greater when the client appear suddenly happy, begin to give away possessions, or becomes more relaxed and talkative. 287- When assessing a multigravida, the first postpartum day, the nurse finds a moderate amount of lochia rubra, with the uterus firm, and three fingerbreadths above the umbilicus. What action should the nurse implement first? a- Massage the uterus to decrease atony b- Check for a distended bladder c- Increase intravenous infusion d- Review the hemoglobin to determined hemorrhage Rationale: a fundus that is dextroverted (up to the right) and elevated above the umbilicus is indicative of bladder distension/urine retention. 288- A 12-year-old client who had an appendectomy two days ago is receiving 0.9% normal saline at 50 ml/hour. The client's urine specific gravity is 1.035. What action should the nurse implement? a- Evaluate postural blood pressure measurements b- Obtain specimen for uranalysis c- Encourage popsicles and fluids of choice d- Assess bowel sounds in all quadrants Rationale: specific gravity of urine is a measurement of hydration status (normal range of 1.010 to 1.025) which is indicative of fluid volume deficit when Sp Gr increases as urine becomes more concentrated. 289- An older male client arrives at the clinic complaining that his bladder always feels full. He complains of weak urine flow, frequent dribbling after voiding, and increasing nocturia with difficulty initiating his urine stream. Which action should the nurse implement? a- Obtain a urine specimen for culture and sensitivity b- Palpate the client's suprapubic area for distention c- Advise the client to maintain a voiding diary for one week d- Instruct in effective technique to cleanse the glans penis Rationale: the client is exhibiting classic signs of an enlarge prostate gland, which restricts urine flow and cause bothersome lower urinary tract symptoms (LUTS) and urinary retention, which is characterized by the client's voiding patterns and perception of incomplete bladder emptying. 290- The nurse is preparing to administer 1.6 ml of medication IM to a 4-month-old infant. Which action should the nurse include? a- Select a 22 gauge 1 ½ inch (3.8 cm) needle for the intramuscular injection b- Administer into the deltoid muscle while the parent holds the infant securely c- Divide the medication into two injections with volumes under 1ml d- Use a quick dart-like motion to inject into the dorsogluteal site. Rationale: IM injection for children under 3 of age should not exceed 1ml. divide the dose into smaller volumes for injection in two different sites. 291- A client who had a below the knee amputation is experiencing severe phantom limb pain (PLP) and ask the nurse if mirror therapy will make the pain stop. Which response by the nurse is likely to be most helpful? a- Research indicates that mirror therapy is effective in reducing phantom limb pain b- You can try mirror therapy, but do not expect to complete elimination of the pain c- Transcutaneous electrical nerve stimulators (TENS) have been found to be more effective d- Where did you learn about the use of mirror therapy in treating in treating phantom limb pain? Rationale: pain relief associated with mirror therapy may be due to the activation of neurons in the hemisphere of the brain that is contralateral to the amputated limb when visual input reduces the activity of systems that perceive protopathic pain. 292- An older adult client with heart failure (HF) develops cardiac tamponade. The client has muffled, distant, heart sounds, and is anxious and restless. After initiating oxygen therapy and IV hydration, which intervention is most important for the nurse to implement? a- Observe neck for jugular vein distention b- Notify healthcare provider to prepare for pericardiocentesis c- Asses for paradoxical blood pressure d- Monitor oxygen saturation (Sp02) via continuous pulse oximetry Rationale: Cardiac tamponade is pressure on the heart that occurs when blood or fluid builds up in the space between the heart muscle (myocardium) and the outer covering sac of the heart (pericardium). In this condition, blood or fluid collects in the pericardium, the sac surrounding the heart. This prevents the heart ventricles from expanding fully. The excess pressure from the fluid prevents the heart from working properly. As a result, the body does not get enough blood. 293- A new member joins the nursing team spreads books on the table, puts items on two chairs, and sits on a third chair. The members of the group are forced to move closer and remove their possessions from the table what action should the nurse leader take? a- Move to welcome and accommodate a new person b- Ask the new person to move belonging to accommodate others c- Tell the new person to move belongings because of limited space d- Bring in additional chairs so that all staff members can be seated 294- The nurse is caring for a one-week old infant who has a ventriculoperitoneal (VP) shunt that was placed 2 days after birth. Which findings are an indication of a postoperative complication? a- Poor feeding and vomiting b- Leakage of CSF from the incisional site c- Hyperactive bowel sound d- Abdominal distention e- WBC count of 10000/mm3 Rationale: A, B and D are sign of postoperative complications. Shunt malfunction is most often caused by mechanical obstruction, which can result from ventricular exudate, distal end thrombosis or displacement, and/or infection. CNS infection is usually manifested by poor feeding, vomiting, elevated temperature, decreased responsiveness and seizure activity. Incisional leakage should be tested for glucose, an indication of CSF, which place the infant at risk for infection. Abdominal distention is a manifestation of peritonitis or a postoperative ileus from distal catheter placement. C is not a result of a shunt obstruction and E is a normal finding for one-week-old neonate. 295- The nurse is preparing a heparin bolus dose of 80 units/kg for a client who weighs 220 pounds. Heparin sodium injection, USP is available in a 3o ml multidose vial with the concentration of 1,000 USP units/ml. how many ml of heparin should the nurse administer? (Enter numeric value only) a- 8 Calculate the client’s weigh in kg: 220 pounds divides by 2.2 pounds/kg ꞊100 kg Calculate the client’s dose, 80 units x 100 kg ꞊ 8,000 units Use the formula, D / H X Q ꞊ 8,000 units / 1,000 units x 1ml ꞊ 8 296- In monitoring tissue perfusion in a client following an above the knee amputation (aka), which action should the nurse include in the plan of care? a- Evaluate closet proximal pulse. b- Asses skin elasticity of the stump. c- Observe for swelling around the stump. d- Note amount color of wound drainage. Rationale: A primary focus of care for a client with an AKA is monitoring for signs of adequate tissue perfusion, which include evaluating skin color and ongoing assessment of pulse strength. 297- The leg of a client who is receiving hospice care have become mottled in appearance. When the nurse observes the unlicensed assistive personal (UAP) place a heating pad on the mottled areas, what action should the nurse take? a- Remove the heating pads and place a soft blanket over the client’s leg and feet. b- Advise the UAP to observe the client’s skin while the heating pads are in place. c- Elevate the client’s feet on a pillow and monitor the client’s pedal pulses frequently. d- Instruct the UAP to reposition the heating pads to the sides of the legs and feet. 298- A client who underwent an uncomplicated gastric bypass surgery is having difficult with diet management. What dietary instruction is most important for the nurse to explain to the client? a- Chew food slowly and thoroughly before attempting to swallow b- Plan volume-controlled evenly-space meal thorough the day c- Sip fluid slowly with each meal and between meals d- Eliminate or reduce intake fatty and gas forming food Rationale: It is most important for the client to learn how to eat without damaging the surgical site and to keep the digestive system from dumping the food instead of digesting it. Eating volume-control and evenly-space meals thorough the day allows the client to fill full, avoid binging, and eliminate the possibility of eating too much one time. Chewing slowly and thoroughly helps prevent over eating by allowing a filling of fullness to occur. Taking sips, rather than large amounts of fluids keeps the stomach from overfilling and allow for adequate calories to be consumed. Gas forming foods and fatty foods should be avoiding decreasing risk of dumping syndrome and flatulence. 299- If the nurse is initiating IV fluid replacement for a child who has dry, sticky mucous membranes, flushed skin, and fever of 103.6 F. Laboratory finding indicate that the child has a sodium concentration of 156 mEq/L. What physiologic mechanism contributes to this finding? a- The intravenous fluid replacement contains a hypertonic solution of sodium chloride b- Urinary and Gastrointestinal fluid loss reduce blood viscosity and stimulate thirst c- Insensible loss of body fluids contributes to the hemoconcentration of serum solutes d- Hypothalamic resetting of core body temperature causes vasodilation to reduce body heat Rationale: Fever causes insensible fluid loss, which contribute to fluid volume and results in hemoconcentration of sodium (serum sodium greater than 150 mEq/L). Dehydration, which is manifested by dry, sticky mucous membranes, and flushed skin, is often managed by replacing lost fluids and electrolytes with IV fluids that contain varying concentration of sodium chloride. Although other options are consistent with fluid volume deficit, the physiologic response of hypernatremia is explained by hem concentration. 300- During a Woman’s Health fair, which assignment is the best for the Practical Nurse (PN) who is working with a register nurse (RN) a- Encourage the woman at risk for cancer to obtain colonoscopy. b- Present a class of breast-self examination c- Prepare a woman for a bone density screening d- Explain the follow-up need it for a client with prehypertension. Rationale: A bone density screening is a fast, noninvasive screening test for osteoporosis that can be explained by the PN. There is no additional preparation needed (A) required a high level of communication skill to provide teaching and address the client’s fear. (B) Requires a higher level of client teaching skill than responding to one client. (D) Requires higher level of knowledge and expertise to provide needed teaching regarding this complex topic. 301- An adult client present to the clinic with large draining ulcers on both lower legs that are characteristics of Kaposi’s sarcoma lesions. The client is accompanied by two family members. Which action should the nurse take? a- Ask family member to wear gloves when touching the patient b- Send family to the waiting area while the client’s history is taking c- Obtain a blood sample to determine is the client is HIV positive d- Complete the head to toes assessment to identify other sign of HIV Rationale: To protect the client privacy, the family member should be asked to wait outside while the client’s history is take. Gloves should be worn when touching the client’s body fluids if the client is HIV positive and these lesion are actually Kaposi sarcoma lesion. HIV testing cannot legally be done without the client explicit permission. A further assessment can be implemented after the family left the room. 302- An adult client is exhibit the maniac stage of bipolar disorder is admitted to the psychiatric unit. The client has lost 10 pounds in the last two weeks and has no bathed in a week “I’m trying to start a new business and “I’m too busy to eat”. The client is oriented to time, place, person but not situation. Which nursing problem has the greatest priority? a- Hygiene-self-care deficit b- Imbalance nutrition c- Disturbed sleep pattern d- Self-neglect Rationale: The client’s nutritional status has the highest priority at this time, and finger foods are often provided, so the client who is on the maniac phase of bipolar disease can receive adequate nutrition. Other options are nursing problems that should also be addresses with the client’s plan of care, but at this stage in the client’s treatment, adequate nutrition is a priority 303- The nurse is preparing a discharge teaching plan for a client who had a liver transplant. Which instruction is most important to include in this plan? a- Limit intake fatty foods for one month after surgery. b- Notify the healthcare provider if edema occurs. c- Increase activity and exercise gradually, as tolerated. d- Avoid crowds for first two months after surgery. Rationale: Cyclosporine immunosuppression therapy is vital in the success of liver transplantation and can increase the risk for infection, which is critical in the first two months after surgery. Fever is often. 304- The nurse is assessing a client’s nailbeds. Witch appearance indicates further follow-up is needed for problems associated with chronic hypoxia? a- 305- A client who had a percutaneous transluminal coronary angioplasty (PTCA) two weeks ago returns to the clinic for a follow up visit. The client has a postoperative ejection fraction of 30%. Today the client has lungs which are clear, +1 pedal edema, and a 5-pound weight gain. Which intervention the nurse implement? a- Arrange transport for admission to the hospital. b- Insert saline lock for IV diuretic therapy. c- Assess compliance with routine prescriptions. d- Instruct the client to monitor daily caloric intake. Rationale: Fluid retention may be a sign that the client is not taking the medication as prescribed or that the prescriptions may need adjustment to manage cardiac function post-PTCA (normal ejection fraction range is 50 to 75%) 306- The RN is assigned to care for four surgical clients. After receiving report, which client should the nurse see first? The client who is a- Two days postoperative bladder surgery with continuous bladder irrigation infusing. b- One day postoperative laparoscopic cholecystectomy requesting pain medication. c- Three days postoperative colon resection receiving transfusion of packed RBCs. d- Preoperative, in buck’s traction, and scheduled for hip arthroplasty within the next 12 hours. 307- The nurse is preparing an older client for discharge following cataract extraction. Which instruction should be including in the discharge teaching? a- Do not read without direct lighting for 6 weeks. b- Avoid straining at stool, bending, or lifting heavy objects. c- Irrigate conjunctiva with ophthalmic saline prior to installing antibiotic ointment. d- Limit exposure to sunlight during the first 2 weeks when the cornea is healing. Rationale: after cataract surgery, the client should avoid activities which increase pressure and place strain on the suture line. 308- The healthcare provider prescribes potassium chloride 25 mEq in 500 ml D_5W to infuse over 6 hours. The available 20 ml vial of potassium chloride is labeled, “10 mEq/5ml.” how many ml of potassium chloride should the nurse add the IV fluid? (Enter numeric value only. If is rounding is required, round to the nearest tenth.) a- 12.5 Rationale: Using the formula D / H X Q: 25 mEq / 10 mEq x 5ml ꞊12.5ml 309- At 40-week gestation, a laboring client who is lying is a supine position tells the nurse that she has finally found a comfortable position. What action should the nurse take? a- Encourage the client to turn on her left side. b- Place a pillow under the client’s head and knees. c- Explain to the client that her position is not safe. d- Place a wedge under the client’s right hip. Rationale: Hypotension from pressure on the vena cava is a risk for the full-term client. Placing a wedge under the right hip will relieve pressure on the vena cava. Other options will either not relieve pressure on the vena cava or would not allow the client the remaining her position of choice. 310- A client with a history of diabetes and coronary artery disease is admitted with shortness of breath, anxiety, and confusion. The client’s blood pressure is 80/60 mmHg, heart rate 120 beats/minute with audible third and fourth heart sounds, and bibasilar crackles. The client’s average urinary output is 5 ml/hour. Normal saline is infusing at 124 ml/hour with a secondary infusion of dopamine at mcg/kg/minute per infusion pump. With intervention should the nurse implement? a- Irrigate the indwelling urinary catheter. b- Prepare the client for external pacing. c- Obtain capillary blood glucose measurement. d- Titrate the dopamine infusion to raise the BP. Rationale: the client is experiencing cardiogenic shock and requires titration per protocol of the vasoactive secondary infusion, dopamine, to increase the blood pressure. Low hourly urine output is due to shock and does not indicate a need for catheter irrigation. Pacing is not indicated based on the client’s capillary blood glucose should be monitored but is not directly indicated at this time. 311- The nurse ends the assessment of a client by performing a mental status exam. Which statement correctly describes the purpose of the mental status exam? a- Determine the client’s level of emotional functioning’ b- Assess functional ability of the primary support system. c- Evaluate the client’s mood, cognition and orientation. d- Review the client’s pattern of adaptive coping skill Rational: the mental status exam assesses the client for abnormalities in cognitive functioning; potential thought processes, mood and reasoning, the other options listed are all components of the client’s psychosocial assessment. 312- An older adult resident of a long-term care facility has a 5-year history of hypertension. The client has a headache and rate the pain 5 on a pain scale 0 to 10. The client’s blood pressure is currently 142/89. Which interventions should the nurse implement? (Select all that apply) a- Administer a daily dose of lisinopril as scheduled. b- Assess the client for postural hypotension. c- Notify the healthcare provider immediately d- Provide a PRN dose of acetaminophen for headache e- Withhold the next scheduled daily dose of warfarin. Rational: the client’ routinely scheduled medication, lisinopril, is an antihypertensive medication and should be administered as scheduled to maintain the client’s blood pressure. A PRN dose of acetaminophen should be given for the client’s headache. The other options are not indicated for this situation. 313- When conducting diet teaching for a client who is on a postoperative soft diet, which foods should eat? (Select all that apply) a- Pasta, noodles, rice. a- Egg, tofu, ground meat. b- Mashed, potatoes, pudding, milk. c- Brussel sprouts, blackberries, seeds. d- Corn bran, whole wheat bread, whole grains. Rational: a client’s postoperative diet is commonly progressed as tolerated. A soft diet includes foods that are mechanically soft in texture (pasta, egg, ground meat, potatoes, and pudding. High fiber foods that require thorough chewing and gas forming foods, such as cruciferous vegetables and fresh fruits with skin, grains and seeds are omitted. 314- The nurse is preparing a 4-day-old I infant with a serum bilirubin level of 19 mg/dl (325 micromol/L) for discharge from the hospital. When teaching the parents about home phototherapy, which instruction should the nurse include in the discharge teaching plan? a- Reposition the infant every 2 hours. b- Perform diaper changes under the light. c- Feed the infant every 4 hours. d- Cover with a receiving blanket. Rational: An infant, who is receiving phototherapy for hyperbilirubinemia, should be repositioned every two hours. The position changes ensure that the phototherapy lights reach all of the body surface areas. Bathing, feedings, and diaper changes are ways for the parents to bond with the infant and can occur away from the treatment. Feedings need to occur more frequently than every 4 hours to prevent dehydration. The infant should wear only a diaper so that the skin is exposed to the phototherapy. 315- When planning care for a client with acute pancreatitis, which nursing intervention has the highest priority? a- Withhold food and fluid intake. b- Initiate IV fluid replacement. c- Administer antiemetic as needed. d- Evaluate intake and output ratio. Rational: The pathophysiologic processes in acute pancreatitis result from oral fluid and ingestion that causes secretion of pancreatic enzymes, which destroy ductal tissue and pancreatic cells, resulting in auto digestion and fibrosis of the pancreas. The main focus of the nursing care is reducing pain caused by pancreatic destruction through interventions that decrease GI activity, such as keeping the client NPO. Other choices are also important intervention but are secondary to pain management. 316- Assessment by the home health nurse of an older client who lives alone indicates that client has chronic constipations. Daily medications include furosemide for hypertension and heart failure and laxatives. To manage the client’s constipation, which suggestions should the nurse provide? (Select all that apply) a- Decrease laxative use to every other day and use oil retention enemas as needed. b- Include oatmeal with stewed pruned for breakfast as often as possible. c- Increase fluid intake by keeping water glass next to recliner. d- Recommend seeking help with regular shopping and meal preparation. e- Report constipation to healthcare provider related to cardiac medication side effects. Rational: older adult are at higher risk for chronic constipation due to decreased gastrointestinal muscle tone leading to reduce motility. Oatmeal with prunes increases dietary fiber and bowel stimulation, thereby decreasing need for laxatives. Increased fluid intake also decreases constipations. Assistance with food preparation might help the client eat more fresh fruits and vegetables and result on less reliance on microwaved and fast foods, which are usually high in sodium and fat with little fiber. Laxatives can be reduced gradually by improving the diet, without resorting to using enemas. 317- A young boy who is in a chronic vegetative state and living at home is readmitted to the hospital with pneumonia and pressure ulcers. The mother insists that she is capable of caring for her son and which action should the nurse implement next? a- Report the incident to the local child protective services. b- Find a home health agency that specializes in brain injuries. c- Determine the mother’s basic skill level in providing care. d- Consult the ethics committee to determine how to proceed. Rational: Although the mother states she is a capable caregiver, the client is manifesting disuse syndrome complications, and the mother’s skill in providing basic care should be determined. Further assessment is needed before implementing other nursing actions. 318- After the risk and benefits of having a cardiac catheterization are reviewed by the healthcare provider, an older adult with unstable angina is scheduled for the procedure. When the nurse presents the consent form for signature, the client asks how the wires will keep a heart heating during the procedure. What action should the nurse take? a- Explain the procedure again in detail and clarify any misconceptions. b- Notify the healthcare provider of the client’s lack of understanding. c- Call the client’s next of kin and have them provide verbal consent. d- Postpone the procedure until the client understands the risk and benefits. Rational: the nurse is only witnessing the signature and is not responsible for the client’s understanding of the procedure. The healthcare provider needs to clarify any questions and misconceptions. Explaining the procedure again is the healthcare provider’s legal responsibility. The other options are not indicated. 319- In assessing a client at 34-weeks’ gestation, the nurse notes that she has a slightly elevated total T4 with a slightly enlarged thyroid, a hematocrit of 28%, a heart rate of 92 beats per minute, and a systolic murmur. Which finding requires follow-up? a- Elevated thyroid hormone level. b- Hematocrit of 28%. c- Heart rate of 92 beats per minute. d- Systolic murmur. Rational: although physiologic anemia is expected in pregnancy, a hematocrit of 28% is below pregnant norms and could signify iron-deficiency anemia. Other options are normal finding pregnancy 320- A client with osteoporosis related to long-term corticosteroid therapy receives a prescription for calcium carbonate. Which client’s serum laboratory values requires intervention by the nurse? a- Total calcium 9 mg/dl (2.25 mmol/L SI) b- Creatinine 4 mg/dl (354 micromol/L SI) c- Phosphate 4 mg/dl (1.293 mmol/L SI) d- Fasting glucose 95 mg/dl (5.3 mmol/L SI) 321- A clinical trial is recommended for a client with metastatic breast cancer, but she refuses to participate and tells her family that she does not wish to have further treatments. The client’s son and daughter ask the nurse to try and convince their mother to reconsider this decision. How should the nurse respond? a- Ask the client with her children present if she fully understands the decision she has made. b- Discuss success of clinical trials and ask the client to consider participating for one month. c- Explain to the family that they must accept their mother’s decision. d- Explore the client’s decision to refuse treatment and offer support Rationale: as long as the client is alert, oriented and aware of the disease prognosis, the healthcare team must abide by her decisions. Exploring the decision with the client and offering support provides a therapeutic interaction and allows the client to express her fears and concerns about her quality of life. Other options are essentially arguing with the client’s decisions regarding her end of life treatment or diminish the opportunity for the client to discuss her feelings 322- An adult client with severe depression was admitted to the psychiatric unit yesterday evening. Although the client ran one year ago, his spouse states that the client no longer runs, bur sits and watches television most of the day. Which is most important for the nurse to include in this client’s plan of care for today? a- Assist client in identifying goals for the day. b- Encourage client to participate for one hour in a team sport. c- Schedule client for a group that focuses on self-esteem. d- Help client to develop a list of daily affirmations. Rationale: clients with severe depression have low energy and benefit from structured activities because concentration is decreased. The client participates in care by identifying goals for the day is the most important intervention for the client’s first day at the unit. Other options can be implemented over time, as the depression decreases. 323- An adult who is 5 feet 5 inches (165.1 cm) tall and weighs 90 lb. (40.8 Kg) is admitted with a diagnosis of chronic anorexia. The client receives a regular diet for 2 days, and the client’s medical records indicates that 100% of the diet provided has been consumed. However, the client’s weight on the third day morning after admission is 89 lb. (40.4 Kg). What action should the nurse implement? a- Examine the client’s room for hidden food. b- Assign staff to monitor what the client eats. c- Ask the client if the food provided is being eaten or discarded. d- Provide the client with a high calorie diet. Rationale: clients with an eating disorder have an unhealthy obsession with food. The client’s continued weight loss, despites indication that the client has consumed 100% of the diet, should raise questions about the client’s intake of the food provided, so the client should be observed during meals to prevent hiding or throwing away food. Other options may be accurate but ineffective and unnecessary. 324- A client exposed to tuberculosis is scheduled to begin prophylactic treatment with isoniazid. Which information is most important for the nurse to note before administering the initial dose? a- Conversion of the client’s PPD test from negative to positive. b- Length of time of the exposure to tuberculosis. c- Current diagnosis of hepatitis B. d- History of intravenous drug abuse. Rationale: prophylactic treatment of tuberculosis with isoniazid is contraindicated for persons with liver disease because it may cause liver damage. The nurse should withhold the prescribed dose and contact the healthcare provider. Other options do not provide data indicating the need to question or withhold the prescribed treatment. 325- The nurse walks into a client’s room and notices bright red blood on the sheets and on the floor by the IV pole. Which action should the nurse take first? a- Clean up the spilled blood to reduce infection transmission. b- Notify the healthcare provider that the client appears to be bleeding. c- Apply direct pressure to the client’s IV site. d- Identify the source and amount of bleeding. Rationale: the nursed should first assess the client to determine the action that should be taken. Patient safety is the priority; other options are not priority. 326- During a routine clinic visit, an older female adult tells the nurse that she is concerned that the flu season is coming soon but is reluctant to obtain the vaccination. What action should the nurse take first? a- Determine when the client last had an influenza vaccination. b- Discuss the concerns expressed by the client about the vaccination. c- Ask about any recent exposure to persons with the flu or other viruses. d- Review the informed consent form for the vaccination with the client. Rationale: the nurse should first address the concerns identified by the client, before taking other actions, such as obtaining information about past vaccinations, exposure to the flu, or reviewing the informed consent form. 327- A client is admitted with acute pancreatitis. The client admits to drinking a pint of bourbon daily. The nurse medicates the client for pain and monitors vital signs q2 hours. Which finding should the nurse report immediately to the healthcare provider? a- Confusion and tremors b- Yellowing and itching of skin. c- Abdominal pain and vomiting d- Anorexia and abdominal distention Rationale: daily alcohol is the likely etiology for the client’s pancreatitis. Abrupt cessation of alcohol can result in delirium tremens (DT) causing confusion and tremors, which can precipitate cardiovascular complications and should be reported immediately to avoid life-threatening complications. The other options are expected findings in those with liver dysfunction or pancreatitis, but do not require immediate action. 328- The nurse is teaching a mother of a newborn with a cleft lip how to bottle feed her baby using medela haberman feeder, which has a valve to control the release of milk and a slit nipple opening. The nurse discusses placing the nipple’s elongated tip in the back of the oral cavity. What instructions should the nurse provide the mother about feedings? a- Squeeze the nipple base to introduce milk into the mouth b- Position the baby in the left lateral position after feeding c- Alternate milk with water during feeding d- Hold the newborn in an upright position Rationale: the mother should be instructed to hold the infant during feedings in a sitting or upright position to prevent aspiration. Impaired sucking is compensated using special feeding appliances and nipples such as the Haberman feeder that prevents aspiration by adjusting the flow of mild according to the effort of the neonate. Squeezing the nipple base may introduce a volume that is greater than the neonate can coordinate swallowing. The preferred position of an infant after feeding is on the right side to facilitate stomach emptying. Sucking difficulty impedes the neonate’s intake of adequate nutrient needed for weight gain and water should be provided after the feeding to cleanse the oral cavity and not fill up the neonate’s stomach. 329- Following a gunshot wound, an adult client a hemoglobin level of 4 grams/dl (40 mmol/L SI). The nurse prepares to administer a unit of blood for an emergency transfusion. The client has AB negative blood type and the blood bank sends a unit of type A Rh negative, reporting that there is not type AB negative blood currently available. Which intervention should the nurse implement? a- Transfuse Type A negative blood until type AB negative is available. b- Recheck the client’s hemoglobin, blood type and Rh factor. c- Administer normal saline solution until type AB negative is available d- Obtain additional consent for administration of type A negative blood Rationale: those who have type AB blood are considered universal recipients using A or B blood types that is the same Rh factor. The client’s hemoglobin is critically low, and the client should receive a unit of blood that is type A, which must be Rh negative blood. Other options are not indicated in this situation. 330- A young adult female college student visits the health clinic in early winter to obtain birth control pills. The clinic nurse asks if the student has received an influenza vaccination. The student stated she did not receive vaccination because she has asthma. How should the nurse respond? a- Offer to provide the influenza vaccination to the student while she is at the clinic b- Encourage the student to obtain a vaccination prior to the next influenza season. c- Confirm that a history of asthma can increase risks associated with the vaccine. d- Advise the student that the nasal spray vaccine reduces side effects for people with asthma. Rationale: person with asthma are at increased risk related to influenza and should receive the influenza vaccination prior to or during influenza season. Waiting until the start of the next season places the student at risk for the current season. The vaccination does not increase risk for persons with asthma, but the nasal spray may result in increased wheezing after receiving that form of the vaccination. 331- A client with eczema is experiencing severe pruritus. Which PRN prescriptions should the nurse administer? (Select all that apply) a- Topical corticosteroid. b- Topical scabicide. c- Topical alcohol rub. d- Transdermal analgesic. e- Oral antihistamine Rationale: anti-inflammatory actions of topical corticosteroids and oral antihistamines provide relief from severe pruritus (itching). Other options are not indicated. 332- The nurse is using a straight urinary catheter kit to collect a sterile urine specimen from a female client. After positioning am prepping this client, rank the actions in the sequence they should be implemented. (Place to first action on the top on the last action on the bottom.) a- 1. Open the sterile catheter kit close to the client’s perineum. b- 2. Don sterile gloves and prepare to sterile field c- 3. Cleanse the urinary meatus using the solution, swabs, and forceps provided d- 4. Place distal end of the catheter in sterile specimen cup and insert catheter into meatus Rationale: First the kit should be open near the clients to minimize the risk of contamination during the collection of the sterile specimen. Once the kit is opened, sterile gloves should be donned to prepare the sterile field. Then the clients’ meatus should be cleansed, and the catheter inserted while to distal end of the catheter drains urine into the sterile specimen cup or receptacle. 333- An adult male was diagnosed with stage IV lung cancer three weeks ago. His wife approaches the nurse and asks how she will know that her husband's death is imminent because their two adult children want to be there when he dies. What is the best response by the nurse? a- Explain that the client will start to lose consciousness and his body system will slow down b- Reassure the spouse that the healthcare provider will let her know when to call the children c- Offer to discuss the client’s health status with each of the adult children d- Gather information regarding how long it will take for the children to arrive Rationale: Expected signs of approaching death include noticeable changes in the client’s level of consciousness and a slowing down of body systems. The nurse should answer the spouse’s questions about the signs of imminent death rather than offering reassurance that may or may not be true. Other options listed may be implemented but the nurse should first answer the spouse’s question directly. 334- When should intimate partner violence (IPV) screening occur? a- As soon as the clinician suspects a problem b- Only when a client presents with an unexplained injury c- As a routine part of each healthcare encounter d- Once the clinician confirms a history of abuse Rationale: Universal screening for IPV is a vital means to identify victims of abuse in relationship. The suspicious of different clinicians vary greatly, so screening would not be implemented consistently. The client should be screened regardless of the presence of injury. Although history of abuse is difficult to confirm, screening should occur regardless, and this incident may know may be initial case of abuse. 335- A child newly diagnosed with sickle cell anemia (SCA) is being discharged from the hospital. Which information is most important for the nurse to provide the parents prior to discharge? a- Instructions about how much fluid the child should drink daily b- information about non-pharmaceutical pain reliever measures c- Referral for social services for the child and family d- Signs of addiction to opioid and medications Rationale: It is essential that the child and family understands the importance of adequate hydration in preventing the stasis-thrombosis-ischemia cycle of a crisis that has a specific plan for hydration is developed so that a crisis can be delayed. Other choices listed are not the most important topics to include in the discharge teaching. 336- What action should the school nurse implement to provide secondary prevention to a school-age child? a- Collaborate with a science teacher to prepare a health lesson b- Prepare a presentation on how to prevent the spread of lice c- Initiate a hearing and vision screening program for first-graders d- Observe a person with type 1 diabetes self-administer a dose of insulin Rationale: Community care occurs at primary, secondary, and tertiary levels of prevention. Primary prevention involves interventions to reduce the incidence of disease. Secondary prevention includes screening programs to detect disease. Tertiary prevention provides treatment directed toward clinically apparent disease. Secondary prevention focuses on screening children for a specific disease processes such as hearing and vision screening. The other options are not examples of secondary prevention. 337- While assisting a client who recently had a hip replacement into a bed pan, the nurse notices that there is a small amount of bloody drainage on the surgical dressing, the client’s skin is warm to the touch, and there is a strong odor from the urine. Which action should the nurse take? a- Obtain a urine sample from the bed pan b- Remove dressing and assess surgical site c- Insert an indwelling urinary catheter d- Measure the client’s oral temperature Rationale: The strong odor from the urine and skin that is warm to the touch may indicate that the client has a urinary tract infection. Assessing the client’s temperature provides objective information regarding infection that can be reported to the healthcare provider. Urine should be obtained via a clean catch, not the bed pan where it has been contaminated. The drainage on the dressing is normal and does not require direct conservation currently. An indwelling catheter should be avoided if possible because it increases the risk of infection. 338- While making rounds, the charge nurse notices that a young adult client with asthma who was admitted yesterday is sitting on the side of the bed and leaning over the bed-side-table. The client is currently receiving at 2 litters/minute via nasal cannula. The client is wheezing and is using pursed-lip breathing. Which intervention should the nurse implement? a- Assist the client to lie back in bed b- Call for an Ambu resuscitating bag c- Increase oxygen to 6 litters/minute d- Administer a nebulizer Treatment Rationale: The client needs an immediate medicated nebulizer treatment. Sitting in an upright position with head and arms resting on the over-bed table is an ideal position to promote breathing because it promotes lung expansion. Other actions me be accurate but not yet indicated. 339- A client with emphysema is being discharged from the hospital. The nurse enters the client’s room to complete discharge teaching. The client reports feeling a little short of breath and is anxious about going home. What is the best course of action? a- Postpone discharge instructions at this time and offer to contact the client by phone in a few days b- Invite the client to return to the unit for discharge teaching in a few days, when there is less anxiety c- Provide only necessary information in short, simple explanations with written instructions to take home d- Give detailed instructions speaking slowly and clearly while looking directly at the client when speaking Rationale: Simple, short explanations should be provided. Information is not retained when the recipient is anxious, and too much information can increase worry. Ethically, discharge instructions may not be postponed. 340- An older adult male who had an abdominal cholecystectomy has become increasingly confused and disoriented over the past 24 hours. He is found wandering into another client’s room and is return to his room by the unlicensed assistive personnel (UAP). What actions should the nurse take? (Select all that apply). a- Apply soft upper limb restrains and raise all four bed rails b- Report mental status change to the healthcare provider c- Assess the client’s breath sounds and oxygen saturation d- Assign the UAP to re-assess the client’s risk for falls e- Review the client’s most recent serum electrolyte values Rationale: The healthcare provider should be informed of changes in the client’s condition (B) because this behavior may indicate a postoperative complication. Diminished oxygenation (C) and electrolyte imbalance (E) may cause increased confusion in the older adult. Raising all four bed rails (A) may lead to further injury if the client climbs over the rails and falls and restrains should not be applied until other measures such as re-orientation are implemented. The nurse should assess the client’s increased risk for falls, rather than assigning this to the UAP (D). 341- A client is admitted to a medical unit with the diagnosis of gastritis and chronic heavy alcohol abuse. What should the nurse administered to prevent the development of Wernicke's syndrome? a- Lorazepam (Ativan) b- Famotidine (Pepcid) c- Thiamine (Vitamin B1) d- Atenolol (Tenormin) Rationale: Thiamine replacement is critical in preventing the onset of Wernickes encephalopathy, an acute triad of confusion, ataxia, and abnormal extraocular movements, such as nystagmus related to excessive alcohol abuse. Other medications are not indicated. 342- When conducting diet teaching for a client who was diagnosed with nutritional anemia in pregnancy, which foods should the nurse encourage the client to eat? (Select all that apply) a- Seeds, spices, lettuce b- Consomme, celery, carrot c- Oranges, orange juice, bananas d- Fortified whole wheat cereals, whole-grain pasta, brown rice e- Spinach, kale, dried raisins and apricots Rationale: Nutritional anemia in pregnancy should be supplemented with additional iron in the diet. Foods that are high in iron content are often protein based, whole grains (D), green leafy vegetables and dried fruits (E). (A, B, and C) are not iron rich sources 343- A client with type 2 diabetes mellitus is admitted for antibiotic treatment for a leg ulcer. To monitor the client for the onset of hyperosmolar hyperglycemic nonketotic syndrome (HHNS), what actions should the nurse take? (Select all that apply) a- Check urine for ketones b- Measure blood glucose c- Monitor vital signs d- Assessed level of consciousness e- Obtain culture of wound Rationale: Blood glucose greater than 600 mg/dl (33.3 mmol/L SI), vital sign changes in mental awareness are indicators of possible HHNS. Urine ketones are monitored in diabetic ketoacidosis. Wound culture is performed prior to treating the wound infection but is not useful in monitoring for HHNS. 344- An infant is receiving penicillin G procaine 220,000 units IM. The drug is supplied as 600,000 units/ml. How many ml should the nurse administer? (Enter numeric value only. If rounding is required, round to the nearest tenth) 0.4 Rationale: Calculate using the formula, desired dose (220,000 units) over dose on hand (600,000 units) x the volume of the available dose (1 ml). 220,000 / 600,000 x 1 ml = 0.36 = 0.4 ml 345- After receiving report, the nurse can most safely plan to assess which client last? The client with… a- A rectal tube draining clear, pale red liquid drainage b- A distended abdomen and no drainage from the nasogastric tube c- No postoperative drainage in the Jackson-Pratt drain with the bulb compressed d- Dark red drainage on a postoperative dressing, but no drainage in the Hemovac®. Rationale: The most stable client is the one with a functioning drainage device and no drainage. This client can most safely be assessing last. Other clients are either actively bleeding, have an obstruction in the nasogastric tube which may result in vomiting, or may be bleeding and / or may have a malfunction in the Hemovac® drain. 346- The nurse instructs an unlicensed assistive personnel (UAP) to turn an immobilized elderly client with an indwelling urinary catheter every two hour. What additional action should the nurse instruct the UAP to take each time the client is turned? a- Empty the urinary drainage bag b- Feed the client a snack c- Offer the client oral fluids d- Assess the breath sounds Rationale: Increasing oral fluid intake reduces the risk of problems associated with immobility, so the UAP should be instructed to offer the client oral fluids every two hours, or whenever turning he client. It is not necessary to empty the urinary bag or feed the client every two hours. Assessment is a nursing function, and UAPs do not have the expertise to perform assessment of breath sounds. 347- The nurse is preparing a client who had a below-the-knee (BKA) amputation for discharge to home. Which recommendations should the nurse provide this client? (Select all that apply) a- Inspect skin for redness b- Use a residual limb shrinker c- Apply alcohol to the stump after bathing d- Wash the stump with soap and water e- Avoid range of motion exercises Rationale: Several actions are recommended for home care following an amputation. The skin should be inspected regularly for abnormalities such as redness, blistering, or abrasions. A residual limb shrinker should be applied over the stump to protect it and reduce edema. The stump should be washed daily with a mild soap and carefully rinse and dried. The client should avoid cleansing with alcohol because it can dry and crack the skin. Range of motion should be done daily. 348- When assessing the surgical dressing of a client who had abdominal surgery the previous day, the nurse observes that a small amount of drainage is present on the dressing and the wound’s Hemovac suction device is empty with the plug open. How should the nurse respond? a- Replace the dressing and remove the drainage device b- Reposition the drainage device and keep the plug open c- Notify the healthcare provider that the drain is not working d- Recompress the wound suction device and secure to plug Rationale: The plug of a wound suction device, such as a Hemovac, should be closed after compressing the device to apply gentle suction in a closed surgical wound to facilitate the evacuation of subcutaneous fluids into the device. Compressing the device and securing the plug should restore function of the closed wound device. A small amount of drainage should be marked on the dressing, but replacing the dressing is not necessary and the nurse should not remove the device. Other options are not indicated. 349- A mother brings her 4-month-old son to the clinic with a quarter taped over his umbilicus and tells the nurse the quarter is supposed to fix her child’s hernia. Which explanations should the nurse provide? a- This hernia is a normal variation that resolves without treatment. b- Restrictive clothing will be adequate to help the hernia go away. c- An abdominal binder can be worn daily to reduce the protrusion. d- The quarter should be secured with an elastic bandage wrap. Rational: an umbilical hernia is a normal variation in infants that occurs due to an incomplete fusion of the abdominal musculature through the umbilical ring that usually resolves spontaneously as the child learns to walk. Other choices are ineffective and unnecessary. 350- A client who is admitted to the intensive care unit with syndrome of inappropriate antidiuretic hormone (SIADH) has developed osmotic demyelination. Which intervention should the nurse implement first? a- Patch one eye. b- Reorient often. c- Range of motion. d- Evaluate swallow Rational: Osmotic demyelination, also known as central pontine myelinolysis, is nerve damage caused by the destruction of the myelin sheath covering nerve cells in the brainstem. The most common cause is a rapid, drastic change in sodium levels when a client is being treated for hyponatremia, a common occurrence in SIADH. Difficulty swallowing due to brainstem nerve damage should be care but determining the client’s risk for aspiration is most important. 351- A client with possible acute kidney injury (AKI) is admitted to the hospital and mannitol is prescribed as a fluid challenge. Prior to carrying out this prescription, what intervention should the nurse implement? a- Collect a clean catch urine specimen. b- Instruct the client to empty the bladder. c- Obtain vital signs and breath sounds. d- No specific nursing action is required Rational: the client’s baseline cardiovascular status should be determined before conducting the fluid challenge. If the client manifests changes in the vital signs and breath sounds associated with pulmonary edema, the administration of the fluid challenge should be terminating. Other options would not assure a safe administration of the medication. 352- A male client with COPD smokes two packs of cigarettes per day and is admitted to the hospital for a respiratory infection. He complains that he has trouble controlling respiratory distress at home when using his rescue inhaler. Which comment from the client indicates to the nurse that he is not using his inhaler properly? a- “I have a hard time inhaling and holding my breath after I squeeze the inhaler, but I do my best” b- “I never use the inhaler unless I am feeling really short of breath” c- I always shake the inhaler several times before I start” d- “After I squeeze the inhaler and swallow, I always feel a slight wave of nausea, bit it goes away” Rationale: It is vital for the nurse to emphasize to the client that the mist should be inhaled, not swallowed. This assessment should be done for all COPD clients, not just those who complain about their inhalers. 353- A nurse is planning to teach infant care and preventive measures for sudden infant death syndrome (SIDS) to a group of new parents. What information is most important for the nurse to include? a- Swaddle the infant in a blanket for sleeping b- Place the infant in a prone position whenever possible c- Prop that the infant’s crib matter is firm d- Ensure that the infant’s crib mattress is firm. Rationale: Sudden infant death syndrome is the unexplained death of infants under the age of one year. Parents should be educated about the methods to reduce the risk of SIDS, which include use of a firm crib mattress, maternal smoking cessation before and after pregnancy, avoidance of pillows in the crib, and placing the infant in the supine position. (Back to Sleep Campaign) 354- A 6 -year-old who has asthma is demonstrating a prolonged expiratory phase and wheezing and has 35% personal best peak expiratory flow rate (PEFR). Based on these finding, which action should the nurse implement first? a- Administer a prescribed bronchodilator. b- Report finding to the healthcare provider. c- Encourage the child to cough and deep breath d- Determine what trigger precipitated this attack. Rationale: If the PEFR is below 50% in as asthmatic child, there is severe narrowing of the airway, and a bronchodilator should be administered immediately. B should be implemented after A. C will not alleviate the symptoms and D is not a priority. 355- A client is receiving lactulose (Portalac) for signs of hepatic encephalopathy. To evaluate the client’s therapeutic response to this medication, which assessment should the nurse obtain? a- Level of consciousness b- Percussion of abdomen c- Serum electrolytes d- Blood glucose. Rationale: Colonic bacteria digest lactulose to create a drug-induces acidic and hyperosmotic environment that draws water and blood ammonia into the colon and coverts ammonia to ammonium, which is trapped in the intestines and cannot be reabsorbed into the systemic circulation. This therapeutic action of lactulose is to reduce serum ammonia levels, which improves the client’s level of consciousness and mental status. 356- When administering an immunization in an adult client, the nurse palpates and administer the injection one inch below the acromion process into the center of the muscle mass. The nurse should document that the vaccine was administered at what site? a- Rectus femenis b- Ventrogluteous c- Vastus lateralis d- Deltoid Rationale: The acromion process is a parameter identified for the deltoid site. 357- A primigravida a 40-weeks’ gestation with preeclampsia is admitted after having a seizure in the hot tub at a midwife’s birthing center. Based on documentation in the medical record, which action should the nurse implement? (Click on each chart tab for additional information. Please be sure to scroll to the bottom right corner of each tab to view all information contained in the client’s medical record.) a- Continue to monitor the client’s blood pressure hourly. b- Inform the healthcare provider of CBC results c- Update the nursery staff on the client’s status d- Give a dose of calcium gluconate per preeclampsia protocol. Rationale: The laboratory results, urinary output, FHR, and vital signs are within expected ranges for a client who is receiving magnesium sulfate for preeclampsia. The client remains hypertensive, son continued hourly monitoring A is indicated Client magnesium center therapeutic range (5 to 7 mEq/L) 358- A female nurse who took drugs from the unit for personal use was temporarily released from duty. After completion of mandatory counseling, the nurse has asked administration to allow her to return to work. When the nurse administrator approaches the charge nurse with the impaired nurse request, which action is best for the charge nurse to take? a- Since treatment is completed, assign the nurse to the route RN responsibilities b- Ask to meet with impaired nurse’s therapist before allowing her back on the unit. c- Allow the impaired nurse to return to work and monitor medication administration d- Meet with staff to assess their feelings about the impaired nurse’s return to the unit. Rationale: provides essential monitoring and helps ensure nurse compliance and promote client safety. 359- In making client care assignment, which client is best to assign to the practical nurse (PN) working on the unit with the nurse? a- An immobile client receiving low molecular weight heparin q12 h. b- A client who is receiving a continuous infusion of heparin and gets out of bed BID c- A client who is being titrated off heparin infusion and started on PO warfarin (Coumadin) d- An ambulatory client receiving warfarin (Coumadin) with INR of 5 second. Rationale: A describe the most stable client. The other ones are at high risk for bleeding problems and require the assessment skills. 360- A client who is admitted to the intensive care unit with a right chest tube attached to a THORA-SEAL chest drainage unit becomes increasingly anxious and complain of difficulty breathing. The nurse determines the client is tachypneic with absent breath sounds in the client’s right lungs fields. Which additional finding indicates that the client has developed a tension pneumothorax? a- Continuous bubbling in the water seal chamber b- Decrease bright red blood drainage c- Tachypnea and difficulty breathing d- Tracheal deviation toward the left lung. Rationale: Tracheal deviation toward the unaffected left lung with absent breath sounds over the affected right lung are classic late signs of a tension pneumothorax. 361- A low-risk primigravida at 28-weeks’ gestation arrives for her regular antepartum clinic visit. Which assessment finding should the nurse consider within normal limits for this client? a- Pulse increase of 10 beats/minute b- Proteinuria c- Glucosuria d- Fundal height 0f 22 centimeters Rationale: Blood volume increases 25 to 40 % in pregnancy which increases cardiac output and increases heart rate by approx. 10 to 20 beats/ mints. Proteinuria is for preeclampsia, Glucosuria is for gestational diabetes. A fundal height for 28 weeks should be at 28 cm not 22 362- The nurse discovers that an elderly client with no history of cardiac or renal disease has an elevated serum magnesium level. To further investigate the cause of this electrolyte imbalance, what information is most important for the nurse to obtain from the client’s medical history? a- Genetically inherited disorders of family members b- Length and frequency of the client’s tobacco use. c- Ingestion of selfish or fish oil capsules daily. d- Frequency of laxative use for chronic constipation Rationale: Elderly clients are at risk of developing hypermagnesemia as a result of chronic laxative abuse. 363- Which action should the nurse implement with auscultating anterior breath sounds? (Place the first action on top and last action on the bottom) a- Correct order: (PADD) 1. Place stethoscope in suprasternal area to auscultate for bronchial sounds 2. Auscultate bronchovesicular sounds from side to side the first and second intercostal spaces 3. Displace female breast tissue and apply stethoscope directly on chest wall to hear vesicular sounds 4. Document normal breath sounds and location of adventitious breath sounds 364- A client with chronic alcoholism is admitted with a decreased serum magnesium level. Which snack option should the nurse recommend to this client? a- Cheddar cheese and crackers. b- Carrot and celery sticks. c- Beef bologna sausage slices. d- Dry roasted almonds. Rational: alcoholism promotes inadequate food intake and gastrointestinal loss of magnesium include green leafy vegetables and nuts and seeds. Other snacks listed provide much lower amounts of magnesium per serving. 365- The nurse is preparing a teaching plan for an older female client diagnosed with osteoporosis. What expected outcome has the highest priority for this client? a- Identifies 2 treatments for constipation due to immobility. b- Names 3 home safety hazards to be resolve immediately. c- State 4 risk factors for the development of osteoporosis. d- Lists 5 calcium-rich foods to be added to her daily diet. Rational: a major teaching goal for an elderly client with osteoporosis is maintenance of safety to prevent falls. Injury due to a fall, usually resulting in a hip fracture, can result in reduced mobility and associated complications. Other goals are also important when teaching clients who have osteoporosis, but they do not have the priority of preventing falls, which relates to safety. 366- The nurse is teaching a male adolescent recently diagnosed with type 1diabetes mellitus (DM) about self-injecting insulin. Which approach is best for the nurse to use to evaluate your effectiveness of the teaching? a- Ask the adolescent to describe his level of comfort with injecting himself with insulin. b- Observe him as he demonstrates self-injection technique in another diabetic adolescent c- Have the adolescent list the procedural steps for safe insulin administration. d- Review his glycosylated hemoglobin level 3 months after the teaching session. Rational: watching the adolescent perform the procedure with another adolescent provides peer support the most information regarding his skill with self-injection. Other options do not provide information about the effectiveness of nurse’s teaching. 367- A young adult woman visits the clinic and learns that she is positive for BRCA1 gene mutation and asks the nurse what to expect next. How should the nurse respond? a- Explain that counseling will be provided to give her information about her cancer risk. b- Gather additional information about the client’s family history for all types of cancer. c- Offer assurance that there are a variety of effective treatments for breast cancer. d- Provide information about survival rates for women who have this genetic mutation. Rational: BRACA1or BRACA2 genetic mutation indicates an increased risk for developing breast or ovarian cancer and genetic counseling should be provided to explain the increased risk (A)to the client along with options for increased screening or preventative measures. (B) Is completed by the genetic counselor before the client undergoes genetic testing. a positive BRACA1test is not an indicator of the presence of cancer and (C and D) are not appropriate responses prior to genetic counseling. 368- A mother runs into the emergency department with s toddler in her arms and tells the nurse that her child got into some cleaning products. The child smells of chemicals on hands, face, and on the front of the child's clothes. After ensuring the airway is patent, what action should the nurse implement first? a- Call poison control emergency number. b- Determine type of chemical exposure. c- Obtain equipment for gastric lavage. d- Assess child for altered sensorium. Rational: once the type of chemical is determined, poison control should be called even if the chemical is unknown. If lavage is recommended by poison control, intubation and nasogastric tube may be needed as directed by poison control. Altered sensorium, such as lethargy, may occur if hydrocarbons are ingested 369- The nurse assigned unlicensed assistive personnel (UAP) to apply antiembolism stockings to a client. The nurse and UAP enters the room, the nurse observes the stockings that were applying by the UAP. The UAP states that the client requested application of the stockings as seen on the picture, for increased comfort. What action should the nurse take? a- Ask the client if the stocking feel comfortable. b- Supervise the UAP in the removal of the stockings. c- Place a cover over the client’s toes to keep them warm. d- Discussed effective use of the stockings with the client and UAP Rational: antiembolism stockings are designed to fit securely and should be applied so that there are no bands of the fabric constricting venous return. The nurse should discuss the need for correct and effective use of the stockings with both the client and UAP to improve compliance. Other options do not correct the incorrect application of the stockings. 370- Nurses working on a surgical unit are concerned about the physician’s treatment of clients during invasive procedures, such as dressing changes and insertion of IV lines. Clients are often crying during the procedures, and the physician is usually unconcerned or annoyed by the client’s response. To resolve this problem, what actions should the nurses take? (Arrange from the first action on the top of the list on the bottom) 1. Talk to the physician as a group in a non-confrontational manner. 2. Document concerns and report them to the charge nurse. 3. Submit a written report to the director of nursing. 4. Contact the hospital’s chief of medical services. 5. File a formal complaint with the state medical board. Rational: nurses have both an ethical and legal responsibility to advocate for clients’ physical and emotional safety. Talking with the physician in a non-confrontational manner is the first step in conflict resolution. If this is not effective, the organizational chain of ineffective, a formal complaint with the state medical board should be implemented. 371- While changing a client’s chest tube dressing, the nurse notes a crackling sensation when gentle pressure is applied to the skin at the insertion site. What is the best action for the nurse to take? a- Apply a pressure dressing around the chest tube insertion site. b- Assess the client for allergies to topical cleaning agents. c- Measure the area of swelling and crackling. d- Administer an oral antihistamine per PRN protocol. Rational: a crackling sensation, or crepitus, indicates subcutaneous emphysema, or air leaking into the skin. This area should be measured, and the finding documented. Other options are not indicated for crepitus. 372- To prevent infection by auto contamination during the acute phase of recovery from multiple burns, which intervention is most important for the nurse to implement? a- Dress each wound separately. b- Avoid sharing equipment between multiple clients. c- Use gown, mask and gloves with dressing change. d- Implement protective isolation. Rational: each wound should be dressed separately using a new pair of sterile glove to avoid auto contamination (the transfer of microorganisms’ form one infected wound to a non-infected wound). The other choices do not prevent auto contamination. 373- The nurse is preparing an intravenous (IV) fluid infusion using an IV pump. Within 30 seconds of turning on the machine, the pump’s alarm beeps “occlusion”. What action should the nurse implement first? a- Flush the vein with 3 ml of sterile normal saline. b- Assess the IV catheter insertion site for infiltration. c- Verify the threading of the tubing through the IV pump. d- Determine if the clamp on the IV tubing is released Rational: When the pump immediately beeps, it is often because the IV tubing clamp is occluding the flow, so the clamp should be checked first to ensure that it is open. If the alarm is not eliminated after the tubing clamp is released, flushing the IV site with saline is a common practice to clean the needle or to identify resistance due to another source. Local signs of infiltration may indicate the need to select another vein, but the pump’s beeping-this early in the procedure is likely due to a mechanical problem. If beeping continues after verifying that the clamp is released the placement or threading of the tubing through the pump should be verified. 374- A client with arthritis has been receiving treatment with naproxen and now reports ongoing stomach pain, increasing weakness, and fatigue. Which laboratory test should the nurse monitor? a- Sed rate (ESR) b- Hemoglobin c- Calcium d- Osmolality. Rational: naproxen can cause gastric bleeding, so the nurse should monitor the client’s hemoglobin to assess for possible bleeding. Other options are not likely to be affected by the used of naproxen and are not related to the client’s current symptoms. 375- The nurse assesses a child in 90-90 traction. Where should did nurse assess for signs of compartment syndrome? Rationale: compartment syndrome is the result of swelling and subsequent reduction in circulation to the area distal to the compartment. This can be a complication of traumatic injury and cast administration, so it is important to assess circulation distal to the casted prolonged capillary refill. 376- After receiving the Braden scale findings of residents at a long-term facility, the charge nurse should to tell the unlicensed assistive personnel (UAP) to prioritize the skin care for which client? a- An older adult who is unable to communicate elimination needs. b- An older man whose sheets are damped each time he is turned. c- A woman with osteoporosis who is unable to bear weight. d- A poorly nourished client who requires liquid supplement. Rational: a Braden score of less than 18 indicates a risk for skin breakdown, and clients with such score require intensive nursing care. Constant moisture places the client at a high risk for skin breakdown, and interventions should be implemented to pull moisture away from the client’s skin. Other options may be risk factors but do not have as high a risk as constant exposure to moisture. 377- A client with acute renal failure (ARF) is admitted for uncontrolled type 1 diabetes Mellitus and hyperkalemia. The nurse administers an IV dose of regular insulin per sliding scale. Which intervention is the most important for the nurse to include in this client’s plan of care? a- Monitor the client’s cardiac activity via telemetry. b- Maintain venous access with an infusion of normal saline. c- Assess glucose via fingerstick q4 to 6 hours. d- Evaluate hourly urine output for return of normal renal function. Rational: as insulin lowers the blood glucose of a client with diabetic ketoacidosis (DKA), potassium returns to the cell but may not impact hyperkalemia related to acute renal failure. The priority is to monitor the client for cardiac dysrhythmias related to abnormal serum potassium levels. IV access, assessment of glucose level, and monitoring urine output are important interventions, but do not have the priority of monitoring cardiac function. 378- A client with C-6 spinal cord injury rehabilitation. In the middle of the night the client reports a severe, pounding headache, and has observable piloerection or “goosebumps”. The nurse should asses for which trigger? a- Loud hallway noise. b- Fever c- Full bladder d- Frequent cough. Rational: a pounding headache is a sign of autonomic hyperreflexia, an acute emergency that occurs because of an exaggerated sympathetic response in a client with a high level spinal cord injury. Any stimulus below the level of injury can trigger autonomic hyperreflexia, but the most common cause is an overly distended bladder. The other options are unlikely to produce the manifestation of autonomic hyperreflexia. 379- A nurse working on an endocrine unit should see which client first? a- An adolescent male with diabetes who is arguing about his insulin dose. b- An older client with Addison’s disease whose current blood sugar level is 62mg/dl (3.44 mmol/l). c- An adult with a blood sugar of 384mg/dl (21.31mmol/l) and urine output of 350 ml in the last hour. d- A client taking corticosteroids who has become disoriented in the last two hours. Rational: meeting the client’s need for safety is a priority intervention. Mania and psychosis can occur during corticosteroids therapy, places the client at risk for injury, so the patient taking corticosteroids should be seen first. 380- A client is receiving oral antibiotic suspension labeled 250 mg/2ml. The healthcare provider prescribes 200mg every 6 hours. How many ml should the nurse administer at each dose? (Enter numerical value only. If rounding is required, round to the nearest tenth) a- Answer: 1.6 Rational: using the formula D/H x Q 200mg/250 mg x 2ml = 200/250 = 1.6 ml 381- Four hours after surgery, a client reports nausea and begins to vomit. The nurse notes that the client has a scopolamine transdermal patch applied behind the ear. What action should the nurse take? a- Reposition the transdermal patch to the client’s trunk. b- Remove the transdermal patch until the vomiting subsides. c- Notify the healthcare provider of the vomiting. d- Explain that this is a side effect of the medication in the patch. Rational: transdermal scopolamine is used to prevent nausea and vomiting from anesthesia and surgery. The nurse should notify the healthcare provider if the medication is ineffective. The patch should be applied behind the ear and should remain in place to reduce the nausea and vomiting. Nausea and vomiting are no side effects of the medication. 382- The nurse identifies an electrolyte imbalance, an elevated pulse rate, and elevated BP for a client with chronic kidney disease. Which is the most important action for the nurse to take? a- Monitor daily sodium intake. b- Record usual eating patterns. c- Measure ankle circumference. d- Auscultate for irregular heart rate. Rational: Chronic kidney failure (CKF) is a progressive, irreversible loss of kidney functions, decreasing glomerular filtration rate (GFR), and the kidney’s inability to excrete metabolic waste products and water, resulting in fluid overload, elevated pulse, elevated BP and electrolytes imbalances. The most important action for the nurse to implement is to auscultate for irregular heart rate (D) due to the decreased excretion of potassium by the kidneys. (A, B, and C) are not as important as monitoring for fatal cardiac dysrhythmias related to hyperkalemia. 383- A client with persistent low back pain has received a prescription for electronic stimulator (TENS) unit. After the nurse applies the electrodes and turns on the power, the client reports feeling a tingling sensation. How should the nurse respond? a- Determine if the sensation feels uncomfortable. b- Decrease the strength of the electrical signals. c- Remove electrodes and observe for skin redness. d- Check the amount of gel coating on the electrodes. Rational: electronic stimulators, such as a transelectrical nerve stimulator (TENS) unit, have been found to be effective in reducing low back pain by “closing the gate” to pain stimuli. A tingling sensation should be felt when the power is turned on, and the nurse should assess whether the sensation is too strong, causing discomfort or muscle twitching. Decreasing the electrical signal may be indicated if the sensation is too strong. Other options are not necessary because the tingling sensation is expected. 384- A female client is extremely anxious after being informed that her mammogram was abnormal and needs to be repeated. Client is tearful and tells the nurse her mother died of breast cancer. What action should the nurse take? a- Provide the client with information about treatment options for breast cancer. b- Reassure the client that the final diagnosis has not been made. c- Encourage the client to continue expressing her fears and concerns. d- Suggest to the client that she seek a second opinion. Rational: the nurse should show support for the client by encouraging her to continue expressing her concerns. A diagnosis has not yet been made, so it is too early to discuss treatment options. Other options dismiss the client’s feelings or are premature given that the diagnosis is not yet made. 385- The psychiatric nurse is talking to a newly admitted client when a male client diagnosed with antisocial behavior intrudes on the conversation and tells the nurse, “I have to talk to you right now! It is very important!” how should the nurse respond to this client? a- Put his behavior on extinction and continue talking with the newly admitted. b- Inform him that the nurse is busy admitting a new client and will talk to him later. c- Encourage him to go to the nurse’s station and talk with another nurse. d- Introduce him to the newly admitted client and ask him to him to join in the conversation. Rational: the psychiatric nurse must set limits with antisocial behavior so that appropriate behavior is demonstrated. Interrupting a conversation is rude and inappropriate, so telling the client that they can talk later is the best course of action. Other options may cause the client to become angry and they do not address the client’s behavior. The nurse should not involve this client with newly admitted client’s admission procedure. 386- The charge nurse is planning for the shift and has a registered nurse (RN) and a practical nurse (PN) on the team. Which client should the charge nurse assign to the RN? a- A 64-year-old client who had a total hip replacement the previous day. b- A 75-year-old client with renal calculi who requires urine straining. c- An adolescent with multiple contusions due to a fall that occurred 2 days ago. d- A 30-year-old depressed client who admits to suicide ideation. RATIONALE: A client who is suicidal requires psychological assessment, therapeutic communication and knowledge beyond the educational level of a practical nurse (RN). Other clients could be cared for by the PN or the UAP, with supervision by the registered nurse. 387- A female client presents in the Emergency Department and tells the nurse that she was raped last night. Which question is most important for the nurse to ask? a- Does she know the person who raped her? b- Has she taken a bath since the raped occurred? c- Is the place where she lived a safe place? d- Did she report the rape to the police Department? RATIONALE: The priority action is collected the forensic evidence, so asking if the has taken a bath since the rape occurred is the most important information to obtain. Other options are used by law enforcement to determine the perpetrator and are not vital in providing client care at this time. 388- While caring for a client’s postoperative dressing, the nurse observes purulent drainage at the wound. Before reporting this finding to the healthcare provider, the nurse should review which of the client’s laboratory values? a- Serum albumin b- Creatinine level c- Culture for sensitive organisms. d- Serum blood glucose (BG) level RATIONALE: A client who has a postoperative dressing with purulent drainage from the wound is experiencing an infection. The nurse should review the client’s laboratory culture for sensitive organisms (C) before reporting to the healthcare provider. (A, B and D) are not indicated at this time. 389- The nurse is demonstrating correct transfer procedures to the unlicensed assisted personnel (UAP) working on a rehabilitation unit. The UAPs ask the nurse how to safely move a physically disabled client from the wheelchair to a bed. What action should the nurse recommended? a- Hold the client at arm’s length while transferring to better distribute the body weight. b- Apply the gait belt around the client’s waits once standing position has been assumed. c- Place a client’s locked wheelchair on the client’s strong side next to the bed. d- Pull the client into position by reaching from the opposite side of the bed. RATIONALE: Placing the wheelchair on the client’s strong side offers the greatest stability for the transfer. Holding the client arm’s length or pulling from the opposite site of the bed reflect poor body mechanism. Using a gait belt offers additional safety for the client but should be done after the wheelchair has be put into the proper place and the wheels have been locked and before the client has assumed a standing position. 390- A client who is experiencing musculoskeletal pain receives a prescription for ketorolac 15mg IM q6 hours. The medication is depended in a 30mg/ml pre-filled syringe. Which action should the nurse implement when giving the medication? a- Administer the entire pre-filled syringe deep in the dorsogluteal site. b- Use a separate syringe to remove 15mg from the pre-filled syringe and give in the back of the arm. c- Waste 0.5 ml from the pre-filled syringe and inject the medication in the ventrogluteal site. d- Call the healthcare provider to request a prescription change to match the dispensed 30mg dose. RATIONALE: The pre-filled contain 30mg /1ml, so 0.5ml should be wasted to obtain the correct dosage of 15mg for administration in the preferred IM ventrogluteal site. The nurse is responsible for calculating and preparing the prescribed dose using the available concentration, so other options are not indicated. 391- A client with a lower respiratory tract infection receives a prescription for ciprofloxacin 500mg PO q 12hours. When the client requests an afternoon snack, which dietary choice should the nurse provide? a- Vanilla-flavored yogurt b- Low fat chocolate milk. c- Calcium fortified juice d- Cinnamon applesauce RATIONALE: Dairy products and calcium fortified dairy products decrease the absorption of ciprofloxacin. Cinnamon applesauce contains no calcium, so this is the best snack selection. Since other options contains calcium, these snacks should be avoided by a client who is taking ciprofloxacin. 392- The healthcare provider prescribes a low-fiber diet for a client with ulcerative colitis. Which food selection would indicate to the nurse the client understands they prescribed diet? a- Roasted turkey canned vegetables b- Baked potatoes with skin raw carrots c- Pancakes whole-grain cereal's d- Roast pork fresh strawberries Rationale: Foods allowed on a low-fiber diet includes roasted or baked turkey and canned vegetables the foods in the other options are not low in fiber 393- An adult client with schizophrenia begin treatment three days ago with the Antipsychotic risperidone. The client also received prescription for trazodone as needed for sleep and clonazepam as needed for severe anxiety. When the client reports difficulty with swallowing, what action should the nurse take? a- Obtain a prescription for an anticholinergic medication b- Determine how many hours declined slept last night c- Administer the PRN prescription for severe anxiety d- Watch the thyroid cartilage move while the client swallows Rationale: Antipsychotic medications have an extrapyramidal side effects one of which is difficult to swallowing the nurse should obtain a prescription for an anticholinergic medication which is used for the treatment of extrapyramidal symptoms. Other options are not warranted actions based on the symptoms presented. 394- One year after being discharged from the burn trauma unit, a client with a history of 40% full-thickness burns is admitted with bone pain and muscle weakness. Which intervention should the nurse include in the clients plan of care? a- Encourage Progressive active range of motion b- Teach need for dietary and supplementary vitamin D3 c- Explain the need for skin exposure to sunlight without sunscreen d- Instruct the client to use of muscle strengthening exercises Rationale: Burn injury results in the acute loss of bone as well as the development of progressive vitamin D deficiency because burn scar tissue and adjacent normal-appearing skin cannot convert normal quantities of the precursors for vitamin D3 that is synthesized from ultraviolet sun rays which is needed for strong bones. Clients with a history of full thickness burns should increase their dietary resources of vitamin D and supplemental D3 (B). range of motion (A) and muscle strengthening exercises (D) do not treat he is underlying causes of the bone pain and weakness unprotected sunlight (C) should be avoided. 395- When teaching a group of school-age children how to reduce the risk of Lyme disease which instruction should the camp nurse include? a- Wash hands frequently b- Avoid drinking lake water c- Wear long sleeves and pants d- Do not share personal products Rationale: Lyme disease is it tick bone disorder and is transmitted to a child via a tick bite. Keeping the skin covered reduces the risk of being bitten by a tick. Other options are not reduce the risk for tick bites. 396- A native-American male client diagnosed with pneumonia, states that in addition to his prescribed medical treatment of IV antibiotics he wishes to have a spiritual cleaning performed. Which outcome statement indicates that the best plan of care was followed? a- Identifies his ethnocentric values and behaviors b- States an understanding of the medical treatment c- Participated actively in all treatments regimens d- Expresses a desire for cultural assimilation Rationale: indicates active participation by the client, which is required for treatment to be successful. The best plan of care should incorporate the valued and treatments of both cultures and in this case there is no apparent cultural clash between the two forms of treatment. The client has already identify he's cultural values (A). (B) Only considers one of the two treatment modalities desired by the client the client has already chosen how he wishes to assimilate his cultural values with the prescribed medical treatment (D). 397- A male client with cancer is admitted to the oncology unit and tells the nurse that he is in the hospital for palliative care measures. The nurse notes that the client’s admission prescription include radiation therapy. What action should the nurse implement? a- Ask the client about his expected goals for the hospitalization b- Explain the palliative care measures can be provided at home c- Notify do radiation department to withhold the treatment for now d- Determine if the client wishes to cancel further radiation treatment Rationale: Palliative care measures provide relief or control of symptoms, so it is important for the nurse to determine the client’s goals for symptom control while receiving treatment in the hospital. Although home care is available the client may not be legible for palliative care at home. Radiation therapy is an effective positive care measure used to manage symptoms and would be appropriate unless the radiation conflicts with the client goals. 398- A client with myasthenia Gravis (MG) is receiving immunosuppressive therapy. Review recent laboratory test results show that the client’s serum magnesium level has decreased below the normal range. In addition to contacting the healthcare provider, what nursing action is most important? a- Check the visual difficulties b- Note most recent hemoglobin level c- Assessed for he and Hand joint pain d- Observe rhythm on telemetry monitor Rationale: If not treated a low little Serum magnesium level can affect myocardial depolarization leading to a lethal arrhythmia, and the nurse should assess for dysrhythmias before contacting the healthcare provider. Other choices are common in MG but do not contribute the Safety risk of low magnesium levels. 399- A young adult female presents at the emergency center with acute lower abdominal pain. Which assessment finding is most important for the nurse to report to the healthcare provider? a- Pain scale rating at 9 on a 0-10 scale b- Last menstrual period was 7 weeks ago c- Reports white curdy vaginal discharge d- History of irritable bowel syndrome IBS Rationale: Acute lower abdominal pain in A young adult female can be indicative of an ectopic pregnancy, which can be life threatening. Since the clients last menstrual period was seven weeks ago a pregnancy test to be obtained to ruled out ectopic pregnancy, which can result in intra-abdominal hemorrhage caused by a ruptured Fallopian tube. Although the severity of pain requires treatment, the most significant finding is the clients last menstrual period. Other options are not the most important concerns. 400- A 154-pound client with diabetic ketoacidosis is receiving an IV of normal saline 100 ML with regular insulin 100 units. The healthcare provider prescribes a rate of 0.1 units/kg/hour. To deliver the correct dosage, the nurse should set the infusion pump to Infuse how many ml/hour? enter numeric value only a- 7 Rationale: Convert the client’s weight to kg, 2.2 pound: 1 kg:: 154 pounds: x kg = 154/2.2 = 70kg. Calculate the client infusion rate, 0.1 x 70 kg = 7 units/hour. Using the formula, D/H x Q = 7 units/hour / 100 units x 100 ml = 7ml / hour 401- The nurse is assessing a postpartum client who is 36 hours post-delivery. Which finding should the nurse report to the healthcare provider? a- White blood count of 19,000 mm3 b- Oral temperature of 100.6 F c- Fundus deviated to the right side d- Breasts are firm when palpated Rationale: A temperature greater than 100.4 F (38 C) (B), which is indicative of endometriosis (infection of the lining of the uterus), should be reported to the health care provider. (A and D) are findings that are within normal limits in the postpartum period. Fundal deviation to one side (C) is an expected finding related to a full bladder, so the nurse should encourage the client to void. 402- A nurse who is working in the emergency department triage area is presented with four clients at the same time. The client presented with which symptoms requires the most immediate intervention by the nurse? a- Low-grade fever, headache, and malaise for the past 72 hours b- Unable to bear weight on the left foot, with the swelling and bruising c- Chest discomfort one hour after consuming a large, spicy meal d- One-inch bleeding laceration on the chain of the crying five-year-old Rationale: Emergency triage involves quick assessment to prioritize the need for further evaluation and care. Those with trauma, chest pain, respiratory distress, or acute neurological changes are priority. In this example, while clients with other conditions require attention, the client with chest discomfort is at greatest risk and is a priority. 403- The nurse is planning to assess a client's oxygen saturation to determine if additional oxygen is needed via nasal cannula. The client has a bilateral below-the-knee amputation and pedal pulses that are weak and threaty. What action should the nurse take? a- Document that an accurate oxygen saturation reading cannot be obtained b- Elevate to client's hands for five minutes prior to obtaining a reading from the finger c- Increase the oxygen based on the clients breathing patterns and lung sounds d- Place the oximeter clip on the ear lobe to obtain the oxygen saturation reading Rationale: Pulse oximeter clips can be attached to the earlobe to obtain an accurate measurement of oxygen saturation. Other options will not provide the needed assessment. 404- A young adult male who is being seen at the employee health care clinic for an annual assessment tell the nurse that his mother was diagnosed with schizophrenia when she was his age and that life with a schizophrenic mother was difficulty indeed. Which response is best for the nurse to provide? a- Ask the client if he is worried about becoming schizophrenic at the age his mother was diagnosed. b- Encourage the client to seek genetic counseling to determine his risk for mental illness. c- Informed the client that his mother schizophrenic has affected his psychological development. d- Tell the client that mental illness has a familial predisposition, so he should see a psychiatrist. 405- A client on a long-term mental health unit repeatedly takes own pulse regardless of the circumstance. What action should the nurse implement? a- Overlook the client’s behavior. b- Distract client to interfere with the ritual. c- Ask why the client checks the pulse. d- Hold client’s hand to stop the behavior. 406- A client is discharged with automated peritoneal dialysis (PD) to be used nightly…which instructions should the nurse include? a- Wash hands before cleaning exit site b- Keep the head of the bed flat at night c- Feel for a thrill and a distal pulse nightly d- Do not get up if fluid is left in the abdomen Rationale: meticulous hand hygiene is essential when performing care for a peritoneal dialysis, infections is a common complication of peritoneal dialysis. 407- The charge nurse observes the practical nurse (PN) apply sterile gloves in preparation for performing a sterile dressing change. Which action by the PN requires correction by the charge nurse? a- Opening the package b- Picking up the second glove c- Picking up the first glove d- Positioning of the table 408- A male client reports to the clinic nurse that he has been feeling well and is often “dizzy” his blood pressure is elevated. Based on this finding, this client is at a greatest risk for which pathophysiological condition? a- Pulmonary hypertension b- Left ventricular hypertrophy c- Renal failure d- Stroke Rationale: The client is significantly elevated, and combined with his symptoms, places the client at risk of stroke… B is a compensatory change that is caused by hypertension and is not related to an acute BP elevation. A is a separate disease process, not directly linked to arterial hypertension. C… chronic condition that is not related to an acute BP elevation. 409- The nurse asks the parent to stay during the examination of a male toddler’s genital area. Which intervention should the nurse implement? a- Examine the genitalia as the last part of the total exam. b- Use soothing statements to facilitate cooperation c- Allow the child to keep underpants on to examine genitalia d- Work slowly and methodically so not to stress the child Rationale: Examination of a child’s genitalia is particularly stressful to toddles, so this assessment is best left as the last part of the examination. B are best done by a parent, not the nurse. The genitals must be completely visualized and sometimes palpates underwear for a brief period of. 410- The nurse is changing a client’s IV tubing and closes the roller clamp on the new tubing setup when the bag of solution is…. which action should the nurse take to ensure adequate filling of the drip chamber? a- Lower the IV bag to a flat surface b- Compress the drip chamber c- Open the roller clamp d- Squeeze the bag of IV solution Rationale: The nurse should fill drip chamber halfway by compressing it, and the roller clamp sho…prevent free flow of the solution into the tubing, which can create air bubbles and leaks from the… solution may be placed on a flat surface when spiking the bag with the tubing, but should be r… the drip chamber, Squeezing the bag of IV fluid does not ensure filling of the drip chamber. 411- …starting an Insulin infusion for a client with diabetes mellitus who is experiencing hyperglycemic hyperosmolar syndrome (HHS). In addition to the client’s glucose, which laboratory value is most important for the nurse to monitor? a- Urine ketones b- Urine albumin c- Serum protein d- Serum potassium Rationale: When insulin is infused, both glucose and potassium are able to enter the cells, lowering serum potassium levels…life-threatening hypokalemia does not develop, the nurse should monitor the serum potassium level (D). HHS does not typically develop ketosis (A). The presence of albumin in the urine (B) is an indicator of nephropathy… during acute HHS C is of less priority than D for client who are experiencing HHS 412- A young adult who is hit with a baseball bat on the temporal area of the left skull is conscious when admitted to the ED and is transferred to the Neurological Unit to be monitored for signs of closed head injury. Which assessment finding is indicative of a developing epidural hematoma? a- Altered consciousness within the first 24 hours after injury. b- Cushing reflex and cerebral edema after 24 hours c- Fever, nuchal rigidity and opisthotonos within hours d- Headache and pupillary changes 48 hours after a head injury 413- In planning strategies to reduce a client's risk for complications following orthopedic surgery, the nurse recognizes which pathology as the underlying cause of osteomyelitis? a- Infectious process b- Metastatic process c- Autoimmune disorder d- Inflammatory disorder 414- A client with bipolar disorder began taking valproic acid (Depakote) 250 mg PO three times daily two months ago. Which finding provides the best indication that the medication regimen is effective? a- The nurse note that no pills remain in the prescription bottle. b- The client serum Depakote level is 125 mcg/ml c- The family reports a great reduction in client’s maniac behavior d- The client denies any occurrence of suicidal ideation. 415- A client with a serum sodium level of 125 meq/mL should benefit most from the administration of which intravenous solution? a- 0.9% sodium chloride solution (normal saline) b- 0.45% sodium chloride solution (half normal saline) c- 10% Dextrose in 0.45% sodium chloride d- 5% dextrose in 0.2% sodium chloride 416- A client with Alzheimer’s disease falls in the bathroom. The nurse notifies the charge nurse and completes a fall follow-up assessment. What assessment finding warrants immediate intervention by the nurse? a- Urinary incontinence b- Left forearm hematoma c- Disorientation to surroundings d- Dislodge intravenous site Rationale: The left forearm hematoma may be indicative an injury, such as broken bone, that requires immediate intervention. A may be likely be due to the inability to use the toilet due to the fall. Disorientation is a common symptom of Alzheimer’s disease. IV Dislodged is not an urgent concern. 417- The nurse is triaging clients in an urgent care clinic. The client with which symptoms should be referred to the health care provider immediately? a- headache, photophobia, and nuchal rigidity b- high fever, skin rash, and a productive cough c- nausea, vomiting, and poor skin turgor d- malaise, fever, and stiff, swollen joints Rationale: Headache, photophobia, and nuchal rigidity are classic signs of meningeal infection, so this client should immediately be referred to the health care provider. AC D do not have priority of B 418- An adult male is brought to the emergency department by ambulance following a motorcycle accident. He was not wearing a helmet and presents with periorbital bruising and bloody drainage from both ears. Which assessment finding warrants immediate intervention by the nurse? a- Rebound abdominal tenderness b- nausea and projectile vomit c- rib pain with deep inspiration d- diminished bilateral breath sounds Rationale: Projective vomiting is indicative of increasing intracranial pressure, which can lead to ischemic brain damage or death, so this finding warrants immediate intervention. Rebound abdominal tenderness may indicate internal bleeding. Diminished breath sound may be related to pain. Rib pain with inspiration may indicate rib fracture. 419- After placement of a left subclavian central venous catheter (CVC), the nurse receives report of the x-ray findings that indicate the CVC tip is in the client’s superior vena cava. Which action should the nurse implement? a- Initiate intravenous fluid as prescribed b- Notify the HCP of the need to reposition the catheter c- Remove the catheter and apply direct pressure for 5 minutes. d- Secure the catheter using aseptic technique Rationale: Venous blood return to the heart and drains from the subclavian vein into the superior vena cava. The X-ray findings indicate proper placement of the CVC, so prescribed intravenous fluid can be started. A and B are not indicated at this time. The catheter should be secure immediate following insertion (C) 420- The nurse has received funding to design a health promotion project for African-American women who are at risk for developing breast cancer. Which resource is most important in designing this program? a- A listing of African-American women so live in the community b- Participation of community leaders in planning the program c- Morbidity data for breast cancer in women of all races d- Technical assistance to produce a video on breast self-examination. Rationale: When developing a culturally-competent health promotion project, the participation of stakeholders and community leaders is most important. A and B might be useful background information, but t=first the program should be developed. D may be useful fulfilling the plan developed by the health care team and the community leaders if funding for this assistance is included in the budget. 421- The home care nurse provides self-care instruction for a client chronic venous insufficiency cause by deep vein thrombosis. Which instructions should the nurse include in the client’s discharge teaching plan? Select all that apply a- Avoid prolonged standing or sitting b- Use recliner for long period of sitting c- continue wearing elastic stocking d- Maintain the bed flat while sleeping e- Cross legs at knee but not at ankle 422- The nurse is interviewing a client with schizophrenia. Which client behavior requires immediate intervention? a- Lip smacking and frequent eye blinking b- Shuffling gait and stooped posture c- Rocks back and forth in the chair d- Muscle spasms of the back and neck Rationale: An extrapyramidal symptom (EPS) characterized by abnormal muscle spasms of the neck (A) requires immediate intervention because it can cause difficulty swallowing and jeopardize the airway. Though (A, B and C) are also EPS caused by antipsychotic medication medications used to manage schizophrenia (D) has the highest priority to insure client safety is (A). 423- A male client was transferred yesterday from the emergency department to the telemetry unit because he had ST depression and resolved chest pain. When his EKG monitor alarms for ventricular tachycardia (VT), what action should the nurse take first? a- Determine the client’s responsiveness and respirations b- Bring the crash cart to the room to defibrillate the client. c- Immediately initiate chest compressions. d- Notify the emergency response team Rationale: Activities, such as brushing teeth, can mimic the waveform of VI, so first he client should be assessed (A) to determine if the alarm is accurate. The crash cart can be brought to the room by someone else and defibrillation (B) delivered as indicated by the client’s rhythm. Based on as assessment of the client, CPR© as summoning the emergency response team (D) may be indicated. 424- A client with a large pleural effusion undergoes a thoracentesis. Following the procedure, which assessment finding warrants immediate intervention by the nurse? a- The client has asymmetrical chest wall expansion b- The clients complain of pain at the insertion site c- The client chest’s x-ray indicates decreased pleural effusion d- The client’s arterial blood gases are pH 7.35, PaO2 85, Pa CO2 35, HCO3 26 Rationale: A potential complication of thoracentesis is a pneumothorax. The symptoms of a pneumothorax are uneven, unequal movement of the chest wall. A is an expected finding after the local anesthetic effects “wear off” B is a desired result of thoracentesis and C is within normal limits. 425- A client is receiving an IV solution labeled Heparin Sodium 20,000 Units in 5% dextrose injection 500 ml at 25 ml/hour. How many units of heparin is the client receiving each hour? a- 1000 units/hour Rationale:20000/500=40x25=1000 426- The nurse is preparing a client for discharge from the hospital following a liver transplant. Which instruction is most important for the nurse to include in this client’s discharge teaching plan? a- Monitor for an elevated temperature b- Measure the abdominal girth daily c- Report the onset of sclera jaundice d- Keep a record of daily urinary output Rationale: The client should be instructed to monitor or elevated temperature because immunosuppressant agents, which are prescribed to reduce rejection after transplantation, place the client at risk for infection. The client should recognize sign of liver rejection, such as sclera jaundice and increasing abdominal girths, but fever may be the only sign of infection. A is not as important and monitoring for signs of infection. 427- The nurse is conducting health assessments. Which assessment finding increases a 56-year-old woman’s risk for developing osteoporosis? a- Body mass index of (BMI) of 31 b- 20 pack-year history of cigarette smoking c- Birth control pill usage until age 45 d- Diabetes mellitus in family history Rationale: Cigarette smoking (2 packs/day x 310 years = 20 packs-year) increases the risk of osteoporosis. BMI of 30 or greater falls in the category of obesity which increase weight bearing that is protective against osteoporosis. C contain estrogens and are also protective against development of osteoporosis. D is not related to the development of osteoporosis. 428- A young couple who has been unsuccessful in conceiving a child for over a year is seen in the family planning clinic. During an initial visit, which intervention is most important for the nurse to implement? a- Determine current sexual practice b- Prepare a female client for an ultrasound c- Request a sperm sample for ovulation d- Evaluate hormone levels on both clients Rationale: First a history should be obtained including practices that might be related to the infertility, such as douching, daily ejaculation or the male partner’s exposure to heat, such as frequent sauna or work environment which can decrease sperm production (A B or C) may be indicated after a complete assessment is obtained. 429- The nurse administers an oral antiviral to a client with shingles. Which finding is most important for the nurse report to the health care provider? a- Decreased white blood cell count b- Pruritus and muscle aches c- Elevated liver function tests d- Vomiting and diarrhea Rationale: Elevated liver function enzymes are a serious side effect of antivirals and should be reported. A decrease white blood count is a consistent finding with shingle B and (C and D) are side effects that affect that are of less priority than A 430- A client in the intensive care unit is being mechanically ventilated, has an indwelling urinary catheter in place, and is exhibiting signs of restlessness. Which action should the nurse take first? a- Review the heart rhythm on cardiac monitors b- Check urinary catheter for obstruction c- Auscultated bilateral breath sounds d- Give PRN dose of lorazepam (Ativan) Rationale: Restlessness often results from decreased oxygenation, so breath sounds should be assessed first. Giving an anxiolytic such as lorazepam, might be indicated but first the client should be assessed for the cause of the restlessness. An obstruction in the urinary drainage system can cause a distended bladder that may result in restlessness, but patent airway is the priority intervention. The client should be assessed before evaluating the cardiac rhythm on the monitor. 431- The nurse makes a supervisory home visit to observe an unlicensed assistive personnel (UAP) who is providing personal care for a client with Alzheimer’s disease. The nurse observes that whenever the client gets upset, the UAP changes the subject. What action should the nurse take in response to this observation? a- Tell the UAP to offer more choices during the personal care to prevent anxiety b- Meet with the UAP later to role model more assertive communication techniques c- Assume care of the client to ensure that effective communication is maintained. d- Affirm that the UAP is using and effective strategy to reduce the client’s anxiety. Rationale: Reduction is an effective technique is managing the anxiety of client with Alzheimer’s disease, so the nurse should affirm the UAP is using an effective strategy (A). Nurse assertive communication and offering more choices (B) may increase… an agitation (C) is not indicated since the UAP is using redirection, an effective strategy. 432- An older female who ambulate with a quad-cane prefer to use a wheel chair because she has a halting and unsteady gait at times. Which interventions should the nurse implement? (Select all that apply) a- Move personal items within client’s reach b- Lower bed to the lower possible position c- Raise all bed rails when the client is resting d- Give directions to call for assistance e- Assist client to the bathroom in 2 hours. f- Encourage the use of the wheelchair Rationale: A client who needs assistive devices, such as quad-cane is at risk for falls. Precautions that should implement include ensuring that personal items are within reach the bed is in the lowest position and directions are given to call assistance to minimize the risk for falls. Frequently assisting the client to the bathroom help ensure this client does not go the bathroom by herself, thereby decreasing the possibility of falling. 433- In evaluating the effectiveness of a postoperative client’s intermittent pneumatic compression devices, which assessment is most important for the nurse to complete? a- Evaluate the client’s ability to use an incentive spirometer b- Monitor the amount of drainage from the client’s incision c- Observe both lower extremities for redness and swelling d- Palpate all peripheral pulse points for volume and strength Rationale: Intermittent compression devices (ICDs) are used to reduce venous stasis and prevent venous thrombosis in mobile and postoperative clients and its effectiveness is best assessed by observing the client’s lower extremities for early signs of thrombophlebitis. 434- A school-age child who weighs 42 pounds receives a post-tonsillectomy prescription for promethazine (Phenergan) 0.5 mg/kg IM to prevent postoperative nausea. The medication is available in 25 mg/ml ampules. How many ml should the nurse administer? (Enter numeric value only. If rounding is required, round to the nearest tenth). a- 0.4 Rationale: Convert pounds to kg 42lbs = 19.09 kg Next calculate to prescribed dose, 0.5 mg x 1909 kg = 9.545 Then use the desired dose/ dose on hand x volume on hand (9.545/25x1ml =0.3818=0.4 ml) Or use ratio proportion (9.545 mg: x ml = 25 mg: 1ml 25x = 9.545 X= 0.3818 = 0.4) 435- A nurse stops at the site of a motorcycle accident and finds a young adult male lying face down in the road in a puddle of water. It is raining, no one is available to send for help, and the cell phone is in the car about 50 feet away. What action should the nurse take first? a- Examine the victim’s body surfaces for arterial bleeding b- Stabilize the victim’s neck and roll over to evaluate his status c- Return to the car to call emergency response 911 for help d- Open the airway and initiate resuscitative measures 436- During a well-baby 6-month visit, a mother tells the nurse that her infant has had fewer ear infections than her 10-year-old daughter. The nurse should explain that which vaccine is likely to have made the difference in the siblings’ incidence of otitis media? a- Varicella Virus Vaccine Live b- Hemophilic Influenza Type B (HiB) vaccine c- Pneumococcal vaccine d- Palivizumab vaccine for RSV 437- The healthcare provider prescribes Morphine Sulfate Oral Solution 38 mg PO q4 hours for a client who is opioid-tolerant. The available 30 mL bottle is labeled, 100 mg/5 mL (20mg/mL) and is packaged with a calibrated oral syringe to provide to provide accurate dose measurements. How many mL should the nurse administer? (Enter the numerical value only. If rounding is required, round to the nearest tenth.) a- 1.9 Rationale: D/H x Q 38/20x1=1.9 mL 438- The nurses observe that a postoperative client with a continuous bladder irrigation has a large blood clot in the urinary drainage tubing. What actions should the nurse perform first? a- Observe the amount of urine in the client’s urinary drainage bag 439- Which medication should the nurse anticipate administering to a client who is diagnosed with myxedema coma? a- Intravenous administration of thyroid hormones b- Oral administration of hypnotic agents c- Intravenous bolus of hydrocortisone d- Subcutaneous administration of vitamin k Rationale: The high mortality of myxedema coma requires immediate administration of IV thyroid hormones (A). (B) Is contraindicated, because eves small doses can cause profound somnolence lasting longer than expected. (C) Is administered to clients diagnosed with adrenal insufficiency (Addisonian crisis) and (D) to clients who have had an overdose of warfarin. 440- The nurse who works in labor and delivery is reassigned to the cardiac care unit for the day because of a low census in labor and delivery. Which assignments is best for the nurse to give this nurse? a- Transfer a client to another unit b- Monitor the central telemetry c- Perform the admission d- Assist cardiac nurses with their assignments Rationale: When receiving staff from another specialty unit, the charge nurse should allow the nurse to assist where possible (D) without taking a client assignment so that the nurse is not asked to perform unfamiliar skills (A, B, C) are likely to involve skills the nurse is not accustomed to performing. 441- A client who had an emergency appendectomy is being mechanically ventilated, and soft wrist restrain are in place to prevent self extubation. Which outcome is most important for the nurse to include in the client’s plan of care? a- Understand pain management scale b- Maintain effective breathing patterns c- Absence of ventilator associated pneumonia d- No injuries refer to soft restrains occur Rationale: Basic airway management (B) is the priority. Pain management (A), risk of infection (C), and prevention of injury (D) do not have the same priority as (C) 442- After a routine physical examination, the healthcare admits a woman with a history of Systemic Lupus Erythematous (SLE) to the hospital because she has 3+ pitting ankle edema and blood in her urine. Which assessment finding warrants immediate intervention by the nurse? a- Dark, rust-colored urine b- Urine output 300 ml/hr c- Joint and muscle aches d- Blood pressure 170/98 Rationale: SLE can result in renal complication such as glomerulonephritis, which can cause a critically high blood pressure that necessitates immediate intervention. A, B and C are symptoms of glomerulonephritis and should be treated once the blood pressure is under control 443- The nurse is explaining the need to reduce salt intake to a client with primary hypertension. What explanation should the nurse provide? a- High salt can damage the lining of the blood vessels b- Too much salt can cause the kidneys to retain fluid c- Excessive salt can cause blood vessels to constrict d- Salt can cause information inside the blood vessels Rationale: Excessive salt intake can contribute to primary hypertension by causing renal salt retention which influence water retention that expands blood volume and pressure (ACD) are not believed to contribute to primary hypertension. 444- In assessing a pressure ulcer on a client’s hip, which action should the nurse include? a- Determine the degree of elasticity surrounding the lesion b- Photograph the lesion with a ruler placed next to the lesion c- Stage the depth of the ulcer using the Braden numeric scale d- Use a gloved finger to palpate for tunneling around the lesion Rationale: An ulcer extends into the dermis or subcutaneous tissue and is likely to increase in size and depth, so assessment should include photograph with measuring device to document the size of the lesion. 445- A nurse is planning discharge care for a male client with metastatic cancer. The client tells the nurse that he plans to return to work despite pain, fatigue, and impending death. Which goals is most important to include in this client’s plan of care? a- Implements decisions about future hospices services within the next 3 months. b- Maintaining pain level below 4 when implementing outpatient pain clinic strategies. c- Request home health care if independence become compromised for 5 days. d- Arranges for short term counseling stressors impact work schedule for 2 weeks. Rationale: An outpatient pain clinic provides the interdisciplinary services needed to manage chronic pain. Also, the client has a terminal disease and is being discharge home, hospice and health care are not indicating currently. Short term counseling is not an option. 446- The first paddle has been placed on the chest of a client who needs defibrillation. Where should the nurse place the second paddle? (Mark the location where the second paddle should be placed on the image). a- 447- A client who had an open cholecystectomy two weeks ago comes to the emergency department with complaints of nausea, abdominal distention, and pain. Which assessment should the nurse implement? a- Auscultate all quadrant of the abdomen. b- Perform a digital rectal exam c- Palpate the liver and spleen d- Obtain a hemoccult of the client’s stool 448- The nurse is caring for several clients on a telemetry unit. Which client should the nurse assess first? The client who is demonstrating a- A paced rhythm with 100% capture after pacemaker replacement b- Normal sinus rhythm and complaining of chest pain c- Atrial fibrillation with congestive heart failure and complaining of fatigue d- Sinus tachycardia 3 days after a myocardial infarction 449- A 12-lead electrocardiogram (ECG) indicates a ST elevation in leads V1 to V4, for a client who reports having chest pain. The healthcare provider prescribes tissue plasminogen activator (t-PA). Prior to initiating the infusion, which interventions is most important for the nurse to implement? a- Complete pre-infusion checklist 450- The nurse is evaluating the health teaching of a female client with condyloma acuminate. Which statement by the client indicates that teaching has been effective? a- These warts are caused by a fungus b- Early treatment is very effective c- I need to have regular pap smears d- I will clean my hot tub better Rationale: Condyloma acuminate, warts caused by the HPV, is associated with… smear screening for early cervical changes is very important for clients with this rather than fungus. Early treatment is often difficult and painful… 451- While the nurse is conducting a daily assessment of an older woman who resides in a long-term facility, the client begins to cry and tells the nurse that her family has stopped calling and visiting. What action should the nurse take first? a- Ask the client when a family member last visited her. b- Determine the client’s orientation to time and space c- Review the client’s record regarding social interactions d- Reassure the client of her family’s love for her 452- A female client with severe renal impairment is receiving enoxaparin (lovenox) 30 mg SUBQ BID. Which laboratory value due to enoxaparin should the nurse report to the healthcare provider? a- creatinine clearance 25 mL/ minute b- calcium 9 mg/dl c- hemoglobin 12 grams/dl d- partial thromboplastin time (PTT) 30 seconds 453- The nurse notes an increase in serosanguinous drainage from the abdominal surgical wound from an obese client. What action should the nurse implement? a- Observe the wound for dehiscence b- Teach the client to splint the incision while coughing c- Assess the skin surrounding the wound for maceration d- Obtain a culture of the wound drainage. Rationale: The abdominal surgical wound of a client who is obese is at risk for dehiscence (opening...) layer. An increase in serosanguinous drainage may indicate that the wound edges have s… should observe the wound. Splinting the incision is useful in reducing discomfort and all… to obtain culture, and there is no indication that this client has such drainage. 454- The nurse is assigned to care for clients on a medical unit. Based on the notes taken during the shift report, which client situation warrants the nurse’s immediate attention? a- A young adult with Crohn’s disease who reports having diarrheal stools b- An older adult with type 2 diabetes whose breakfast tray arrives 20 minutes late. c- A 10-year-old who is receiving chemotherapy and the infusion pump is beeping. d- A teenager who reports continued pain 30 minutes after receiving an oral analgesic. Rationale: The nurse should immediately assess the child whose infusion pump is alarming during chemotherapy administration because infiltration of a caustic agent can cause tissue damage and children are at greater risk for fluid volume imbalance. Diarrhea is a common occurrence for Crohn’s disease. Late consumption of food for a diabetic is of concern, but 20 minutes late is usually not life-threatening. Treatment of pain is most important but has been only 30 mints since the client was medicated and this issue can be assessed in 10 mints or delegated to another nurse. 455- A nurse is conducting a physical assessment of a young adult. Which information provides the best indication of the individual nutritional status? a- A 24-hour diet history b- History of a recent weight loss c- Status of current petite d- Condition of hair, nails, and skin Rationale: The assessment of hair, nails and skin is most indicative of long-term nutritional status, which is important in the healing process. 456- The nurse is preparing to administer an infusion of amino acid-dextrose total parenteral nutrition (TPN) through a central venous catheter (CVC) line. Which action should the nurse implement first? a- Attached de IV tubing to the central line. b- Check the TPN solution for cloudiness c- Set the infusion PUMP at the prescribed rate. d- Prime the IV tubbing with the TPN solution. Rationale: The solution should be checked for any cloudiness or particles prior to A C or D 457- A newly admitted client vomits into an emesis basin as seen in the picture. The nurse should consult with the healthcare provider before administering which of the clients prescribes medications? a- Clopidogrel (Plavix), an antiplatelet agent, given orally b- Methylprednisolone (solu-medrol), a corticosteroid, to be given IV c- Nitroglycerin (Nitro-Dur) an antianginal, to be given transdermally. d- Enoxaparin (lovenox), a low-molecular weight heparin to be given subcutaneous. e- Furosemide (Lasix), a loop diuretic, to be given intravenously. Rationale: Because of the emesis is coffee brown appearance, which is an indicator of bleeding in GI track, the nurse should consult the health care provider because increase the risk of bleeding. 458- A client diagnosed with bipolar disorder is going home on a week-end pass. Which suggestions should give the client’s family to help them prepare for the visit? a- Discuss the importance of continuing the usual at-home activities b- Encourage the family to plan daily activities to keep the client busy c- Have friends and family visit the client at a welcome party. d- Instruct family to monitor the client’s choice of television programs. Rationale: Week-end pass are schedules to help the client ease back into the family’s routine, so the client can back to normal activities. 459- On a busy day, one hour after the shift report is completed, the charge nurse learns that a female staff nurse who lives one hour away from the hospital forgot her prescription eye glasses at home. What action should the charge nurse take? a. Encourage the nurse purchase the reading glasses in the hospital gift shop b. Request another nurse to assist the staff nurse with her documentation c. Ask the nurse to return home and get her prescription eyeglasses for work. d. Tell the staff nurse to take a day off and change her weekly work schedule. Rationale: The ability of the staff nurse to function is affected by her visual acuity, and no having her prescription. Even though it is a busy day, the staff nurse should be asked to obtain her prescription glass. Current staff should be notified that additional help may be needed until the staff nurse returns. 460- A client with pneumonia has an IV of lactated ringer’s solution infusing at 30ml/hr current labor.…sodium level of 155 mEq/L, a serum potassium level of 4mEq/L or mmol/ L and an increase…. what nursing intervention is most important? a- Provide a high-potassium snack, such as bananas. a- Obtain a prescription to increase the IV rate b- Administer the next scheduled dose of antibiotic c- Review the report of the most recent chest x-ray. Rationale: The priority action is to promote increased fluid intake, either orally of by increase in the IV rate. Patient is exhibiting signs of fluid volume deficit (hypernatremia and increase serum osmolarity) and is necessary to increase the IV rate. The potassium level is normal, so A is not a priority now. The fluid volume deficit …………… pneumonia, but fluid replacement should be initiated before C. The report of the chest x-ray information about the resolution of the pneumonia, but the resolution of the fluid deficit is of higher importance. 461- After teaching a male client with chronic kidney disease (CKD) about therapeutic diet…which menu of foods indicates that the teaching was effective? Select all that apply a- A slice of whole grain toast b- Half cup of black beans c- A ham and cheese sandwich d- A bowl of cream of wheat e- Two bananas. Rationale: Patient with CKD have elevated serum potassium, sodium and protein levels. A and D are low in potassium, sodium and protein, Beans are rich in proteins. C are high in sodium and potassium and E are rich in potassium. 462- When five family members arrive at the hospital, they all begin asking the nurse questions regarding the prognosis of their critically ill mother. What intervention should the nurse implement first? a- Include the family in client’s care b- Request the chaplain’s presence c- Ask the family to identify a specific spokesperson d- Page the healthcare provider to speak with family. Rationale: First the nurse should ask for a designated family spokesperson which should eliminate multiple…. The client’s prognosis is poor; a chaplain’s services may be needed. Allowing the family to assist …. Them understand more about the client’s condition and limitations (B) one the spokesperson is identified can be paged to arrange a time to meet with the family (D). 463- An older male who is admitted for end stage of chronic obstructive pulmonary disease (COPD) tells the nurse …. CPR to provided is the need arises. The client provides the nurse with a living will and DNR. What action should the nurse implement? a- Inform the family of the client whishes b- Obtain a prescription for DNR c- Clergy consultation d- Ask the patient why he made this choice Rationale: The healthcare provider should be notified, and a prescription obtained that reflects the client’s whishes… decision maker regarding treatment or refusal of treatment. A and C violate the client’s privacy. D provides useful information but does not address the legal issue. 464- A client who is recently diagnosed with type 2 diabetes mellitus (DM) ask the nurse how this type of diabetes leads to high blood sugar. What Pathophysiology mechanism should the nurse explain about the occurrence of hyperglycemia in those who have type 2 DM? a- Immune antibodies attack pancreatic beta cells resulting in no insulin b- The body cells develop resistance to the action of insulin. c- Body organs produce less insulin and more glucagon d- The liver produces excess glucose in response to excess glycotrophic hormones Rationale: Normally insulin activates a glucose transporter in cell walls. In type 2 DM, the body’s cell wall develop resistance to the action of insulin. A describe mechanism for type 1 diabetes mellitus, which is an autoimmune process………. production occurs in an effort to facilitate glucose transport into the cells not C. The problem is……. Insufficient insulin production, not D. 465- During a left femoral artery aortogram, the healthcare provider inserts an arterial sheath and initiate. Through the sheath to dissolve an occluded artery. Which interventions should the nurse implement? a- Instruct the client to keep the left leg straight b- Keep the head of bed at 60-degree angle. c- Observe the insertion site for a hematoma d- Manually flush the arterial sheath hourly e- Circle first noted drainage on the dressing 466- A client whose wrists are sutured from a recent suicide attempt is been transferred from a medical unit. Which nursing diagnosis is of the highest priority? a- Risk for self-directed violence related to impulsive actions b- Risk for violence related to feeling of guilt and failure c- Low self-esteem related to feeling of loss of control d- Ineffective coping related to violent actions towards self. Rationale: Usually B, C and D occur after an unsuccessful suicide act, but these diagnoses do not have the highest priority. 467- The nurse reviews the signs of hypoglycemia with the parents of a child with Type I diabetes mellitus. The parents correctly understand signs of hypoglycemia if they include which symptoms? a- Fruity breath odor b- Polyphagia c- Diaphoresis d- Polydipsia Rationale: Sweating or diaphoresis is a sign of hypoglycemia. The other symptoms not. 468- One day following a total knee replacement, a male client tells the nurse that he is unable to transfer because it is too painful. What action should the nurse implement? a- Encourage use of analgesics before position change b- Assess anxiety about transferring to commode chair c- Assist client during transfer on the first two days d- Review use of assistive devices for weight bearing. Rationale: Controlling the amount of pain prior to activity with the use of analgesics (A) enhances client’s…. 469- The nurse is caring for a client with hypovolemic shock who is receiving two units of packed red blood cells (RBCs) through a large bore peripheral IV. What action promotes maintenance of the client’s cardiopulmonary stability during the blood transfusion? a- Increase the oxygen flow via nasal cannula if dyspnea is present. b- Place in a Trendelenburg position to increase cerebral blood flow c- Monitor capillary glucose measurements hourly during transfusion. d- Encourage increased intake of oral fluid to improve skin turgor. Rationale: Increasing the client’s oxygen flow rate has an immediate impact on cardiopulmonary stability since provides increased hemoglobin for oxygen transport to the tissues. Oral fluids contribute to fluid retention to be adequate to reestablish or maintain cardiopulmonary stability for a client with hypovolemic shock… during a transfusion and is monitored in septic shock. Slightly elevating the client’s legs may promote… but placing the client in a Trendelenburg position can reduce both the client’s systemic blood pressure. 470- Which information is more important for the nurse to obtain when determining a client’s risk for obstruction (Obstructive Sleep Apnea Syndrome OSAS)? e- Body mass index f- Level of consciousness g- Self-description of pain h- Breath sounds Rationale: Obesity is a risk factor for OSAS, and calculation of Body Mass Index BMI provides date related to… provide data related to risk for OSAS. 471- During the transfer of a client who had major abdominal surgery this morning, the post anesthesia care unit (PACU) nurse reports that the client, who is awake and responsive continues to report pain and nausea after receiving morphine 2 mg IV and ondansetron 4 mg IV 45 mints ago. Which elements of SBAR communication are missing from the report given by the PACU nurse? (Select all that apply) b- Situation c- Background d- Assessment e- Recommendation f- Rationales. Rationale: BCD are correct. The current situation is reported regarding the client’s nausea and pain (A). Based on SBAR communication, critical information about the client’s clinical history (B), and assessment (C) such as pain scale or vital signs related to client’s response to medication, are not included, nor are any recommendations for further follow-up (D). (E) Is not a component of SBAR communication 472- The nurse is triaging victims of a tornado at an emergency shelter. An adult woman who has been wandering and crying comes to the nurse. What action should the nurse take? a- Check the client’s temperature, blood sugar, and urine output. b- Transport the client for laboratory client for laboratory test and electrocardiogram (EKG) c- Delegate care of the crying client to an unlicensed assistant d- Send the client to the shelter’s nutrient center to obtain water and food. Rationale: According to the simple triage and Rapid Treatment (START) protocol of triage, the nurse should determine which client fit the objective of providing the greatest good for the greatest number of people who are most likely to survive. Delegating the care of the crying person to an unlicensed assistant allow the nurse to care for the injured who require intervention based on their ability to breath, maintain circulation and follow simple commands. A and B are not indicated at this time. Although food and water may be indicative, the woman’s distress should not be dismissed by sending her to the shelter alone. 473- A client in septic shock has a double lumen central venous catheter with one liter of 0.9% Normal Saline Solution infusing at 1 ml/hour through one lumen and TPN infusing at 50 ml/hr. through one port. The nurse prepared newly prescribed IV antibiotic that should take 45 mints to infuse. What intervention should the nurse implement? e- Use a secondary port of the Normal Saline solution to administer the antibiotic. f- Add the antibiotic to the TPN solution, and continue the normal saline solution. g- Stop the TPN infusion for the time needed to administer the prescribed antibiotic. h- Add the antibiotic to the Normal Saline solution and continue both infusions. Rationale: A client in septic shock needs antibiotic administered in a timely manner to ensure maintenance of therapeutic serum level. The nurse should administer the antibiotic using a secondary port of the Normal Saline solution. No other medications should be administered using TPN tubing or solution. TPN not should be place on hold because sudden cessation will cause rapid change in serum glucose levels. Excessively delays in the administration of the antibiotics. 474- A male client returns to the mental health clinic for assistance with his anxiety reaction that is manifested by a rapid heartbeat, sweating, shaking, and nausea while driving over the bay bridge. What action I the treatment plan should the nurse implement? i- Tell the client to drive over the bridge until fear is manageable j- Teach client to listen to music or audio books while driving k- Encourage client to have spouse drive in stressful places. l- Recommend that the client avoid driving over the bridge. m- Rationale: Desensitization is component in the treatment plan for clients with panic attacks which is best approached with anxiety-reducing strategies, such as listening to audio book (B) during situation that precipitate symptoms (A) is a flooding technique that requires professional guidance. 475- Which intervention should the nurse include in the plan of care for a client with leukocytosis? n- Avoid intramuscular injections o- Monitor temperature regularly p- Assess skin for petechiae or bruising q- Implement protective isolation measures Rationale: Leukocytosis refers to a high white blood cells count, which place the client at risk for infection, so temperature should be monitor regularly. 476- The nurse is teaching a client about the antiulcer medications ranitidine which was… statement best describes the action of this drug? a- It blocks the effects of histamine, causing decreased secretion of acid b- Ranitidine will neutralize gastric acid and decrease gastric pH c- This drug provides a protective coating over the gastric mucosa d- It effectively blocks 97% of the gastric acid secreted in the stomach Rationale: Ranitidine is a histamine 2 receptor antagonist and A is the best describes these drugs. C describe the action of sucralfate and D omeprazole. 477- A client with superficial burns to the face, neck, and hands resulting from a house fire…which assessment finding indicates to the nurse that the client should be monitored for carbon monoxide…? e- Expiratory stridor and nasal flaring f- Mucous membranes cherry red color g- Carbonaceous particles in sputum h- Pulse oximetry reading of 80 percent The saturation of hemoglobin molecules with carbon monoxide causes vasodilation and mucous membrane turned in cherry red color. 478- A female client who was mechanically ventilated for 7 days is extubated. Two hours later…productive cough, and her respirations are rapids and shallow. Which intervention is most important? i- Review record of recent analgesia j- Provide frequent pulmonary toilet k- Prepare the client for intubation l- Obtain STAT arterial blood gases Rationale: respirations are rapids and shallow are sign of imminent respiratory failure. 479- The nurse delegates to an unlicensed assistive personnel (UAP) denture care for a client with…daily leaving. When making this assignment, which instruction is most important for the nurse to do? m- Do not remove the dentures, but instead brush them within the mouth n- Place a washcloth in the sink while cleaning the dentures. o- Use tepid, not hot, water to clean the dentures p- Avoid damaging the dentures using a soft-bristled toothbrush. 480- The nurse is assessing the emotional status of a client with Parkinson’s disease. Which client… goals to meet the client’s emotional needs? q- Stares straight ahead without blinking r- Face does not convey any emotion s- Cries frequently during the interview t- Uses a monotone when speaking 481- When changing a diaper on a 2-day-old infant, the nurse observes that the baby’s legs are… this finding, what action should the nurse take next? a- Notify the healthcare provider b- Continue care since this is a normal finding c- Document the finding in the record d- Perform range of motion to the joint. Rationale: Limited abduction could indicate developmental hip displasia and the helthcare provider should be notified. 482- A school-aged child was recently diagnosed with celiac disease. Which instruction should the nurse give the classroom teacher? e- The child should avoid eating homemade cookies and cupcakes during parties. f- No products containing any form of peanuts should be allowed in the classroom g- Report a runny nose or head cold to the nurse immediately for further revaluation. h- Avoiding direct contact sports and games will reduce the child’s risk of bruising. Rationale: Celiac disease is an intolerance to products containing gluten. Wheat flour, used to make… 483- The nurse is presenting information about fetal development to a group of parents with…When discussing cephalocaudal fetal development, which information should the nurse gives the parents? a- A set order in fetal development is expected b- Growth normally occurs within one organ at a time c- Development progress from head to rump d- Organ formation is directed by brain development Rationale: Fetal system are developed in a predetermined order, best described by in the … true, cephalocaudal describes the specific order of fetal system development. 484- A client has a prescription for lorazepam 2mg for alcohol withdrawal symptoms. Which finding… the client? e- Blood pressure 149/101 f- Irregular pulse rate of 80 g- Oral temperature is 98.9 F (37.1 C) h- Pain rated 7 on scale 1-10 Rationale: Alcohol sedate s the CNS…. Adrenergic hypersensitivity, which is manifested as an elevated blood pressure. 485- A client with end-stage liver failure is declared brain dead. The family wants to discontinue feeding and donate any viable organs. Which action should the nurse take? a- Contact the regional organ procurement agency b- Convene a multidisciplinary care conference c- Explain that client may not be an organ donor candidate d- Discontinue feeding and fluids per the family’s request. Rationale: First the organ procurement agencies should be contacted because they are trained to work with... procurement process. The procurement agency is responsible for calling a care conference (B), disc…organs are eligible for procurement (C), and specifying when to discontinue nutrition and hydration 486- A male client who was hit by a car while dodging through traffic is admitted to the emergency department with intracranial pressure (ICP). A computerized tomography (CT) scan reveals an intracranial bleed. After evacuation of hematoma, postoperative prescription includes: intubation with controlled mechanical ventilation to PaCO2…what is the pathophysiological basis for this ventilator settings? a- Hypoxemia reduces ICP. b- Hypocapnea reduces ICP. c- Hyperventilation reduces need for temperature control. d- Controlled ventilation reduces need for oxygen to brain. Rationale: The treatment goal after a cerebral bleed is reduction of ICP. Elevated carbon dioxide (PaCO2) causes… to increase cerebral perfusion, but increasing blood volume in a confined space increases the pressure... (hypocapnea) leads to a vasoconstriction, thereby lowering ICP. Controlled ventilation increases oxygen supply, but does not influence oxygen need (D) 487- During a cardiopulmonary resuscitation of an intubated client, the nurse detects a palpable pulse throughout the two minutes’ cycle chest compression and absent breath sounds over the left lung. What action should the nurse implement? e- Instruct the compressor to stop chest compression. f- Advise ventilator to increase bag-mask ventilation rate. g- Plan to suction endotracheal tube at two-minute check. h- Prepare for the endotracheal tube to be repositioned Rationale: The absent breath sound on the left indicate that the endotracheal tube was advanced too fat in the bronchus. The tube needs to be repositioned by pulling it back a few centimeters and reassess palpable pulse is being generated by the compressions and should not be stopped. 488- A male client is admitted with burns to his face and neck. Which position should the nurse place the client to prevent contract? a- Flexed with the chin toward the chest. b- Hyperextended with neck supported by a rolled towel. c- Side-lying with the head on a pillow d- Prone with face supported by an inflated rubber ring. Rationale: The position of function is hyperextension of the head and neck. A is contracted position and results in limited motion is not satisfactory unless the neck is hyperextended D requires that the client turn his head to the side, put his face in the pillow, mattress or an inflated rubber ring, all of which are undesirable…. 489- A male client is discharged from the intensive care unit following a myocardial infarction, and the healthcare provider low-sodium diet. Which lunch selection indicates to the nurse that this client understands the dietary restrictions? a- Turkey salad sandwich. b- Clam chowder c- Macaroni and cheese d- Bacon, lettuce, and tomato sandwich Rationale: 490- The nurse prepares an intravenous solution and tubing for a client with a saline lock, as seen in the video. Which nurse takes next e- f- 0 a- Attachd the tubing to the saline lock b- Open the roller clamp on the tubing. c- Label the bag on IV solution. d- Flush the saline lock with saline Rationale: After connecting the IV tubbing to the IV solution, the nurse needs to clear the air out of the tubing first squeezing the drip… chamber and then opening the roller clamp (B) to allow IV fluid to displace air out of the tubing. After the tubing is cleared… of the saline lock can be flushed 9D) and the tubing connected to the lock (A). (C) is indicated if additional medication is added to.. IV solution. 491- The healthcare provider prescribes heparin protocol at18 units/kg/hr for a client with a possible pulmonary embolism. This client weighs 144 pounds. The available solution is labeled, heparin sodium 25,000 units in 5% dextrose 250 ml. the nurse should program the pump to deliver how many ml/hr? (Enter numeric value only. If rounding is requiring round to the nearest whole number.) a- Answer 12 Rationale: 144/2.2= 65kg 18units/kg/hr 65 kg x 18units/kg/hr= 1170 units/hr 25000 units heparin/250 ml of D5W = 100 units heparin per ml of solution Formula D/H x A = X 492- A client is admitted with a wound on the right hand and associated cellulitis. In assessing the client’s hand, which finding required most immediate follow-up by the nurse? a- Localized tenderness b- Diffuse erythema c- Skin hot to touch d- Cyanotic nailbeds 493- The nurse is arranging home care for an older client who has a new colostomy following a large bowel resection three day. The clients plan to live with a family member. Which action should the nurse implement? Select all that apply a- Assess the client for self-care ability b- Provide pain medication instructions c- Call home care agency to set up oxygen d- Request a home safety inspection e- Teach care of ostomy to care provider. 494- A female client with chronic urinary retention explains double voiding technique to the nurse by stating she voids partially, hold the remaining urine in her bladder for three minutes, then voids again to empty her bladder fully. How should the nurse respond? a- Affirm that the client is effectively performing the double voiding. b- Advise the client to empty her bladder fully when she first voids c- Suggest that the client drink water between the two voiding. d- Explain that Kegel exercises help promote full bladder empty. Rationale: Double voiding is a technique that promotes more complete empty bladder in those with chronic urinary retention. The client should empty the bladder completely during the first void, then wait three minutes and void again. By partially voiding during the first void, the client is not using the most effective technique. 495- When conducting diet teaching for a client who was diagnosed with hypoparathyroidism, which foods should the nurse encourage the client to eat? a- Nuts b- Yogurt. c- Fresh turkey d- Fresh chicken e- Processed cheese. Rationale: In hypoparathyroidism, the client’s diet should be supplemented with calcium rich foods which include dairy products. 496- The nurse is assessing a middle-aged adult who is diagnosed with osteoarthritis. Which factor in this client’s history is a contributor to the osteoarthritis? a- Long distance runner since high school. b- Lactose intolerant since childhood c- Photosensitive to a drug currently taking d- Recently treated for deep vein thrombosis. Rationale: Osteoarthritis is a degenerative joint disease of the cause by traumatic or repetitive stress to weight-bearing joint such as high impact sport like running. 497- When assessing a male client, the nurse notes that he has unequal lung expansion. What conclusion regarding this finding is most likely to be accurate? The client has a- A collapsed lung b- A history of COPD c- A chronic lung infection d- Normally functioning lungs Rationale: Unilateral absence of chest movement (or unequal lung expansion because one lung is not moving at all) may be indicative previous surgical removal of that lung, a bronchial obstruction, or a collapsed lung caused by air or fluid in the pleural space. 498- The nurse manager is conducting an in-services education program on the fire evacuation of the newborn recovery. What intervention should the nurse manager disseminate to the staff? a- Place infants on a blanket for evacuation via stairwell b- Secure three infants on a stretcher for transport c- Evacuate each infant with mother via wheelchair d- Use the bassinet in evacuate two infants at a time. Rationale: Rooming-in and newborn babies are counted with their mothers. To exposure safety and accountability during the evacuations newborns should be evacuated with their mother in a wheelchair while maneuver with fire extinguisher are performed (PASS)… 499- An adult male reports that he recently experienced an episode of chest pressure and breathlessness when he was jogging in the neighborhood. He expresses concern because both of his deceased parents had heart disease and his father was a diabetic. He lives with his male partner, is a vegetarian, and takes atenolol which maintain his blood pressure at 138/74. Which risk factors should the nurse explore further with the client? Select all that apply a- History of hypertension. b- Homosexual lifestyle c- Vegetarian diet d- Excessive aerobic exercise e- Family heath history. Rationale: Based on the client’s family history and medication for management of hypertension, the nurse should further explore these risk for ischemic heart disease. 500- A client with severe full-thickness burns is scheduled for an allografting procedure. Which information should the nurse provide the client? a- The donor site will be painless a few days after the surgery b- Allografts are made from human and nonhuman material sources. c- Human sources graft require monitor for signs of graft injection d- Human source grafts require monitoring for signs of graft rejection. Rationale: Allograft is a graft created from the client’s own skin, which is called harvest site. All types of grafts, from human and nonhuman sources should be monitor for signs of rejection. Graft site are painful. (A). Allografts are obtained from the client, which is a human source (B). scaring does occur under the graft (D) 501- The nurse is administering a 750 ml cleansing enema to an adult client. After approximately150 ml of enema has informed, the client states, ‘stop I can’t hold anymore.” What action should the nurse take? a- Clamp the tubing and instruct the client to breathe deeply before continuing. b- Discontinue infusing the enema and record the client’s response. c- Slow infusion of the enema and instruct the client to use paint breathing d- Place the client on the bedpan and continue infusion of the enema. Rationale: Clamping the tube momentarily allows the muscle to relax and prevents expulsion of the solution prematurely. B may be eventually necessary but A should be tried first. 502- The nurse requests a meals tray for a client follows Mormon beliefs and who is on clear liquid diet following abdominal surgery. Which meal item should the nurse request for this client? (Select all that apply) a- Apple juice b- Chicken broth. c- Hot chocolate d- Orange juice e- Black coffee Rationale : Apple juice and chicken broth are included in a clear diet liquid and are consumed by Mormons, B is included in clear liquid diet but caffeinated beverage are not consumed by Mormons. Hot chocolate and orange juice are not considered clear liquid diet. 503- The nursing staff on a medical unit includes a registered nurse (RN), practical nurse (LPN), and unlicensed assistant personnel (UAP). Which task should the charge nurse assign to the RN? f- Supervised a newly hired graduate nurse during an admission assessment 504- Following breakfast, the nurse is preparing to administer 0900 medications to clients on a medical floor. Which medication should be held until a later time? a- The loop-diuretic furosemide (Lasix) for a client with a serum potassium level of 4.2 mEq/L b- The mucosal barrier, sucralfate (Carafate), for a client diagnosed with peptic ulcer disease. c- The antiplatelet agent aspirin, for a client who is scheduled to be discharged within the hour d- The antifungal nystatin (mycostatin) suspension, for a client who has just brushed his teeth. Rationale: Carafate coats the mucosal lining prior to eating a meal, so this medication should be held until prior to the next meal. 505- The father of 4-year-old has been battling metastatic lung cancer for the past 2 years. After discussing the remaining options with his healthcare provider, the client requests that all treatment stop and that no heroic measures be taken to save his life. When the client is transferred to the palliative care unit, which action is most important for the nurse working on the palliative care unit to take in facilitating continuity of care? a- Reassure the client that his child will be allowed to visit b- Obtain a detailed report from the nurse transferring the client. c- Mark the chart with client’s request for no heroic measure d- Provide the client whitening information about end-of-life care Rationale: To maintain continuity of care, it is important for the nurse working on the palliative care unit to obtain a detailed “situation, background, assessment, recommendation (SBAR) report, which provide clinical and no clinical information, as well as further information about the client may need. A, C and D are important intervention but not have priority at this time. 506- The nurse is making a home visit to a male client who is in the moderate stage of Alzheimer’s diseases. The client’s wife is exhausted and tells the nurse that the family plans to take turns caring for the client in their home, each keeping him for two weeks at a time. How should the nurse respond? a- Advise the client’s spouse to consider inpatient hospice care as an alternative b- Suggest that each rotation last one week, rather than two, to prevent caregiver fatigue c- Use active listening to allow the client and spouse to express their feelings about the plan d- Suggest enrolling the client in adult daycare instead of rotating among family. Rationale: Suggesting a viable alternative, such as adult daycare provides an option to allow the spouse respite the least disruption to routines and environment. 507- The healthcare provider prescribes oxycodone/ aspirin 1-tab PO every 4h as needed for pain, for a client with polycystic kidney disease. Before administering this medication, which component of the prescription should the nurse question? a- Aspirin content. b- Dose c- Route d- Risk for addiction Rationale: Aspirin content medication are contraindicated for client with polycystic kidney disease because the risk for bleeding. 508- A young adult male was admitted 36 hours ago for a head injury that occurred as the result of a motorcycle accident. In the last 4 hours, his urine output has increased to over 200 ml/H. Before reporting the finding to the healthcare provider, which intervention should the nurse implement? a- Evaluate the urine osmolality and the serum osmolality values. b- Obtain blood pressure and assess for dependent edema c- Measure oral secretions suctioned during last hours d- Obtain capillary blood samples q2 hours for glucose monitoring. Rationale: With a known head injury, sudden inadequate secretion of antidiuretic hormone (ADH) can cause excessive output of diluted urine. Evaluating laboratory results should de determined to identify findings of neurogenic diabetes insipidus (DI0, such as low urine osmolarity and normal serum osmolarity (A) prior to notify the healthcare provider so that these finding can be included in the report. Massive diuresis, dehydration, and thirst manifest hypotension, irregular tachycardia, decrease skin turgor, but B or C are not related to DI. 509- A female client is taking alendronate, a bisphosphate, for postmenopausal osteoporosis. The client tells the nurse that she is experiencing jaw pain. How should the nurse respond? a- Determine how the client is administering the medication b- Confirm that this is a common symptom of osteoporosis c- Report the client’s jaw pain to the healthcare provider. d- Advise the client to gargle with warm salt water twice daily. Rationale: Bisphosponates, including alendronate, can cause osteonecrosis of jaw, which should be reported to the healthcare provider © for evaluation. Incorrect administration (A) such as failing to remain upright after taking the medication, can contribute to esophageal reactions, but does not causes haw pain. Jaw pain is not a symptom of osteoporosis and is not relieved with saline throat gargles. 510- A male client has received a prescription for orlistat for weight and nutrition management. In addition to the medication, the client states he plans to take a multivitamin. What teaching should the nurse provide? a- As a nutritional supplement, orlistat already contains all the recommended daily vitamins and minerals. b- Multivitamins are contraindicated. During treatment with weight-control medications such as orlistat c- Be sure to take the multivitamin and the medication at least two hours apart for best absorption and effectiveness. d- Following a well-balanced diet is a much healthier approach to good nutrition than depending on a multivitamin. Rationale: Orlistat decreases intestinal absorption of fats, including fat soluble vitamins. Therefore it is important to take a multivitamin fat soluble vitamin at least two hours before of after the orlistat so that the vitamins are absorbed. 511- Which intervention should the nurse implement for a client with a superficial (first degree) burn? a- Spray an anesthetic agent over the burn every 3 to 4 hours b- Position the burn victim in front of a cool fan to decrease discomfort c- Apply ice pack for 30 mints to lower surface temperature d- Place wet clothes on the burned areas for short periods of time. Rationale: D provides comfort and helps to relive the pain of a first degree burn, which involves only the epidermal layer of the skin. 512- What is the primary goal when planning nursing care for a client with degenerative joint disease (DJD)? a- Obtain adequate rest and sleep b- Achieve satisfactory pain control. c- Improve stress management skills d- Reduce risk for infection. Rationale: The primary manifestation of DJD is pain. Although fatigue is not a major manifestation of DJD, pain control may improve 513- An adult woman who is seen in the clinic with possible neuropathic pain of the right leg rates her pain as a 7 on a 10-point scale. What action should the nurse take? a- Elevate the foot and leg on two pillows b- Measure the client’s capillary glucose c- Ask the client to dorsiflex the right foot. d- Encourage the client to describe the pain. Rationale: Neuropathic pain is caused by damage within the nervous system. Description of the pain such as burning or numbness helps identify the pain as neuropathic, allowing appropriate treatment to be initiated. Elevation is to unlikely to impact the pain. Persons with diabetes mellitus may develop peripheral neuropathy, nut there is no immediate need to measure this client’s capillary glucose. (C) is not a useful intervention in assessing or managing neuropathic pain. 514- A client has both primary IV infusion and a secondary infusion of medication. An infusion pump is not available. The nurse needs to determine the current rate of infusion of the primary IV. Where should the nurse observe to determine the rate of infusion? e- 515- The nurse is conducting the initial assessment of an ill client who is from another culture…. What response should the nurse provide? a- Can you read the written instructions is English? b- “What practices do you believe will help you heal?” c- What prescriptions must be strictly followed to get well. d- You must believe that the medications will help you. Rationale: The most culturally- sensitive interaction is to ask the client specifically. It is most important to identify the client’s cultural… and cultural beliefs of the client. 516- The nurse is caring a client with NG tube. Which task can the nurse delegate to the UAP? a- Replace the NG tube as prescribed by the healthcare provider b- Secure the NG tube if it slides out of the client’s nasal passage c- Disconnect the NG suction so the client can ambulate in the hallway. d- Reconnect the NG suction when the client returns form ambulating. Rationale: Disconnect the NG tube is within the scope of practice for the UAP. A, B and D No. 517- The nurse is collecting a sterile urine specimen using a straight catheter tray for culture…. (Arrange from first action to last). 1. Drape the client in a recumbent position for privacy 2. Open the urinary catheterization tray 3. Don sterile gloves using aseptic technique 4. Use forceps and swaps to clean the urinary meatus Rationale: To prevent the risk of infection, urinary catheterization is a sterile technique. And the privacy for the client is first. 518- The nurse is caring for a toddler with a severe birth anomaly that is dying. The parents have… holding the child as death approaches. Which intervention is most important for the nurse? a- Notify nursing supervisor and hospital chaplain of the child’s impending death. b- Verify that the no resuscitate forms are in the child’s medical record c- Ask the parents if they have made arrangements with a funeral home d- Provide staff coverage to sit with them as the child’s death approaches. Rationale: The impact of a child’s death even if expected, can be devastating, so the nurse.. that the child’s death is imminent the priority of emotional support by the chaplain and implementation of post-mort… are in place. 519- The nurse is assessing a 4-year-old boy admitted to the hospital with the diagnosis of possible nephrotic syndrome. Which statement by the parents indicates a likely correlation to the child’s diagnosis? e- “I couldn’t get my son’s socks and shoes on this morning” f- My son has been on amoxicillin/clavulanate for 2 days for an ear infection g- My son has had a red rash over his entire body for the past 4 days. h- I couldn’t get my son calm down and sleep last night. Rationale: Edema is a classic manifestation of the nephrotic syndrome and parents may first notice clothes, dia… B is for streptococcal infections that cause upper respiratory infections. 544- Which interventions should the nurse include in a long-term plan of care for a client with COPD? a- Reduce risk factors for infection b- Administer high flow oxygen during sleep c- Limit fluid intake to reduce secretions d- Use diaphragmatic breathing to achieve better exhalation Rationale: Interventions aimed at reducing the risk factors of infections should be included in the plan of care COPD client are at particular risk for respiratory infection. Prevention and early detection of infections are necessary 545. A health care provider continuously dismisses the nursing care suggestions made by staff nurses. As a result…dealing with the healthcare provider. What action should the nurse-manager implement? f- Confront the health care provider about the perceived lack of respect for the staff nurses. g- Plan an interdisciplinary staff meeting to develop strategies to enhance client care h- Request an investigation about the perceived incivility of the healthcare provider interaction. i- Remind the staff that avoidance behavior is not a professional way to handle the problem. 546. A 2-year-old girl is brought to the clinic for a routine assessment and all findings are within the normal limits. However, the mom expresses concern over her daughter’s protruding abdomen and tells the nurse that she is worry that her child is becoming overweight. How should the nurse respond to the mother’s comment? a- Tell the mother to keep a twenty- four-hour food diary for the child. b- Explain that a protruding abdomen is typical for toddlers. c- Discuss way to increase the child’s daily activity level d- Ask the mother is she has weight problems when she was a child. 547. A client admitted with an acute coronary syndrome (ACS) receives eptifibatide, a glycoprotein (GP) IIB IIIA inhibitor, which important finding places the client at greatest risk? a. Blood pressure of 100/60 b. Incontinent with blood in urine c. Unresponsive to painful stimuli d. Presence of hematemesis. Rationale: Eptifibatide, is an inhibitor of platelet aggregation, is administer IV for ACS, and bleeding is a significant side effect. A sudden onset of unresponsiveness may indicate intracranial bleeding, which is the life threatening finding related to bleeding. Although hypotension may indicate bleeding, it is not as significant as unresponsiveness to pain. This medication has a short half-life, so B and D are not life threatening findings. 548. In asseA toddler presents to the clinic with a barking cough, strider, refractions with respiration, the child's skin is pink with capillary refill of 2 seconds. Which intervention should the nurse implement? a. Encourage the child to cough b. Obtain a throat specimen for culture c. Administer nebulized epinephrine d. Collect blood for arterial blood gasses Rationale: The child’s airway is closing, and nebulized epinephrine C should be given promptly to cause mucosal vaso… decrease subglottic edema, which helps to maintain the child’s airway. Encouraging the child to cough (A)… edema. B can precipitate laryngeal spasm and is contraindicated. The priority … respiratory exchange and oxygenation, which is more commonly monitored using pulse oximetry, instead D. 549. The nurse caring for a client with dysphagia is attempting to insert an NG tube, but the client will not swallow and is not gagging. What action should the nurse implement to facilitate the NGT passage into the esophagus? a. Push the NGT beyond the oropharynx gently yet swiftly. b. Offer the client sips of water or ice and coax to swallow c. Elevate the bed 90 degree and hyperextend the head. d. Flex the client’s head with chin to the chest and insert. Rationale: The position that allow the epiglottis to close during the swallow reflex is best achieved with the head flex to the chest (D). B could potentially be aspirated when…. C the epiglottis will open when the head is hyperextended and allow the NGT to enter the airway. 550. The nurse plans to use an electronic digital scale to weight a client who is able to stand. Which intervention should the nurse implement to ensure that measurement of the client’s weight is accurate? a. Ask the client to remove shoes before stepping on the scale b. Ensure that the scale is calibrated before a weight is obtained. c. Slide the balancing weights until the scale is at zero. d. Compare client’s weight at various time of the day. Rationale: Electronic digital scales should be calibrated before obtaining weight. Removing shoes help to ensure correct…client’s body weight is being measured but does not ensure the scale’s accuracy. The client should be weighed at the same time of day and comparing weights…. 551. The nurse observes a newly hired unlicensed assistive personnel (UAP) performing a fingestick to obtain a client’s blood glucose. Prior to sticking the client’s finger, the UAP explains the procedure and tell the client that it is painless. What action should the nurse take? a. Allow the UAP to complete the procedure, then discuss the painless comment privately with the UAP. b. Stop the UAP before the procedure and explain to the client that some discomfort may be felt c. Interject that while the procedure is not extremely painful, the client will feel a prick on the finger. d. Report the incident to the education director and request additional instruction for the UAP. Rationale: Communicating what clients can expect with regard to their care is their right and such actions reinforced… should be reminded to be truthful and tell the client that the fingerstick involves a small prick of the finger… care worker in front of the client 9Band C) decrease client trust. The issue can easily be addressed in a timely manner by the nurse. 552. An African-American man come into the hypertension screening booth at a community fair. The nurse finds that is blood pressure is 170/94 mmHg. The client tells the nurse that he has never been treated for high blood pressure. What response should the nurse make? a. Your blood pressure indicate that you have hypertension. You need to see a physician at once. b. Your blood pressure is quite high. Go to the closest emergency room for immediate treatment. c. Your blood pressure is a little high. You need to have it rechecked within one week. d. Your blood pressure is little high, but it is within the normal range for your age group. 553. While attempting to stablish risk reduction strategies in a community, the nurse notes that the regional studies have indicated….persons with irreversible mental deficiencies due to hypothyroidism. The nurse should seek funding to implement which screening measure? a. T4 levels in newborns b. TSH level on women over 45 c. T3 level in school- aged children d. Iodine levels in all persons over 60. 554. After applying an alcohol-based hand rub to the palms of the hand and rubbing the hand together, what action should the nurse do next? a. Vigorous rub both hands together under running water b. Path both hands dry keeping the fingers lower that the arm c. Place one hand on top of the other and interlace the fingers d. Hold both hand with the fingers pointing upward until dry. Rationale: evaluate fter apply alcohol put the hands together, the palm of one hand should be place over the dorsum of the other… promote further cleansing of additional surfaces of the hand. 555. A nurse is preparing to feed a 2-month-old male infant with heart failure who was born with congenital heart defect. Which intervention should the nurse implement? a. Feed the infant when he cries b. Allow the infant to rest before feeding c. Weigh before and after feeding. d. Insert a nasogastric feeding tube. 556. While removing an IV infusion from the hand of a client who has AIDS, the nurse is struck with the needle. After washing the puncture site with soap & water, which action should the nurse take? a. Complete a usual incident report b. Start prophylactic treatment c. Seek psychological resources d. Notify the employee health nurse. 557. A nurse receives a shift report about a male client with Obsessive compulsive disorder (OCD). The nurse does morning rounds and reaches the client while he is repeatedly washing the top of the same table. What intervention should the nurse implement? a. Encourage the client to be calm and relax for a little while b. Assist the client to identify stimuli that precipitates the activity. c. Allow time for the behavior and then redirect the clients to other activities. d. Teach the client thought stopping techniques and ways to refocus. 558. The nurse is caring for a client immediately after inserting a PICC line. Suddenly, the client becomes anxious and tachycardiac, and loud churning is heard over the pericardium upon auscultation. What action should the nurse take first? a. Place client in Trendelenburg position on the left side. b. Administer precordial thump c. Monitor the client with a 12-lead electrocardiogram d. Request a STAT portable chest x-ray. 559. A client admitted to the telemetry unit is having unrelieved chest pain after receiving 3 sublingual nitroglycerin tablets and morphine 8 mg IV. The electrocardiogram reveals sinus bradycardia with ST elevation. In what order should the nurse implement the nursing actions? (Arrange first to last) 1. Call the rapid response team to assist 2. Move the crash cart to the client room 3. Notify the client’s healthcare provider 4. Inform the family of the critical situation 560. The nurse is preparing dose # 7 of an IV piggyback infusion of tobramycin for a 73-year-old client with... Infected pseudomonas aeruginosa. Which assessment data warrants further intervention by the nurse? a- Peak and through levels has not been drawn since the tobramycin was started b- Today labs report indicates a white blood cell count of 13,000 cell/mm3 or 13 x 10777/L (S1) c- A serum creatinine level of 1.0 mg/dl or 88 mcmol/L (S1) is documented on yesterday flowsheet. d- The culture growth form the burn areas is sensitive to aminoglycosides. Rationale: Serum peak and trough levels should be drawn periodically during IV aminoglycoside therapy to… tobramycin level is at the therapeutic or toxic range, which can cause nephrotoxicity and ototoxicity. B is expected because there is an infection. C is within normal values. Nebcin or Tobramycin is the desired antibiotic for the culture. 561. A client with HIV and pulmonary coccidioidomycosis is receiving amphotericin B. which assessment finding should the nurse report to the healthcare provider? a- Urinary output of 25mL per hour b- Hemoglobin level of 10 g/dL or 100 g/L (S1) c- Hyperactive bowel sounds d- Oral temperature of 100.4 F (38 C) Rationale: A major adverse effect of amphotericin B, and antifungal, is nephrotoxicity, so the nurse should …throughout therapy, and report decreased urinary output, the client’s hemoglobin is low... fatigue and possible bleeding. C is related to diarrhea and nephrotoxicity is more important. D is an expected finding since the client clinically presents... with this fungal lung infection. (Coccidioides Immitis) 562. A client delivers a viable infant, but begins to have excessive uncontrolled vaginal (bleeding)…notifying the health care provider of the clients’ condition, what information is most…. a. Maternal blood pressure b. Maternal apical pulse rate c. Time Pitocin infusion completed d. Total amount of Pitocin infused Rationale: Maternal blood pressure provides information about the client’s current hemodynamic (status)… healthcare provider’s decision about which drug to use to stop the bleeding. B C and D …. Healthcare provider to have, but they do not have the priority of A. 563. An infant born to a heroin-addicted mother is admitted to the neonatal care unit. What behaviors can the newborn…to exhibit? a. Lethargy and poor suck b. Facial abnormalities and microcephaly c. Irritability and a high-pitched cry d. Low birth weight and intrauterine growth retardation Rationale: Infant of heroin addicts will suffer from neonatal narcotic withdrawal syndrome which is characterized by… hyperactivity, high-pitched cry, coarse, flapping tremors, poor feeding, frantic sucking, vomiting/diarrhea… 564. A multigravida, full-term, laboring client complains of “back labor”. Vaginal examination reveals that the client’s 3 cm with 50% effacement and the fetal head is at -1 station. What should the nurse implement? a. Turn the client to a lateral position b. Apply counter-pressure to the sacral area c. Notify the scrub nurse to prepare the OR d. Ambulate the client between contractions Rationale: B provides pain relief during labor. 565. A client with gestational diabetes is undergoing a non-stress test (NST) at 34-week gestation… is 144 beats/minute. The client is instructed to mark the fetal monitor by pressing a button each time the baby moves. After 20 minutes, the nurse evaluates the fetal monitor strip. What… a- The mother perceives and marks at least four fetal movements b- Fetal movements must be elicited with vibroacoustic stimulator c- Two FHR accelerations of 15 beats/minute x 15 seconds are recorded. d- No FHR late deceleration occur in response to fetal movement 566. A male infant born at 28-weeks gestation at an outlying hospital is being prepared for transport to a respiration are 92 breaths/minute and his heart rate is 156 beats/minute. Which drug is the transport administration to this infant? e- Give ampicillin 25 mg/kg slow IV push f- Deliver 1:10,000 epinephrine 0.1 ml/kg per endotracheal tube g- Administer digoxin 20 mcg/kg IV h- Instill beractant 100 mg/kg in endotracheal tube. Rationale: RDS (Respiratory Distress Syndrome) results primarily from immature lungs and the lack of surfactant… mixture that coats the alveoli and prevents their collapse at the endo of respiration. Beractant (D) be given per endotracheal tube 15 minutes to 8 hours after birth A is indicated if sepsis is suspected… this premature infant. Although B is indicated in cardiac arrest, the risk of respiratory distress… rate is within normal limits, so C is not needed. 567. A postpartal client complains that she has the urge to urinate every hour but is only able to void a small amount. What interventions provides the nurse with the most useful information? a. Initiate a perineal pad count b. Catheterize for residual urine after next voiding c. Assess for a perineal hematoma d. Determine the client’s usual voiding pattern Rationale: Voiding small amounts frequently is a sign of urinary retention that can lead to urinary tract infections… urine after voiding assesses for adequate emptying of the client’s bladder. 568. During a 26-week gestation prenatal exam, a client reports occasional dizziness…What intervention is best for the nurse to recommend to this client? a- Elevate the head with two pillows while sleeping b- Lie on the left or right side when sleeping or resting. c- Increase intake of foods that are high in iron d- Decrease the amount of carbohydrates in the diet. Rationale: Lying on the left or right side will reduce the compression of the vena cava… experiencing supine hypotension resulting from vena cava compression that is… 569. Artificial rupture of the membrane of a laboring reveals meconium-stained fluid, what is… the priority? a- Clean the perineal are to prevent infection b- Assess the mother’s blood pressure to check for signs of preeclampsia c- Assess the mother temperature to check for development of sepsis. d- Have a meconium aspirator available at delivery. Rationale: During delivery, meconium can plug the alveolar sac and obstruct the infant’s bronchi…. 570. A 6-year-old child with acute infectious diarrhea is placed on a rehydration therapy…Which action should the nurse instruct the parents to take if the child begins to vomit? a. Continue giving ORS frequently in small amounts b. Withhold all oral intake c. Supplement ORS with gelatin or chicken broth d. Provide only bottle water. Rationale: Vomiting is not a contraindication for oral rehydration therapy, as vomiting…. Be continue. 571. A client is receiving oxytocin (Pitocin) to augment early labor. Which assessment is most important time the infusion rate is increases? a. Contraction pattern b. Blood pressure c. Infusion site d. Pain level Rationale: 572. An infant is placed in a radiant warmer immediately after birth. At one hour of age, the nurse finds the infant tachypneic, and hypotonic. What is the first action that the nurse should take? e- Notify the healthcare provider immediately f- Increase the temperature of the radiant warmer g- Assess the infant’s heart rate. h- Determine the infant’s blood sugar level. Rationale: The nurse should validate the etiology for hypoglycemia by obtaining a blood glucose level. Hypoglycemia in newborns are jitteriness, tachypnea and hypotonia. A is an overreaction. Hypothermia is related to development of hypoglycemia, however, frit the nurse should… experiencing hypoglycemia before implementing A. B is contraindicated 573. A toddler with a history of an acyanotic heart defect is admitted to the pediatric intensive…rate of 60 breaths/ minute, and a heart rate of 150 beats/minute. What action should the nurse take? a. Obtain a pulse oximeter reading b. Assess the child blood pressure c. Perform a neurological assessment d. Initiate peripheral intravenous access. Rationale: Based on the priority of ABC’s a pulse oximeter reading… 574. In preparing a diabetes education program, which goal should the nurse identify as the primary emphasis for a class on diabetes self-management? i- Prepare the client to independently treat their disease process j- Reduce healthcare costs related to diabetic complication. k- Enable clients to become active participants in controlling the disease process. l- Increase client’s knowledge of the diabetic disease process and treatment option. Rationale: The primary goal of diabetic self-management education is to enable the client to become an active participant in the care and control of the disease process, matching levels of self-management to the abilities of the individual client. 575. To obtain an estimate of a client’s systolic B/P. What action should the nurse take first? a. Palpate the client’s brachial pulse b. Pump up the blood pressure cuff c. Position the stethoscope diaphragm d. Release the blood pressure cuff valve 576. A client is admitted to isolation with the diagnosis of active tuberculosis (TB). Which infection control measures should the nurse implement? a. Negative pressure environment b. Contact precautions c. Droplet precautions d. Protective environment 577. A client currently receiving an infusion labeled Heparin Sodium 25,000 Units in 5% Dextrose Injection 500 mL at 14 mL/hour … verify that the client is receiving the prescribed amount of heparin. How many unit is the… (enter numeric value only) 700 D/H *Q . A prescription is received to change the rate of the infusion to 900 units of Heparin per hour. The nurse should set the infusion pump to deliver how many mL/hour? (Enter numeric value only). 700 578. The nurse notes the client receiving heparin infusion labeled, Heparin Na 25,000 Units in 5% Dextrose injection 500 ml at 50ml/hr. What dose of Heparin is the client receiving per hour? a. 2,500 579. A male client is returned to the surgical unit following a left nephrectomy and is medicated with morphine. His dressing has a small amount of bloody drainage, and a Jackson-Pratt bulb surgical drainage device is in place. Which interventions is most important for the nurse to include in this clients plan of care? i- Monitor urine output hourly. j- Assess for back muscle aches k- Record drainage from drain l- Obtain body weight daily Rationale: When one kidney is removed the remained kidney must do all the volume filtering, so A is immediate to postoperative period. 580. The family of a client who just died arrives on the nursing unit after receiving telephone notification of the death. Several family members state they would like to view the body. How should the nurse respond? b. Offer to go with the family members to view the body. 581. The nurse is assessing a first day postpartum client. Which finding is most indicative of a postpartum infection? a. White blood count of 19,000 mm3 b. Oral temperature of 100.2 F (37.9 C) c. Moderate amount of foul-smelling lochia. d. Blood pressure 122/74 mm Hg Rationale: C is indicative of endometritis. B is a variant but not as indicative as C 582. An older woman who has difficulty hearing is being discharged from day surgery following a cataract extraction & lens implantation. Which intervention is most important for the nurse to implement to ensure the client's compliance with self-care? a- Speak clearly and face the clients for lip reading b- Provide written instructions for eyes drop administration c- Ensure that someone will stay with the client for 24 hours. d- Have the client vocalize the instructions provided. Rationale: A client with both hearing and visual sensory deficit should be repeat the instruction provided so the nurse needs to be sure the clients understand the self-care instructions. 583. A primigravida client is 36 weeks’ gestation is admitted to labor and delivery unit because her membranes ruptured 30minutes ago. Initial assessment indicates 2cm dilation, 50% effaced, -2 station, vertex presentation greenish colored amniotic fluid, and contractions occurring 3-5 minutes with a low FHR after the last contraction peaks: a. Administer Oxygen via face mask b. Apply an internal fetal heart monitor c. Notify the healthcare provider d. Use a vibroacoustic stimulator Rationale: The nurse should administer oxygen to increase the amount of oxygen available for the fetus, because is presenting characteristics of late decelerations, caused by uteroplacental insufficiency. 584. A woman just received the Rubella vaccine after a delivery of a normal new born, has two children at home, ages 13 months and 3 years. Which instruction is most important to provide to the client? a- Refrain from eating foods containing eggs for 24 hors b- Breast feeding is recommended to prevent ovulation. c- Do not get pregnant for at least 3 months. d- Avoid exposure to the sunlight for 36 hours. Rationale: The rubella vaccine can be harmful to an unborn child who is conceived within 3 month of the vaccination. 585. Following a motor vehicle collision (MCV), a male adult in severe pain is brought to the emergency department via ambulance. His injured left leg is edematous, ecchymotic around the impact of injury on the thigh, and shorter than his right leg. Based on these findings, the client is at greatest risk for which complication? a. Arterial ischemia b. Tissue necrosis c. Fat embolism d. Nerve damage 586. A 2-year-old is bleeding from a laceration on the right lower extremity that occurred as the result of a motor vehicle collision. The nurse is selecting supplies to start an IV access. Which assessment finding is most significant in the nurse's selection of catheter size? a. Thready brachial pulse. b. Respirations of 24/minute c. Right foot cool to touch. d. Swelling at the site of injury Rationale: When selecting intravenous catheter size, the nurse should consider… victim who is a 2-years-old with a trhready brachial pulse... 587. The nurse prepares to insert an oral airway by first measuring for the correct sized airway. Which picture shows the correct approach to airway size measurement? e. Rationale: From the corner of the mouth to the earlobe 588. A client with a recent colostomy expresses concern about the ability to control flatus. Which intervention is most important for the nurse to include in the client’s plan of care? a. Adhere to a bland diet whenever planning to eat out b. Decrease fluid intake at meal times c. Avoid foods that caused gas before the colostomy d. Eliminate foods high in cellulose 589. A male client arrives at the clinic with a severe sunburn and explains that he did not use sun screen because it was an overcast day. Large blisters are noted over his back and chest and his shirt is soaked with serosanguinous fluid. Which assessment finding warrants immediate intervention by the nurse? a. Hypotension. b. Fever and chills c. Dizziness d. Headache 590. A client with polycystic kidney disease (PKD) receiving antibiotics for an infected cyst is experiencing severe pain. What action should the nurse implement? a. Hold the next dose of antibiotic until contacting the healthcare provider b. Teach the client how to use a dry heating pad over the painful area c. Encourage the client to practice pelvic floor exercises every hour d. Assist the client to splint the site by applying an abdominal binder Rationale: Infected cyst ca be very painful. The nurse should administer prescribed analgesic and management techniques such as dry heat. A does not reduce the pain and slow treatment of the infection. C may cause increased pain. 591. Which statement is accurate regarding the pathological changes in the pulmonary system associated with acute (adult) respiratory distress syndrome (ARDS)? a. Capillary hydrostatic pressure exceeds colloid osmotic pressure, producing interstitial edema b. A high ventilation-to-perfusion ratio is characteristic of affected lung fields in ARDS c. Functional residual capacity and lung compliance increase as the disease progresses d. Interstitial edema that occurs due to capillary fluid shifts is usually more serious than alveolar edema. Rationale: Interstitial edema occurs when hydrostatic pressure exceeds osmotic pressure in the… distress syndrome (ARDS) result in increased permeability of the pulmonary vascular…pulmonary edema. 592. The nurse mixes 250 mg of dobutamine in 250 ml of D5W and plans to administer the solution at rate client weighing 110 pounds. The nurse should set the infusion pump to administer how many ml per hour only. If rounding is required, round the nearest whole number.) a. 45 110 lbs./2.2 (kg in one lb.) =50kg 50 kg x 125 mcg/kg/min x 1 mg/1000 mcg x 250 mg/250 ml x 60 mints/hr. = 45 ml/hr 593. During the intraoperative phase of care, the circulating nurse observes that the client is not adequately… client's privacy. What is the best initial nursing action for the nurse to implement? a- Document the observation in the client’s medical record b- Instruct the scrub nurse to re-drape the client c- Ensure that the client in unaware of the surrounding. d- Consult with operating room manager. Rationale: The circulating nurse must act as a client advocate, which includes ensuring respect for privacy. The scrub nurse should be instructed to complete whatever redraping is necessary. An and D are not necessary. Because the nurse’s roll to ensure privacy C is incorrect. 594- An adult male who was admitted two days ago following a cerebrovascular accident (CVA) is confused and experiencing left-side weakness. He has tried to get out of bed several times, but is unable to ambulate without assistance. Which intervention is most important for the nurse to implement? a- Ask a family member to sit with the client b- Apply bilateral soft wrist restraints c- Assign staff to check client q15 minutes d- Install a bed exit safety monitoring device Rational: the bed exit alarm alerts the nursing staff that the client is shifting weight toward…. Restrain should be the last resource. 595. A client in her first trimester of pregnancy complains of nausea. Which complementary therapy should the nurse recommend? a. Eat food high in garlic with the evening meal b. Drink chamomile tea at breakfast and in the evening. c. Increase cocoa in the diet and drink before bedtime d. Join a yoga class that meets at least weekly Rationale: Chamomile tea is used to aid with digestion and is in fact sometimes used for indigestion. C should not be used by breastfeeding woman or at night when trying to go to sleep. D is for improve circulation, stimulate the internal organs, stretch the body, restore…. 596. When gathering for a group therapy session at 1400 hours, a female client complains to the nurse that a smoking break has not been allowed all day. The nurse responds that 15 minute breaks were called over the unit intercom after breakfast and after lunch. The nurse is using what communication technique in responding to the client? a. Doubt b. Observation c. Confrontation d. Reflection 597. A female client with rheumatoid arthritis (RA) comes to the clinic complaining of joint pain and swelling. The client has been taking prednisone (Deltasone) and ibuprofen (Motrin Extra Strength) every day. To assist the client with self-management of her pain, which information should the nurse obtain? a. Presence of bruising, weakness, or fatigue b. Therapeutic exercise included in daily routine. c. Average amount of protein eaten daily d. Existence of gastrointestinal discomfort 598. The charge nurse of the Intensive Care Unit is making assignments for the permanent staff and one RN who was floated from a medical unit. The client with which condition is the best to assign to the float nurse? a. Diabetic ketoacidosis and titrated IV insulin infusion b. Emphysema extubated 3 hours ago receiving heated mist c. Subdural hematoma with an intracranial monitoring device d. Acute coronary syndrome treated with vasopressors 599. A client admitted to the emergency center had inspiratory and expiratory wheezing, nasal flaring, and thick, tenacious sputum secretions observed during the physical examination. Based on these assessment findings, what classification of pharmacologic agents should the nurse anticipate administering? a. Beta blockers b. Bronchodilators c. Corticosteroids d. Beta-adrenergic 600. The home health nurse is assessing a male client who has started peritoneal dialysis (PD) 5 days ago. Which assessment finding warrants immediate intervention by the nurse? a. Finger stick blood glucose 120 mg/dL post exchange b. Arteriovenous (AV) graft surgical site pulsations. c. Anorexia and poor intake of adequate dietary protein d. Cloudy dialysate output and rebound abdominal pain 601. A client’s telemetry monitor indicates ventricular fibrillation (VF). What should the nurse do first? a. Administer epinephrine IV b. Give an IV bolus of amiodarone c. Provide immediate defibrillation d. Prepare for synchronized cardioversion 602- In conducting a health assessment, the nurse determines that both parents of a child with asthma smoke cigarettes. What recommendation is best to the nurse to recommend to the parents? a- avoid smoking in the house b- stop smoking immediately c- decrease the number of cigarettes smoke daily d- obtain nicotine patches to assist in smoking sensation 603- A client who is schedule for an elective inguinal hernia repair today in day surgery is seem eating in the waiting area. What action should be taken by the nurse who is preparing to administer the preoperative medications? a- Review the surgical consent with the client b- Explain that vomiting can occur during surgery c- Remove the food from the client d- Withhold the preoperative medication 604- The nurse is developing a plan of care for a middle-aged woman who is diagnosed with type 2 diabetes mellitus (DM). To lower her blood glucose and increase her serum high-density lipoprotein (HDL) levels, which instruction is most important for the nurse to provide? a- Exercise at least three times weekly b- Monitor blood glucose levels daily c- Limit intake of foods high in saturated fat d- Learn to read all food product labels 605- A client who has been in active labor for 12 hours suddenly tells the nurse that she has a strong urge to have a bowel movement. What action should the nurse take? a- Allow the client to use a bedpan. b- Assist the client to the bathroom c- Perform a sterile vaginal exam d- Explain the fetal head is descending. Rationale: When a client in active labor suddenly expresses the urge to have a bowel movement, a sterile vaginal exam should be performed to determine if the fetus is descending. 606- The nurse assesses a 78-year-old male client who has left sides heart failure. Which symptoms would the nurse expect this client to exhibit? a- Dyspnea, cough, and fatigue. b- Hepatomegaly and distended neck veins c- Pain over the pericardium and friction rub. d- Narrowing pulse pressure and distant heart sounds. 607- A female client comes to the clinic complaining of fatigue and inability to sleep because she is the full-time caretaker for 22-year-old son who was paralyzed by a motor vehicle collision. She adds that her husband left her because he says he can’t take her behavior any more since all she does is care for their son. What intervention should the nurse implement? a- Schedule a home visit in the afternoon to assess the son and client role as caregiver. b- Acknowledge the client’s stress and suggest that she consider respite care. c- Provide feedback to the client about her atonement for guilt about her son’s impairment. d- Teach the client to problem-solve for herself and establish her own priorities. Rationale: When this amount of disclosure is offered, the client is usually seeking information focuses on the client’s expression of worry, concern and stress and addresses the client’s need to initiate a request for assistance with respite care. 608- The nurse plans to administer a schedule dose of metoprolol (Toprol SR) at 0900 to a client with hypertension. At 0800, the nurse notes that client’s telemetry pattern shows a second-degree heart block with a ventricular rate of 50. What action should the nurse take? a- Administer the Tropol immediately and monitor the client until the heart rate increases. b- Provide the dose of Tropol as scheduled and assign a UAP to monitor the client’s BP q30 minutes. c- Give the Tropol as scheduled if the client’s systolic blood pressure reading is greater than 180. d- Hold the scheduled dose of Tropol and notify the healthcare provider of the telemetry pattern. Rationale: Beta blockers such as metoprolol (Tropol SR) are contraindicated in clients with second or third-degree heart block because they decrease the heart rate. Therefore, the nurse should hold the medication. 609- A client who developed syndrome of inappropriate antidiuretic hormone (SIADH) associated with small carcinoma of the lung is preparing for discharge. When teaching the client about self-management with demeclocycline (Declomycin), the nurse should instruct the client to report which condition to the health care provider? a- Insomnia b- Muscle cramping c- Increase appetite d- Anxiety. Rationale: SIADH causes dilution hyponatremia because of the increased release of ADH, which is treated with water restriction and demeclocycline, a tetracycline derivate that blocks the action of ADH. Signs of hyponatremia (normal 136-145), which indicate the need for increasing the dosage of demeclocycline, should be reported to the healthcare provider. The signs include: plasma sodium level less than 120, anorexia, nausea, weight changes related to fluid disturbance, headache, weakness, fatigue, and muscle cramping. AC& D are not related to hyponatremia. 610- In determine the client position for insertion of an indwelling urinary catheter, it is most important for the nurse to recognize which client condition? a- High urinary PH b- Abdominal Ascites c- Orthopnea d- Fever. Rationale: If the client is orthopneic, the nurse needs to adapt the insertion position that does not place the client in a supine position (the head of the bed should be elevated as much as possible). 611- The nurse is reviewing a client’s electrocardiogram and determines the PR interval (PRI) is prolonged. What does this finding indicate? a- Initiation of the impulses from a location outside the SA node b- Inability of the SA node to initiate an impulse at the normal rate c- Increased conduction time from the SA node to the AV junction d- Interference with the conduction through one or both ventricles. Rationale: A prolonged PRI reflects an increased amount of time for an impulse to travel from the SA node through the AV node and is characteristic of a first-degree heart block. 612- The nurse is teaching a male client with multiple sclerosis how to empty his bladder using the Crede Method. When performing a return demonstration, the client applies pressure to the umbilical areas of his abdomen. What instruction should the nurse provide? a- Stroke the inner thigh below the perineum to initiate urinary flow b- Contract, hold, and then relax the pubococcygeal muscle c- Pour warm water over the external sphincter at the distal glans d- Apply downward manual pressure at the suprapubic regions. Rationale: The Crede Method is used for those clients with atonic bladders, which is a concomitant of demyelinating disorders like multiple sclerosis. The client is applying pressure in the wrong region (umbilical Are) and should be instructed to apply pressure at the suprapubic are. 613- A 35 years old female client has just been admitted to the post anesthesia recovery unit following a partial thyroidectomy. Which statement reflects the nurse’s accurate understanding of the expected outcome for the client following this surgery? a- Supplemental hormonal therapy will probably be unnecessary b- The thyroid will regenerate to a normal size within a few years. c- The client will be restricted from eating seafood d- The remainder of the thyroid will be removed at a later date. 614- A client with gestational diabetes, at 39 weeks of gestation, is in the second stage of labor. After delivering of the fetal head, the nurse recognizes that shoulder dystocia is occurring. What intervention should the nurse implement first? a- Prepare the client for an emergency cesarean birth b- Encourage the client to move to a hands-and-knees position. c- Assist the client to sharply flex her thighs up again the abdomen. d- Lower the head of the bed an apply suprapubic pressure. Rationale: Flexing the client’s thighs against the abdomen (Mc Robert’s maneuver) changes the angle of the pelvis and increase the pelvic diameter, making more room for the shoulders to emerge. ABD are implemented after C 615- The nurse should observe most closely for drug toxicity when a client receives a medication that has which characteristic? a- Low bioavailability b- Rapid onset of action c- Short half life d- Narrow therapeutic index. Rationale: A drug with a narrow therapeutic index has a high risk for toxicity because there is a narrow range between the therapeutic dose and the toxic dose. 616- Following insertion of a LeVeen shunt in a client with cirrhosis of the liver, which assessment finding indicates to the nurse that the shunt is effective? a- Decrease abdominal girth b- Increased blood pressure c- Clear breath sounds d- Decrease serum albumin. 617- When finding a client sitting on the floor, the nurse calls for help from the unlicensed assistive personnel (UAP). Which task should the nurse ask the UAP to do? a- Check for any abrasions or bruises. b- Help the client to stand. c- Get a blood pressure cuff. d- Report the fall to the nurse-manager. 618- During the initial newborn assessment, the nurse finds that a newborn's heart rate is irregular. Which intervention should the nurse implement? a- Notify the pediatrician immediately. b- Teach the parents about congenital heart defects. c- Document the finding in the infant's record. d- Apply oxygen per nasal cannula at 3 L/min. 619- Which assessment finding indicates to the nurse a client’s readiness for pulmonary function tests? a- Expresses an understanding of the procedure. b- NPO for 6 hrs. c- No known drug allergies d- Intravenous access intact. 620- A young adult male is admitted to the emergency department with diabetic ketoacidosis (DKA). His pH is 7.25, HCO3 is 12 mEq/L or 12 mmol/L (SI), and blood glucose is 310 mg/dl or 17.2 mmol/L (SI). Which action should the nurse implement? a- Infuse sodium chloride 0.9% (normal saline) b- Prepare an emergency dose of glucagon c- Determine the last time the client ate d- Check urine for ketone bodies with a dipstick Rationale DKA an increase in glucose and ketone bodies, result in hyperosmolar dehydration, so is necessary to restore fluid balance. 621- The nurse is assessing the thorax and lungs of a client who is having respiratory difficulty. Which finding is most indicative of respiratory distress? a- Contractions of the sternocleidomastoid muscle. b- Respiratory rate of 20 breath/mints c- Downward movement of diaphragm with inspiration d- A pulse oximetry reading of SpO2 95% Rationale: Force inspiration needs to use accessories muscle and rib cage. 622- After receiving lactulose, a client with hepatic encephalopathy has several loose stools. What action should the nurse implement? a- Send stool specimen to the lab b- Measure abdominal girth c- Encourage increased fiber in diet. d- Monitor mental status. Rationale: Administer lactulose to a patient hepatic encephalopathy to lower serum ammonia level, so mental status should be improving. 623- A client presents at the clinic with blepharitis. What instructions should the nurse provide for home care? a- Use bilateral eyes patches while sleeping to prevent injury to eyes. b- Wear sunglasses when out of doors to prevent photophobia c- Apply warm moist compresses then gently scrub eyelids with dilute baby shampoo d- Apply warm moist compresses then gently scrub eyelids with dilute baby shampoo. Rationale: This condition is an inflammation of the eyelids edges that occurs in older adults. Is controlled with eyelid care using warm moist compresses followed by gently scrub eyelids. 624- Dopamine protocol is prescribed for a male client who weigh 198 pounds to maintain the mean arterial pressure (MAP) greater than 65 mmHg. His current MAP is 50 mmHg, so the nurse increases the infusion to 7 mcg/kg/minute. The infusion is labeled dextrose 5% in water (D5W) 500 ml with dopamine 400 mg. The nurse should program the infusion pump to deliver how many ml/hour? a- 47 Rationale: Calculate the client’s weigh in Kg 2.2 pounds: 1 Kg 198 pounds: x kgs = 90 Kgs 7mcg/kg/minute x90 Kg = 630 mcg/ minute x 60 mint (hour) =37,800 mcg/ hour Suing the formula, desired dose/available solution (D/H x Q) 3708 mg/hour divided by … 47 ml/hr. 625- The nurse is teaching a client with atrial fibrillation about a newly prescribed medication, dronedarone. Which information should the nurse include in client interactions? (Select all that apply) a- Discontinue medication when palpitation subside. b- Avoid eating grapefruit or drinking grapefruit juice. c- Report changes in the use of daily supplements d- Notify your health care provider if your skin looks yellow e- If a dose is missed, the next dose should be double. Rationale: Side effects can increase if the client consume grapefruit. OTC medications or herbal should be reported for possible drugs interactions. Hepatic injury can occur, and the client should report sign of jaundice or itching, or right upper quadrant pain. 626- A male client recently released from a correctional facility arrives at the clinic with a cough, fever, and chills. His history reveals active tuberculosis (TB) 10 years ago. What action should the nurse implement? (Select all that apply) a- Administer a PPD test b- Schedule the client for the chest radiograph c- Obtain sputum for acid fast bacillus (AFB) testing d- Place a mask on the client until he is moved to isolation. e- Send the client home with instructions for a prescribe antibiotic. Rationale: Client with history of TB a chest x-ray and sputum are indicated. The client sign and symptoms indicate the pt should wear mask to protect others. 627- A 16-year-old male is admitted to the pediatric intensive care unit after being involved in a house fire. He has full thickness burns to his lower torso and extremities. Before a dressing change to his legs, which intervention is most important for the nurse to implement? a- Encourage the parents to stay at the bedside b- Use distraction techniques to reduce pain. c- Maintain strict aseptic technique d- Place a drape over the pubic area. Rationale: Full thickness burns cause destruction of the epidermis, dermis… minimize post burn colonization of the wound with gram-negative opportunistic organism is necessary to maintain strict aseptic technique is essential during dressing changes. 628- While performing a skin inspection for a female adult client, the nurse observes a rash that is well circumscribed, has silvery scales and plaques, and is located on the elbows and knees. These assessment findings are likely to indicate which condition? a- Tinea corporis b- Herpes zoster c- Psoriasis d- Drug reaction Rationale: Psoriasis is typically located on the elbow and knees. 629- A client with acute pancreatitis is complaining of pain and nausea. Which interventions should the nurse implement (Select all that apply) a- Monitor heart, lung, and kidney function. b- Notify healthcare provider of serum amylase and lipase levels. c- Position client on abdomen to provide organ stability d- Encourage an increased intake of clear oral fluids e- Review client’s abdominal ultrasound findings. Rationale: Client’s heart, lungs and kidney can be damage by en…of pancreatitis, so it is important to monitor de function of these organs. High levels of lipase ad amylase are signs of pancreatic damage or inflammation. Ultrasound is the initial diagnostic test to determine the cause of pancreatitis. Positioning the abdomen may cause abdominal pressure and oral fluids should be withheld until nauseas subsides. 630- A nurse is caring for a client with Diabetes Insipidus. Which assessment finding warrants immediate intervention by the nurse? a- Hypernatremia b- Excessive thirst c- Elevated heart rate d- Poor skin turgor Rationale: A deficiency of antidiuretic hormone (ADH) is the underlying cause of DI, which results in… hypotonic urine. 631- In caring for a client receiving the amino glycoside antibiotic gentamicin, it is most important for the nurse to monitor which diagnostic test? a- Urinalysis b- Serum creatinine c- Serum osmolarity d- Liver enzymes. Rationale: Aminoglycosides can cause nephrotoxicity, so it is important for the nurse to monitor the serum creatinine level can monitor the renal function. 632- The nurse weighs a 6-month-old infant during a well-baby check-up and determines that the baby’s weight has tripled compared to the birth weight of 7 pounds 8 ounces. The mother asks if the baby is gaining enough weight. What response should the nurse offer? a- Your baby is gaining weight right on schedule b- What food does your baby usually eat in a normal day? c- The baby is below the normal percentile for weight gain d- What was the baby’s weight at the last well-baby clinic visit? Rationale: The normal weight gain in the first year of life is approx. twice the birth weight 633- A client who is at 36 weeks gestations is admitted with severe preeclampsia. After a 6-gram loading dose of magnesium sulfate is administered, an intravenous infusion of magnesium sulfate at a rate of 2 grams/hour is initiated. Which assessment finding warrants immediate intervention by the nurse? Urine output 20 ml/hour Rational: urinary output of less than 30 ml/hour indicates that the kidneys are being affected by the high level of magnesium, which is excreted through kidneys. 634- What is the nurse’s priority goal when providing care for a 2-year-old child experiencing seizure…? a- Stop the seizure activity b- Decrease the temperature c- Manage the airway d- Protect the body from injury Rationale: The highest priority is maintaining a patent airway. During seizure activity, this… and assessing for airway obstruction and apnea. A, B and D are also important, but C is priority. 635- The nurse is preparing to discharge an older adult female client who is at risk for hypocalcemia…nurse include with this client’s discharge teaching? a- Report any muscle twitching or seizures b- Take vitamin D with calcium daily c- Avoid seafood, particularly selfish d- Low fat yogurt is a good source of calcium e- Keep a diet record to monitor calcium intake Rationale: Twitching and seizure are signs of low calcium. (A) Vit D supplement with calcium to enhance calcium absorption, especially in older adults. Dairy product should be included in the diet. Keeping a food record is a good healthcare practice. Foods high in calcium are recommended to maintain normal calcium level and it is important to verify if the client has allergy to shellfish. 636- The husband of a client with advanced ovarian cancer wants his wife to have every treatment available. When the husband leaves, the client tells the nurse that she has had enough chemotherapy and wants to stop all treatments but knows her husband will sign the consent form for more treatment. The nurse’s response should include which information? a- The husband cannot sign the consent for the client, her signature is required b- The client’s specific wishes should be discussed with her healthcare provider c- Counseling should be sought to resolve the husband’s desire to control his wife d- The healthcare team will formulate a plan of care to keep the client comfortable e- The client should seek a second medical opinion before deciding to stop treatment. Rationale: An adult client who is mentally competent has the autonomy and the client’s right to make her own decision regarding her treatment. 637- The nurse is preparing a 50 ml dose of 50% dextrose IV for a client with insulin shock…medication? a- Dilute the Dextrose in one liter of 0.9% Normal Saline solution. b- Mix the dextrose in a 50 ml piggyback for a total volume of 100 ml. c- Push the undiluted Dextrose slowly through the currently infusion IV. d- Ask the pharmacist to add the Dextrose to a TPN solution. Rationale: To reverse life-threatening insulin shock, the nurse should administer the 50% Dextrose infusing IV. 638- The daughter of an older female client tells the clinic nurse that she is no longer able…lost the ability to perform activities of daily living (ADLs) due to aging. Which options… a. Home hospice agency b. Long-term care facility c. Rehabilitation facility d. Independent senior apartment e. Home health agency Long term care facilities and home health agencies performs ADLs. Hospice provides empathetic, attentive care for dying. C provide physical therapy to strengthen a part of the body. 639- A male client with cancer, who is receiving antineoplastic drugs, is admitted to the…what findings is most often manifest this condition? a. Ecchymosis and hematemesis b. Weight loss and alopecia c. Weakness and activity intolerance d. Sore throat and fever 640- A 7-year-old boy is brought to the clinic because of facial edema. He reports that he has been voiding small amounts of dark, cloudy, tea-colored urine. The parents state that their son had a sore throat 2 week earlier, but it has resolved. After assessing the child's vital signs and weight, what intervention should the nurse implement next? a. Perform an otoscopic examination b. Measure the child’s abdominal girth c. Collect a urine specimen for routine urinalysis d. Obtain a blood specimen for serum electrolytes. Rationale: Acute glomerulonephritis is an auto-immune reaction to a precursory streptococcus. Manifestation of AGN include oliguria, edema, hypertension. 641- The nurse observes an adolescent client prepare to administer a prescribed corticosteroid medication using a metered dose inhaler as seen in the picture. What action should the nurse take? e. Remind the client to hold his breath after inhaling the medication f. Confirm that the client has correctly shaken the inhaler g. Affirm that the client has correctly positioned the inhaler h. Ask the client if he has a spacer to use for this medication i. 642- The nurse teaches an adolescent male client how to use a metered dose inhaler. Seen in the picture. What instruction should the nurse provide? a. Secure the mouthpiece under the tongue. b. Press down on the device after breathing in fully c. Move the device one to two inches away from the mouth d. Breathe out slowly and deeply while compressing the device Rationale: Optimal position of a metered dose inhaler includes placing the inhaler one two inches away from the mouth. 643- A 3-year-old boy with a congenital heart defect is brought to the clinic by his mother…During the assessment, the mother asks the nurse why her child is at the 5th percent…response is best for the nurse to provide? a. Does your child seem mentally slower than his peers also? b. “His smaller size is probably due to the heart disease” c. Haven’t you been feeding him according to recommended daily allowances for children? d. You should not worry about the growth tables. They are only averages for children Rationale: Poor growth patterns are associated with heart disease. 644- A client with hypertension receives a prescription for enalapril, an angiotensin- convertin enzyme inhibitor. What instruction should the nurse include in the medication teaching plan? a. Increase intake of potassium-rich foods b. Report increased bruising of bleeding c. Stop medication if a cough develops d. Limit intake of leafy green vegetables Rationale: ACEIs can cause thrombocytopenia and increased risk for bruising and bleeding. A is not necessary because is a potassium-sparing 645- When administering ceftriaxone sodium (Rocephin) intravenously to a client before…most immediate intervention by the nurse? a. Stridor b. Nausea c. Headache d. Pruritus Rationale Stridor, a crowing respiration, indicates the client is experiencing bronchospasm, as a reaction to Rocephin, and antibiotic. The finding requires immediate action by the nurse. B and C are side effects that are not life-threatening. Pruritus may be the result as… and need nursing intervention but is of less immediacy than stridor. 646- The nurse is assessing a client with a small bowel obstruction who was hospitalized 2 days ago. What should the nurse report immediately to the healthcare provider? a. Hypoactive bowel sounds in the lower quadrant. b. Rebound tenderness in the upper quadrants. c. Tympani with percussion of the abdomen. d. Light colors gastric aspirate via the nasogastric tube. Rationale: Rebound tenderness in the upper quadrant may be indicative of peritonitis. A is a clinical finding associated with bowel obstruction and does not need to be reported D may be something characteristic of the client’s condition. 647- An adult female client is admitted to the psychiatric unit because of a complex handwashing that takes two hours or longer to complete. She worries about staying clean and refuses to sit…client’s handwashing is an example of which clinical behavior? a. Addiction b. Phobia c. Compulsion d. Obsession Rationale: Compulsion is a ritualistic behavior that the individual feels to compelled to do. A dependence on a substance. B is an unfounded fear. D is a recurrent, persistent… 648- A female client reports that she drank a 3/4 liter of a solution to cleanse her intestines…immediately. How many ml of fluid intake should the nurse document? Whole number a. 760 Rationale: 1L=1000ml Subtract the emesis, 1 cup (8 oz.) =240ml 1000-240=760 ml 649- Following routine diagnostic test, a client who is symptom-free is diagnosed with Paget’s disease…directed toward what important goal for this client? a- Maintain adequate cardiac output b- Promote adequate tissue perfusion c- Promote rest and sleep d- Reduce the risk for injury Rationale: Paget’s is a metabolic bone disorder which place the client at high risk for injury. Once the client is symptom free the next goal is reducing risk for injury 650- The mother of a one-month-old boy born at home brings the infant to his first well-baby was born two weeks after his due date, and that he is a “good, quiet baby” who almost…hypothyroidism, what question is most important for the nurse to ask the mother? a. Has your son had any immunizations yet? b. Is your son sleepy and difficult to feed? c. Are you breastfeeding or bottle feeding your son? d. Were any relatives born with birth defects? Rationale: Like adults with hypothyroidism, excess fatigue is common and a “good” baby is of…. occurs with hypothyroidism and can result in poor sucking. 651- In preparing assignments for the shift, which client is best for the charge nurse to assign to a practical nurse (PN)? a- An older client who fell yesterday and is now complaining of diplopia b- An adult newly diagnosed with type 1 diabetes and high cholesterol c- A client with pancreatic cancer who is experience intractable pain. d- An older client post-stroke who is aphasic with right-sided hemiplegia Rationale: Because the communication difficulty and immobility, the PN can provide needed… elements of self-care (ABC) require higher level nursing skills than D 652- Following a gunshot wound to the abdomen, a young adult male had an emergency bowel…Multiple blood products while in the operating room. His current blood pressure is 78/52…He is being mechanically ventilated, and his oxygen saturation is 87%. His laboratory values…Grams / dl (70 mmol / L SI), platelets 20,000 / mm 3 (20 x 10 9 / L (SI units), and white blood cells. Based on these assessments findings, which intervention, should the nurse implement first? a. Transfuse packed red blood cells b. Obtain blood and sputum cultures. c. Infuse 1000 ml normal saline d. Titrate oxygen to keep o2 saturation 90% Rational: The client is exhibiting signs of multiple organ dysfunction syndrome. Transfusion is the first intervention which provide hemoglobin to carry the oxygen to the tissues, is critical. 653- After checking the fingerstick glucose at 1630, what action should the nurse implement? a. Notify the healthcare provider b. Administer 8 units of insulin aspart SubQ c. Gives an IV bolus of Dextrose 50% 50 ml d. Perform quality control on the glucometer. 654- Progressive kyphoscoliosis leading to respiratory distress is evident in a client with muscul…Which finding warrants immediate intervention by the nurse? a. Extremity muscle weakness b. Bilateral eyelid drooping c. Inability to swallow pills d. Evidence of hypoventilation Rationale: Hypoventilation indicates respiratory muscle weakness, and if the client is unable to breath… respiratory distress and life-threatening. 655- An adult male who lives alone is brought to the Emergency Department by his daughter who is unresponsive. Initial assessment indicated that the client has minimal respiratory effort, and his pupils are fixed and dilated. At the daughter’s request, the client is intubated and…Which nursing intervention has the highest priority? a. Offer to notify the client’s minister of his condition. b. Determine if the client has an executed living will c. Provide the family with information about palliative care d. Explore the possibility of organ donation with the family. Rationale: Once the client is intubated and ventilated, emergency intervention should continue until patient t be stable check if the client has an executed living will. 656- The nurse determines that a client’s pupils constricts as they change focus from a far object. What documentation should the nurse enter about this finding? a. Pupils reactive to accommodation b. Nystagmus present with pupillary focus. c. Peripheral vision intact d. Consensual pupillary constriction present Rationale: Pupillary constriction as the pupil change focus is s normal finding called accommodation. 657- Which nursing intervention has the highest priority for a multigravida who delivered… e. Maintain cold packs to the perineum for 24 hrs. f. Assess the client pain level frequently g. Observe for appropriate interaction with the infants. h. Assess fundal tone and lochia flow Rationale D is the priority intervention because is a multigravida and this pregnancy predisposes the client to uterine atony which could result in hemorrhage. 658- A client who had a gestational trophoblastic disease (GTD) evacuated 2 days ago is being…18 months-old child and lives in a rural area. Her husband takes the family car to work daily…transportation during the day. What intervention is most important for the nurse to implement? a. Teach a client amount the use of a home pregnancy test. b. Schedule a weekly home visit to draw hCG values. c. Make a 5 week follow- up with healthcare provider d. Begin chemotherapy administration during the first home visit Rationale: To monitor for development of choriocarcinoma, a complication TD, level of hCG should be monitor for negative results. 659- A newly graduated female staff nurse approaches the nurse manager and request reassignment to another client because a male client is asking her for a date and making suggestive comments. Which response is best for the nurse manager to provide? a. I have to call the supervisor o get someone else to transfer to this unit to care for him. b. I know you are good nurse and can handle this client in a professional manner. c. I’ll talked to the client about his sexual harassment and I’ll insist that he stop it immediately. d. I’ll change your assignment, but let’s talk about you a nurse should respond to this kind of client. 660- After removing a left femoral arterial sheath, which assessment finding warrant immediately interventions by the nurse? (Select all that apply.) a- Tenderness over insertion b- Unrelieved back and flank pain. c- Cool and pale left leg and foot. d- Left groin egg-size hematoma. e- Quarter size red drainage at site. 661- Which instruction is most important for the nurse to provide a client who receives a new plan of care to treat osteoporosis? a. Begin a weight-bearing exercise plan b. Increase intake of foods rich in calcium c. Schedule a bone density tests every year. d. Remain upright after taking the medication. Rationale: Risendronate, causes reflux and esophageal erosion. 662- A newly hired home health care nurse is planning the initial visit to an adult client who has multiple sclerosis (MS) for the past 20 years and is currently bed-bound and if listed by a hoist. And unlicensed caregiver provides care 8hours/daily, 5 days/week. During the initial visit to this client, which intervention is most important to the nurse to implement? a. Determine how the client is cared for when caregiver is not present. b. Develop a client needs assessment and review with the caregiver c. Evaluate the caregiver’s ability to care for the client’s needs. d. Review with the care giver the interventions provided each day. 663- A client with urticaria due to an environmental allergy is taking diphenhydramine... Which complaint should the nurse identify to the client as a side effect of the OTC medication? a. Nausea and indigestion. b. Hyper salivation c. Eyelid and facial twitching d. Increased appetite 664- In caring for a client with a PCA infusion of morphine sulfate through the right cephalic vein, the nurse assesses that the client in lethargic with a blood pressure of 90/60, pulse rate of 118 beats per minute, and respiratory rate of 8 breaths per minutes. What assessment should the nurse perform next? a. Note the appearance and patency of the client’s peripheral IV site. b. Palpate the volume of the client’s right radial pulse c. Auscultate the client’s breath sounds bilaterally. d. Observe the amount and dose of morphine in the PCA pump syringe. 665- A male client is having abdominal pain after a left femoral angioplasty and stent and is asking for additional pain medication for right lower quadrant pain (9/10), two hours ago, he received hydrocodone / acetaminophen 7.5/7.50 mg his vital signs are elevated from reading of a previous hour: temperature 97.8 F, heart rate 102 beats / minute, respiration 20 breaths/minutes. His abdomen is swollen, the groin access site is tender, peripheral pulses are present, but left is greater than right. Preoperatively, clopidogrel was prescribed for a history of previous peripheral stents. Another nurse is holding manual pressure on the femoral arterial access site which may be leaking into the abdomen. What data is needed to make this report complete? a. Client’s lungs are clear bilaterally and oxygen saturation is 97% b. Surgeon needs to see client immediately to evaluate the situation c. Left peripheral pulses were present only by Doppler pre-procedure d. Client’ history includes multiple back surgeries and chronic pain. 666- Which instruction is most important for the nurse to provide a client who is being discharge with Guillian Barre syndrome? a. Avoid exposure to respiratory infections. b. Use relaxation exercise when anxious c. Continue physical therapy at home d. Plan short, frequent rest periods. Rationale: Guillain Barre syndrome is an autoimmune response to a viral infection that impacts immune system… other infections, especially droplet and airborne transmission. Avoiding exposure to a… critical during recovery. B, C, and D are important but do not have the priority. 667- The nurse assesses a female client with obstructive sleep apnea syndrome (OSAS) who is…pounds (70 kg), the client’s 24 hour diet history includes: no breakfast, cheeseburger and cream and a cola drink for dinner, and 2 glasses of wine in the evening before going to be…calories. What instructions should the nurse provide? (Select all that apply) a. Maintain current caloric intake b. Avoid use of alcohol as a sleep aide at bedtime c. Reduce intake of dairy products d. Start a weight loss program e. Set a goal of increasing BMI (Body Mass Index) Rationale: OSAS is associated with obesity. The client’s BMI indicate overweight. 668- A male client with impaired renal function who takes ibuprofen daily for chronic arthritis…gastrointestinal (GI) bleeding. After administering IV fluids and a blood transfusion, his blood pressure is 100/70, and his renal output is 20 ml / hour. Which intervention should the nurse include in hours? a. Maintain the client NPO during the diuresis phase b. Evaluate daily serial renal laboratory studies for progressive elevations. c. Observe the urine character for sedimentation and cloudy appearance. d. Monitor for onset of polyuria greater than 150ml/hr. Hypovolemia due to GI bleeding can cause oliguria that can lead to renal ischemia, which… acute kidney injury (AKI). Azotemia usually develops over hours and is best... BUN, creatinine, and potassium. 669- The health care provider prescribes atenolol 50 mg daily for a client with angina pectoris…to the health care provider before administering this medication? a. Irregular pulse b. Tachycardia c. Chest pain d. Urinary frequency 670- When obtaining a rectal temperature with an electronic thermometer, which action is most important for the nurse to perform? a. Hold the thermometer in place. b. Place the disposable pad under buttocks c. Instruct the client to breathe deeply d. Return the probe to the charger. Rationale: It is essential the thermometer be held on place to reduce the risk of injury to the client. 671- An adult female client is admitted to the psychiatric unit with a diagnosis of major depressive…medication therapy, the nurse notices the client has more energy, is giving her belongings…mood. Which intervention is best for the nurse to implement? a- Support the client by telling her what wonderful progress she is making. b- Ask the client if she has had any recent thoughts of harming herself. c- Reassure the client that the antidepressant drugs are apparently effective d- Tell the client to keep her belongings because she will need hem at discharge. Rationale: A major sign of suicidal ideation is giving one’s belongings away and mood ele… Antidepressant drugs usually take 2 weeks to become effective, but after only 2… client is likely to have the energy to commit the act, but has not enough drug therapy…. Therefore, this client’s behavior necessitates further assessment for suicidal ideation. 672- An adult female client with chronic kidney disease (CKD) asks the nurse if she can continue…Medications. Which medication provides the greatest threat to this client? a. Magnesium hydroxide (Maalox). b. Birth control pills c. Cough syrup containing codeine d. Cold medication containing alcohol Rationale: Magnesium agents are not usually used by clients with CKD or renal failure due to the risk of hypermagnesemia. 673- The nurse observes an unlicensed assistive personnel (UAP) using an alcohol-based clean…tray to the room. The UAP rub both hands thoroughly for 2 minutes while standing at the…should the nurse take? a. Encourage the UAP to remain in the client’s room, until completed b. Explain that the hand rub can be completed in less than 2 minutes. c. Inform the UAP that handwashing helps to promote better asepsis. d. Determine why the UAP was not wearing gloves in the client’s room Rationale: Hand hygiene with alcohol-based rubs can be effectively completed in 20 to 30 second. The UAP can complete the hand hygiene at the client’s door. Gloves are not indicated when delivering food tray unless contact precaution has been place for the client. 674- An adolescent’s mother calls the clinic because the teen is having recurrent vomiting and…Combative in the last 2 days. The mother states that the teen takes vitamins, calcium, mag…With aspirin. Which nursing intervention has highest priority? a. Advise the mother to withhold all medications by mouth. b. Instruct the mother to take the teen to the emergency room c. Recommend that the teen withhold food and fluids for 2 hours d. Suggest that the adolescent breath slowly and deeply. Rationale: Reyes syndrome may occur at any age and is directly correlated with use of salicylic acid…. Clinical signs may indicate encephalitis, a complication for RS, so the adolescent should go to emergency room immediately. 675- A male Korean-American client looks away when asked by the nurse to describe his…nursing action? a- Ask social services to dins a Korean interpreter b- Establish direct eye contact with the client. c- Allow several minutes for the client to respond. d- Repeat the question slowly and distinctly. Rationale: C respect the client’s need to communicate in a cultural acceptable manner. 676- An older female client tells the nurse that her muscles have gradually been getting weak…what is the best initial response by the nurse? a- Explain that this is an expected occurrence with aging. b- Observe the lower extremity for signs of muscle atrophy c- Review the medical record for recent diagnosis test results. d- Ask the client to describe the changes that have occurred Rationale: Gradual muscle weakness in older adult often occurs as a result of loss of muscle tone. The nurse should ask the client for more specific information before assuming that the increase would… 677- When organizing home visits for the day, which older client should the home health nurse… a. A woman who takes naproxen (Naprosyn) and reports a recent onset of dark, tarry stools. b. A man who receives weekly injections of epoetin (Procrit) for a low serum iron level c. A man with emphysema who smokes and is complaining of white patches in his mouth d. A frail woman with heart failure who reported a 2 pounds’ weight gain in the last week. Rationale: A serious side effect of naproxen is GI bleeding, so the dark, tarry stools, a sign of GI bleeding should be assessed first and possible referred to the physician. 678- A client is admitted for type 2 diabetes mellitus (DM) and chronic Kidney disease (CKD)…which breakfast selection by the client indicates effective learning? a. Scrambled eggs, bacon, one slice of whole wheat toast with butter and jam. b. Oatmeal with butter, artificial sweetener, and strawberries, and 6 ounces’ coffee. c. Banana pancake with maple syrup, sausage links, half grapefruit, and low -fat milk d. Orange juice, yogurt with berries, cold cereal with milk, bran muffin with margarine. Rationale: The diet for those with type 2 DM limit simplex sugars while allowing moderate complex… and increased dietary fiber. The diet for those with CKD limits protein, water, sodium and… provides dietary fiber in the oatmeal, without simple carbohydrates, and fresh strawberries. 679- A client with a postoperative wound that eviscerated yesterday has an elevated temperature…What is the most important for the nurse to implement? a. Initiate contact isolation b. Obtain a wound swab for culture and sensitivity c. Assess temperature q4 hours d. Use alcohol-based solutions for hand hygiene. Rationale: Exposure viscera increases the client’s risk for wound infection related to a variety … (HAI) such as MRSA, which requires selective treatment. Nurse should swab the wound and send the specimen to the lab to determine the primary infectious organism, so the healthcare provider can prescribe a medication. 680- The nurse is reinforcing home care instructions with a client who is being discharged following transurethral resection of the prostate (TURP). Which intervention is most important for the nurse to include in the client… a. Avoid strenuous activity for 6 weeks b. Report fresh blood in the urine. c. Take acetaminophen for fever 101 d. Consume 6 to 8 glasses of water daily. Rational: Blood in the urine may be evidence of bleeding that needs immediate intervention 681- The nurse provides feeding tube instructions to the wife of a client with end stage cancer. The client’s wife performs a return demonstration correctly, but begins crying and tells the nurse, “I just don’t think I can do this every day.” The nurse should direct further teaching strategies toward which learning domain? a- Cognitive b- Affective c- Comprehension d- Psychomotor 682- A male client with rheumatoid arthritis is schedule for a procedure in the morning. The…unable to complete the procedure because of early morning stiffness. Which intervention…implement? a- Assign a UAP to assist the client with a warm shower early in the morning 683- The nurse is caring for a client following a myelogram. Which assessment finding should the nurse report to the healthcare provider immediately? a- Complain of headaches and stiff neck 684- A woman who takes pyridostigmine for myasthenia gravis (MG) arrives at the emergency department complaining of extreme muscle weakness. Her adult daughter tells the nurse that since yesterday her mother has been unable to smile, which assessment finding warrants immediate intervention by the nurse? a- Uncontrollable drooling b- Inability to raise voice c- Tingling of extremities d- Eyelid drooling 685- A client with multiple sclerosis (MS) is admitted to the medical unit. The client reports…which action should the nurse implement to reduce the client’s risk for falls? a- Schedule frequent rest periods b- Provide assistance to bedside commode c- Teach to patch one eye when ambulating 686- A client is complaining of intermittent, left, lower abdominal pain that began two days ago…implement the following interventions? a- Correct orders: (DPIA) 1. Determine when the client had last bowel movement 2. Position client supine with knees bent 3. Inspect abdominal contour 4. Auscultate all four abdominal quadrants 687- The nurse is caring for four clients…postoperative hemoglobin of 8.7 mg/dl; client C, newly admitted with potassium…an appendectomy who has a white blood cell count of 15,000mm3. What intervention… a- Determine the availability of two units of packed cells in the blood bank for client B b- Increase the oxygen flow rate to 4 liters/minute per face mask for client A c- Remove any foods, such as banana or orange juice, for the breakfast tray for client C d- Inform client D that surgery is likely to be delayed until the infection responds to antibiotics 688- A client with a new diagnosis of Raynaud’s disease lives alone. Which instruction should the nurse include in the client’s discharged teaching plan? a- Keep room temperature 80 689- Sublingual nitroglycerin is administered to a male client with unstable angina who complains of crushing chest pain. Five minutes later the client becomes nauseated and his bloods pressure drops to 60/40. Which intervention should the nurse implement? a- Infuse a rapid IV normal saline bolus 690- A male client tells the nurse that he is concerned that he may have a stomach ulcer, because he is experiencing heartburn and a dull growing pain that is relieved when he eats. What is the best response by the nurse? a- Encourage the client to obtain a complete physical exam since these symptoms are consistent with an ulcer 691- A mother calls the nurse to report that at 0900 she administered a PO dose of digoxin to her 4-month-old infant, but at 0920 the baby vomited the medicine, what instruction should the nurse provide to this mother a- Withhold this dose 692- When checking a third grader’s height and weight the school nurse notes that these measurements have not changed in the last year. The child is currently taking daily vitamins, albuterol, and methylphenidate for attention deficit hyperactivity disorder (ADHD). Which intervention should the nurse implement? a- Refer child to the family healthcare provider 693- An adolescent receives a prescription for an injection of s-matriptan succinate 4 mg subcutaneously for a migraine headache. Using a vial labeled, 6 mg/ 0.5 ml, how many ml should the nurse administer? (Enter the numerical value only. If rounding is required, round to the nearest hundredth.) 0.33 mL a- Rationale: 4mg x 0.5 ml=2/6=0.33 ml 694- An unlicensed assistive personnel (UAP) informs the nurse who is giving medications that a female client is crying. The client was just informed that she has a malignant tumor. What action should the nurse implement first? a- Tell the client that the nurse will be back to talk to her after medications are given 695- The husband of an older woman, diagnosed with pernicious anemia, calls the clinic to report that his wife still has memory loss and some confusion since she received the first dose of nasal cyanocobalamin two days ago. He tells the nurse that he is worried that she might be getting Alzheimer’s disease. What action should the nurse take? a- Explain that memory loss and confusion are common with vitamin B12 deficiency. b- Ask if the client is experiencing any changes in bowel habits c- Determine if the client is taking iron and folic acid supplements d- Encourage the husband to bring the client to the clinic for a complete blood count. Rationale: Pernicious anemia is related to the absence of intricic factor in gastric secretions, leading to malabsorption of vit B12, and commonly causes memory loss, confusion and cognitive problems, and GI manifestations. The nurse should reassure the husband that the client’s signs are consistent with the primary disease. Although B, C and D provide additional information about the client’s compliance and response to therapy, a quick and dramatic response can occur after 72 hrs. of B12 injections. 696- While the school nurse is teaching a group of 14-year-olds, one of the participants remarks, “You are too young to be our teacher! You’re not much older than we are!” How should the nurse respond? a- “How old do you think I am?” b- “We need to stay focused on the topic.” c- “I think I am qualified to teach this group.” d- “Do you think you can teach it any better?” 697- An unconscious client is admitted to the intensive care unit and is placed on a ventilator. The ventilator alarms continuously and the client's oxygen saturation level is 62%. What action should the nurse take first? a- Begin manual ventilation immediately. 698- After diagnosis and initial treatment of a 3 year old with Cystic fibrosis, the nurse provides home care instructions to the mother, which statement by the child's mother indicates that she understands home care treatment to promote pulmonary functions? a- Chest physiotherapy should be performed twice a day before a meal. 699- A middle-aged woman, diagnosed with Graves’ disease, asks the nurse about this condition. Which etiological pathology should the nurse include in the teaching plan about hyperthyroidism? (Select all that apply.) a- Graves’ disease, an autoimmune condition, affects thyroid stimulating hormone receptors. b- T3 and T4 hormone levels are increased c- Large protruding eyeballs are a sign of hyperthyroid function d- Weight gain is a common complaint in hyperthyroidism e- Early treatment includes levothyroxine (Synthroid). 700- A male client who was admitted with an acute myocardial infarction receives a cardiac diet with sodium restriction and complains that his hamburger is flavorless. Which condiment should the nurse offer? a- Fresh horseradish 701- While completing an admission assessment for a client with unstable angina, which closed questions should the nurse ask about the client's pain? a- Does your pain occur when walking short distances? 702- A nurse who works in the nursery is attending the vaginal delivery of a term infant. What action should the nurse complete prior to leaving the delivery room? a- place the id bands on the infant and mother 703- A female client with chronic urinary retention explains double voiding technique to the nurse by stating she voids partially, hold the remaining urine in her bladder for three minutes, then voids again to empty her bladder fully. How should the nurse respond? a- Advise the client to empty her bladder fully when she first voids 704- A client is receiving an IV solution of nitroglycerin 100mg/500ml D5W at 10 mcg/ minute. The nurse should program the infusion pump to deliver how many ml/hour? (Enter numeric value only) 3 ml/hour Rationale: 0.01 x 500 x 60 / 100 = 3 705- When assessing a multigravida, the first postpartum day, the nurse finds a moderate amount of lochia rubra, with the uterus firm, and three fingerbreadths above the umbilicus. What action should the nurse implement first? a- Massage the uterus to decrease atony b- Review the hemoglobin to determine hemorrhage c- Increase intravenous infusion d- Check for a distended bladder 706- A-12-years old boy has a body mass index (BMI) of 28, a systolic pressure and a glycosylated hemoglobin (HBA1C) of 7.8%. Which selection indicated that his mother understands the management of his diet? a- One whole-wheat bagel with cream cheese, two strips of bacon, six ounces of orange juice. Rationale: Diet - Foods high in carbohydrates and fiber, low fat. No honey, no ham, no high sugar, no frost food, avoid all whole wheat products. 707- Which class of drugs is the only source of a cure for septic shock? a- Antihypertensives b- Antiinfectives c- Antihistamines d- Anticholesteremics. Rationale: Antiinfective agents, such as antibiotics, are the only drugs that eliminate bacteria. The only to halt destruction to organ system in septic shock is to eliminate the production of endotoxins by bacterial invaders. A is contraindicated due to the low cardias output which in low blood pressure and occurs in late septic shock. While C may reduce some of the destructive effects of massed cell release occurring with the inflammatory response that may occur, endotoxin release would not be stopped. D has no therapeutic effect relevant to septic shock. 708- A 59-year-old male client comes to the clinic and reports his concern over a lump that, “just popped up on my neck about a week ago.” In performing an examination of the lump, the nurse palpates a large, nontender, hardened left subclavian lymph node. There is not overlying tissue inflammation. What do these findings suggest? a- Malignancy b- Bacterial infection c- Viral infection d- Lymphangitis Rationale: Rapid enlargement of a lymph node, particularly the subclavian node with no tenderness of inflammation is suggestive of malignancy. Lymphangitis is characterized by pain and inflammation. Infectious processes C and D the involved nodes become warm and tender to touch. 709- A gravida 2 para 1, at 38-weeks’ gestation, scheduled for a repeat cesarean section in one week, is brought to the labor and delivery unit complaining of contractions every 10 minutes. While assessing the client, the client’s mothers enter the labor suite and says in a loud voice, “I’ve had 8 children and I know she’s in labor. I want her to have her cesarean section right now!” what action should the nurse take? a- Request the mother to leave the room b- Tell the mother to stop speaking for the client c- Request security to remove her from the room d- Notify the charge nurse of the situation Rationale: The nurse should ask the family member to leave the room (A) because the behavior is disruptive to the nurse and to the client. After the assessment is completed, the nurse should the address the family member’s concerns. 710- While caring for a toddler receiving oxygen (02) via face mask, the nurse observes that the child’s lips and nares are dry and cracked. Which intervention should the nurse implement? a- Ask the mother what she usually uses on the child’s lips and nose b- Apply a petroleum jelly (Vaseline) to the child’s nose and lips c- Use a topical lidocaine (Zylocaine viscous) analgesic for cracked lips d- Use a water soluble lubricant on affected oral and nasal mucosa 711- The healthcare provider prescribes carboprost tromethamine (Hemabate) 250 mcg IM for a multigravida postpartum client who is experiencing heavy, bright red vaginal bleeding. Prior to administering this medication, which interventions should the RN implement? a- Obtain a second IV access. b- Decrease the room temperature. c- Give the prescribed antiemetic. d- Insert an indwelling catheter. 712- During the infusion of a second unit of packed red blood cells, the client’s temperature increases from 99 to 101.6 f. which intervention should the nurse implement? a- Stop the transfusion start a saline b- Observe for a maculopapular rash c- Report the fever to the blood bank d- Give a PRN dose of acetaminophen 713- An elderly female client with osteoarthritis reports increasing pain and stiffness in her right knee and asks how to reduce these symptoms. In responding to the client, the nurse recognizes what pathology as the cause of her symptoms? a- Destruction of joint cartilage. 714- When caring for a client with traumatic brain injury (TBI) who had a craniotomy for increased intracranial pressure (ICP), the nurse assesses the client using the Glasgow coma scale (GCS) every two hours. For the past 8 hours the client’s GCS score has been 14. What does this GCS finding indicate about the client? a- Neurologically stable without indications of an increased ICP b- Insertion of ICP monitoring device is necessary c- Rehabilitative prognosis is an expected full recovery d- Risk for irreversible cerebral damage related to increased ICP Rationale: The GCs is valid and reliable neurologic assessment scale that is used to identify early changes in ICP, determine severity of TBI direct treatment, and provide prognostic information. A repeated score of 14 indicates the client is not awake, but not experiencing increase ICP (A) ICP monitoring devices provide a precise quantitative pressure reading, but insertion of such a device B is not indicated at this time. Although preliminary assessment date is used to predict survival of TBI, expectation of a full recovery C is a premature assumption. The GCS is used to guide therapy, but it does not measure outcomes, such as function or cognitive ability (D) 715- A 46-year-old male client who had a myocardial infarction 24-hours ago comes to the nurse’s station fully dressed and wanting to go home. He tells the nurse that he is feeling much better at this time. Based on this behavior, which nursing problem should the nurse formulate? a- Ineffective coping related to denial b- Anxiety related to treatment of choice c- Decisional conflict related to stress d- Deficient knowledge related to lifestyle changes. Rationale: This client is demonstrating behaviors associated with denial of his condition. B may be appropriate, but only if the client were acknowledging that a serious condition existed. Although this client is likely to experiencing stress, which might lead to a decisional conflict a is more specific for the behavior being exhibited. While D should be addressed in this client’s plan of care, it also does not specifically address the behavior the client is currently exhibiting. 716- In assessing a client 48 hours following a fracture, the nurse observes ecchymosis at the fracture site, and recognizes that hematoma formation at the bone fragment site has occurred. What action should the nurse implement? a- Document the extend of the bruising in the medical record b- Call the lab to obtain a stat APTT and prothrombin time c- Assign a UAP to take vital sig measurements q 1h. d- Advise the client that anticoagulant therapy may be needed. Rationale: Since this is a normal and desirable stage, the nurse only needs to document the finding (A). Hematoma formation, often evidenced by bruising is the first in the fracture healing process, and occurs in the initial 72 hrs. post fracture. 717- A client is admitted for cellulitis surrounding an insect bite on the lower, right arm and intravenous (IV) antibiotic therapy is prescribed. Which action should the nurse implement before performing venipuncture? a- Lower the right arm below the level of the heart b- Elevate both arms on two pillows c- Lower the left arm below the level of the heart d- Apply a tourniquet above the right antecubital fossa Rationale: Since the client has an infection in the right lower arm, the IV should be started in the opposite arm, and the nurse should lower the left arm to dilate the vessels and facilitate cannulation of a vein. May be elevate the affected arm should help but not both arms. 718- Which assessment finding of a postmenopausal woman necessitates a referral by the nurse to the healthcare provider for evaluation of thyroid functioning? m- Slow weight loss n- Muscle weakness o- Cold sensitivity p- Leg numbness 719- A client with hyperthyroidism is admitted to the postoperative after subtotal thyroidectomy. Which of the client’s serum laboratory values requires intervention by the nurse? a- Thyroxine 12 ug/dl b- Total calcium 5.0 mg/dl c- T3 uptake at 50% d- Glucose 150 mg/dl 720- A female client on the mental health unit frequently asks the nurse when she can be discharged. Then, becoming more anxious, she begins to pace the hallway. What intervention should the nurse implement first? a- Explore the client’s reasons for wanting to be discharged. 721- The nurse is assessing a primigravida a 39-weeks gestation during a weekly prenatal visit. Which finding is most important for the nurse to report to the healthcare provider? a- Complain of early morning heart burn b- Report intermittent low back pain c- Fetal heart rate of 200 beats/minute d- Maternal hemoglobin of 11.0 grams 722- A female client receives a prescription for alendronate sodium (Fosamax) to treat her newly diagnose osteoporosis. What instruction should the nurse include in the client’s teaching plan? q- Consume a light snack with the medication r- Take on an empty stomach with a full glass of water s- Ingest an antacid 30 minutes of taking the medication. t- Eat within 30 minutes of taking the medication. 723- The nurse is assessing a female client’s blood pressure because she reported feeling dizzy. The blood pressure cuff is inflated to 140 mm hg and as soon as the cuff is deflated a korotkoff sound is heard. Which intervention should the nurse implement next? a- Wait 1 minute and palpate the systolic pressure before auscultating again. b- Educate the client about risk factor that predispose one for hypertension c- Obtain a medication history to assess for drugs that affect blood pressure d- Provide a quiet environment and retake the blood pressure in 20 mints. Rationale: To accurately assess blood pressure, the sphygmomanometer cuff should be inflated above the client’s usual systolic reading, but a Korotkoff sound heard immediately upon deflating the cuff indicated that the cuff was insufficiently inflated. The systolic pressure should be palpated at the radial pulse, which provides a reference as to how high to inflate the cuff when auscultating the blood pressure. B, there is insufficient date to suggest that the client has hypertension. C A medication history can be obtained after an accurate blood pressure measurement is obtained 724- After administering a proton pump inhibitor (PPI), which action should the nurse take to evaluate the effectiveness of the medication? a- Ask the client about gastrointestinal pain b- Auscultate for bowel sounds in all quadrants c- Measure the client’s fluid intake and output d- Monitor the client’s serum electrolyte levels. Rationale: Proton pump inhibitor suppress gastric acid secretion, relieving the symptoms of peptic ulcer disease and GERD. To evaluate the effectiveness of PPIs, the client should be asked about the relief of symptoms such as gastrointestinal discomfort. 725- A client with pneumonia has arterial blood gases levels at: PH 7.33; PaCO2 49 mm/hg; HCO3 25 mEq/L; PaO2 95. What intervention should the nurse implement based on these results? a- Instruct the client to breath into a paper bag. b- Prepare to administer sodium chloride fluids c- Institute coughing and deep breathing protocols d- Initiate oxygen administration at 2 to 3 L per nasal cannula Rationale: Pulmonary hygiene measures will clear the respiratory tract of mucus and purulent drainage, thereby improving ventilation, since these ABG’s reveal respiratory acidosis, and treatment should be directed to improving ventilation. A would be good for respiratory alkalosis, B for metabolic alkalosis A paO2 of 95 is within normal limits do D is not necessary 726- The healthcare provider explains through an interpreter the risks and benefits of a scheduled surgical procedure to a non-English speaking female client. The client gives verbal consent and the healthcare provider leaves, instructing the nurse to witness the signature on the consent form. The client and the interpreter then speak together in the foreign language for an additional 2 minutes until the interpreter concludes, “She says it is OK.” What action should the nurse take next? a- Have the interpreter co- sign the consent to validate client understanding b- Have the client sign the consent and the nurse witness the signature c- Ask for a full explanation from the interpreter of the witnessed discussion. d- Clarify the client’s consent through the use of gestures and simple terms. Rationale: The interpreter’s role is to literally translate exactly what the client indicates not to provide a summary. Further information is needed about what was said during the lengthy conversation before proceeding. The nurse concerns about informed consent are not answered by Ab or D 727- While assisting a male client who has muscular dystrophy (MD) to the bathroom, the nurse observes that he is awkward and clumsy. When he expresses his frustration and complains of hip discomfort, which intervention should the nurse implement? a- Administer a PRN dose of pain medication b- Place a portable toilet next to the bed c- Restrict activity to complete bed rest d- Evaluate the client’s leg muscular strength. Rationale: Due to the contractures and muscle weakness that progress with MD, the client’s awkward movements and clumsiness is an expected sequela. Using assistive devices, such as bedside toilet, should be implement to help limit the client’s frustration and ensure client safety, Discomfort is constant and may not always require pain medication (A). Activity should be encouraged (C) as long as the client is capable. (D) should be implemented before mobilizing the client. 728- A client with hyperthyroidism who has not been responsive to medications is admitted for evaluation. What action should the nurse implement? (Click on each chart tab for additional information. Please scroll to the bottom right corner of each tab to view all information contained in the client’s medical record.) a- Give acetaminophen 650 mg PO b- Obtain a STAT 12 lead electrocardiogram c- Encourage the client to rest d- Notify the healthcare provider Rationale: The client TSH assay reveals a decreased serum TSH and elevated free thyroxine (T4) and triiodothyronine (T3) which are indicative of a hyperthyroid state. An increase in 1 to 2 degree of temperature within a 2-hour period is indicative of a thyroid storm, which is a life-threatening medical emergency, so the health care provider should be notified immediately (D). The client temperature is not elevated enough to warrant using acetaminophen (A) as an antipyretic. Although an electrocardiogram may be needed, (B) ignores the important warning signs of a thyroid storm. C is not indicated at this time. 729- While taking vital signs, a critically ill male client grabs the nurse’s hand and ask the nurse not to leave. What action is best for the nurse to take? a- Allow the client to hold the nurse’s hand until the vital signs can be completed b- Reassure the client that the nurse will return after all vital signs are taken c- Tell the client that he must release the nurse’s hand. d- Pull up a chair and sit beside the client’s bed Rationale: The critically ill client is most likely pleading for the presence of another person. D is the action that a compassionate nurse would implement. A, B, C do not demonstrate the compassion of D 730- The practical nurse (PN) is assigned to work with three registered nurses (RN) who are caring for neurologically compromised clients. The client with which change in status is best to assign to the PN? a- Diabetic ketoacidosis whose Glasgow coma Scale score changed from 10 to 7 b- Myxedema coma whose blood pressure changed from 80/50 to 70/40 c- Viral meningitis whose temperature changed from 101 F to 102 F. d- Subdural hematoma whose blood pressure changed from 150/80 to 170/60. Rationale: The most stable patient should be assigned to the PN, changes in the Glasgow coma Scale indicated the client’s neurological status is worsening. The client decreasing BP is physiologically unstable. An increasing systolic blood pressure and widening pulse pressure is indicative of increasing intracranial pressure. 731- An IV antibiotic is prescribed for a client with a postoperative infection. The medication is to be administered in 4 divided doses. What schedule is best for administering this prescription? a- 0800, 1200, 1600, 2000 b- Administer with meals and a bedtime snack c- Five in equally divided doses during waking hours d- 1000, 1600, 2200, 0400 Rationale: D is the best schedule because the antibiotic should be administered around the clock to keep the blood level of the antibiotic constant. 732- A male client notifies the nurse that he feels short of breath and has chest pressure radiating down his left arm. A STAT 12-lead electrocardiogram (ECG) is obtained and shows ST segment elevation in leads II, II, aVF and V4R. The nurse collects blood samples and gives a normal saline bolus. What action is most important for the nurse to implement? a- Obtain the results for STAT serum cardiac biomarkers b- Asses for contraindications for thrombolytic therapy c- Measure ST-segment height and waveform changes. d- Transfer for percutaneous coronary intervention (PCI) Rationale: ST segment elevation myocardial infarction (STEMI) usually occurs with complete occlusion of an epicardial coronary artery which requires early reperfusion therapy. Screening the client for fibrinolytic therapy (B) is most important to determine PCI option for rapid reperfusion. If the client is not a candidate for fibrinolytic therapy, then transfer to a PCI unit or facility is indicated. Reperfusion therapy should be delayed in STEMI (A). (C) is of significant concern in ECG interpretation with ST-segment depression, not STEMI 733- A client with Addison’s crisis is admitted for treatment with adrenal cortical supplementation. Based on the client’s admitting diagnosis, which findings require immediate action by the nurse? (Select all that apply) a- Headache and tremors b- Irregular heart rate c- Skin hyperpigmentation d- Postural hypotension e- Pallor and diaphoresis 734- A client with rapid respirations and audible rhonchi is admitted to the intensive care unit because of a pulmonary embolism (PE). Low-flow oxygen by nasal cannula and weight based heparin protocol is initiated. Which intervention is most important for the nurse to include in this client’s plan of care? a- Monitor deep vein blood flow using Doppler b- Evaluate daily blood clotting factors. c- Apply antiembolism stockings. d- Maintain strict bed rest. Rationale: Monitoring clotting factors is the most important intervention to include in this client’s plan of care following oxygen administration, IV fluids and heparin administration to prevent clot enlargement. Ac and D should be included in the client’s plan of care, but these interventions do not have the priority of B 735- The nurse enters a client’s room to administer scheduled daily medications and observes the client leaning forward and using pursed lip breathing. Which action is most important for the nurse to implement first? a- Administer schedule medications b- Offer the client PRN anxiolytic c- Assess the lungs for wheezing d- Evaluate the oxygen saturation. Rationale: The client is exhibiting symptoms of an acute exacerbation of a chronic obstructive lung disease such as emphysema. The client… baseline oxygen level should be compared to the current level to determine if respiratory decompensation is occurring. Schedule medications can be administered after completing the oxygen saturation assessment. Respiratory distress often makes a client anxious, which may worsen the symptoms, so should be considered after implementing D. Assessing the lung for wheezing does not reveal further respiratory compromise 736- During a clinic visit, a client with a kidney transplant ask, “What will happen if chronic rejection develops?” which response is best for the nurse to provide? a- The immunosuppressant medication will be increased until the rejection subside b- Dialysis may be necessary until the chronic rejection can be reversed. c- Dialysis would need to be resumed if chronic rejection becomes a reality d- A different combination of immunosuppressant medications will be implemented. Rationale: Chronic rejection is managed conservatively by treating the symptoms until dialysis is needed. Immunosuppressant medication dosage are not increased when chronic rejection occurs, but are during acute rejection. 737- The nurse enters a client’s room and observe the unlicensed assistive personnel (UAP) making an occupied bed as seen in the picture. What action should the nurse take first? a- Provide the gloves for the UAP to apply b- Offer to help reposition the client c- Instruct the UAP to raise the bed level d- Place the side rails in an up position Rationale: To maintain the client safety, it is most important for the nurse to place the side rails in a up position to reduce the risk of falls and injury. A, B and C can then be completed. 738- A client is receiving continuous bladder irrigation via a triple-lumen suprapubic catheter that was placed during prostatectomy. Which report by the unlicensed assistive personnel (UAP) requires intervention by the nurse? a- Pale pink urine output b- Dark red clot in urine c- Leakage around catheter insertion site d- Urinary output greater than 90 ml/hour. Rationale: After genitourinary surgery, the client is at risk for blood clots and mucus fragments occluding the catheter. Leakage of urine around the suprapubic insertion site indicates blockage of the catheter that causes urine back-up resulting in bladder distention and overflow leakage around the catheter. Pink urine and clots are normal finding 739- A client with bleeding esophageal varices receives vasopressin (Pitressin) IV. What should the nurse monitor for during the IV infusion of this medication? a- Chest pain and dysrhythmia b- Vasodilation of the extremities c- Hypotension and tachycardia d- Decreasing GI cramping and nausea. Rationale: In large doses, vasopressin may produce increased blood pressure, coronary insufficiency, myocardial ischemia or infarction and dysrhythmias. 740- A male client with cancer who has lost 10 pounds during the last months tells the nurse that beef, chicken, and eggs, which used to be his favorite foods, now they taste “bitter”. He complains that he simply has no appetite. What action should the nurse implement? a- Instruct the client to add ground beef and chicken in small amount to casseroles. b- Encourage the client to try to eat these foods in moderation despite the taste c- Advise the client to replace the bitter-tasting foods with fruits and vegetables. d- Suggest the use of alternative sources of protein such as dairy products and nuts. Rationale: Beef, chicken, and eggs are good source of protein. To promote weight gain and adequate protein intake, the nurse should teach the client about another source of protein. Attempting to eat food that cause a bitter taste A and B is likely to increase the client’s anorexia. C does not provide a sufficient source of protein. 741- A nurse plans to call the healthcare provider to report an 0600-serum potassium level of 2 mEq/L or mmol/L (SI), but the charge nurse tells the nurse that the healthcare provider does not like to receive early morning calls and will make rounds later in the morning. What action should the nurse make? a- Contact the healthcare provider immediately to report the laboratory value regardless of the advice. b- Call the lab to draw an additional blood sample for a repeat evaluation of the potassium level STAT. c- Flag the client’s medical record to alert the healthcare provider immediately upon arrival to the unit. d- Ask the charge nurse to contact the healthcare provider with the laboratory results by mid-morning. Rationale: A serum potassium level of 2 mEq/L or mm/L (SI) is dangerous low and requires immediate intervention A to prevent potentially fatal cardiac dysrhythmias, regardless of the charge nurse concern regarding disturbing the healthcare provider, B, C and D may result in a potentially fatal delay in responding to the hypokalemia. 742- Which actions should the nurse implement with auscultating anterior breath sounds? (Place the first action on top and last action on the bottom.) 1. Place stethoscope in suprasternal area to auscultate from bronchial sounds 2. Auscultate bronchovesicular sounds from side to side of the first and second intercostal spaces 3. Displace female breast tissue and apply stethoscope directly on chest wall to hear vesicular sounds 4. Document normal breath sounds and location of adventitious breath sounds Rationale: Begin auscultation of anterior breath sounds over the trachea and larynx to identify bronchial breath sounds. Assessment should proceed down the anterior chest from side to hear bronchovesicular sounds, which are located over major bronchi around the upper sternum in first and second intercostal spaces. To hear vesicular sounds over peripheral lung fields where air flows through smaller airway, the breast tissue should be displaced so the stethoscope lies directly on chest wall. Documentation should include normal breath sounds and any adventitious findings. 743- Methylprednisolone (Solu-Medrol) 100 mg IV is prescribed for a client. The medication comes in a vial labeled “125 mg per ml” How many ml should the nurse administer. (enter numeric value only. If rounding is required, round to the nearest tenth) 0.8ml 744- An adult woman who has a history of inferior myocardial infarction, esophageal reflux, and type 1 diabetes mellitus (DM) is admitted to the telemetry unit for sudden onset of dizziness with palpitations and burning sensation in her chest. Which intervention should the nurse implement first? a- Review the client’s last meal choice b- Administer an oral antacid c- Assess blood glucose level d- Evaluate telemetry cardiac rhythm. 745- A client with Huntington’s disease is exhibiting choreic movements. Which action the nurse implement first? a- Assess the client’s vital signs and breath sounds b- Place padding around the side rails of the beds c- Instruct the client to perform deep breathing d- Document the behavior in the client’s medical record. Rationale: The nurse’s highest priority is to initiate measures to protect the client (B) because choreic movements are involuntary hyperkinetic movements, which may result in injury to the client. This is typical of Huntington’s disease and does not cause significant changes in A. Because the movements are involuntary C may help the client relax, but will not reduce the movements. D should be addressed after safety issues are resolved. 746- A client who is in labor tells the nurse: “I just felt a large gush of water- I think my water broke”. After assessing the FHR of 140 beats per minute, which action should the nurse implement next? a- Perform a sterile vaginal exam b- Turn to the left lateral position c- Monitor maternal vital signs d- Document fetal heart rate. 747- A client’s cardiac monitor displays new prominent U waves. What action should the nurse implement? Select all that apply. a- Administer a PRN sublingual nitrate tablet b- Assess rhythm for ST depression c- Evaluate client’s current potassium level d- Notify the rapid response team immediately e- Obtain a 12 lead electrocardiogram 748- A female client experiencing her four episode of heart failure insist that she has followed a strict diet and taken her prescribe medications. What intervention is most important to for the nurse to include in the client’s discharge teaching plan? a- Describe possible medication side effects b- Schedule heart failure educator follow-up c- Encourage recording intake and output d- Instruct the client to keep a diet journal. 749- A female client with vomiting and diarrhea for the past 2 days arrives in the fast-track clinic complaining of thirst but is… tolerate oral fluid at this time. Her blood pressure is 110/70 with heart rate of 80 beats/minute, and her standing blood pressure is 90/60 with a heart rate of 120 beats/minute. Which action is most important for the nurse to take? a- Contact the healthcare provider about initiating parenteral fluids b- Monitor the client’s orthostatic vital sign every 2 hours. c- Review serum sodium and blood urea nitrogen (BUN) results d- In 2 hours offer the client small amounts of clear oral liquids. Rationale: A sign of dehydration is thirst, which does usually occur unless there is 25% fluid deficiency. Parenteral fluids should be considered, so the healthcare provider should be contacted. B, C and D should be implemented but addressing the client need for parenteral fluid is most important 750- The nurse observes a UAP begin to removes exam gloves after emptying a bedpan containing feces. The UAP slides two fingers inside one of the gloves and begins to roll the glove off. What action should the nurse implement? a- Remind the UAP to discard the gloves in the biohazard container after removal b- Suggest that the UAP roll both of the gloves and insider out at the same time c- Advise the UAP that the technique being used will result in hand contamination d- Instruct the UAP to use two pairs of gloves when fecal contamination is likely. Rationale: By sliding glove fingers inside the other glove the UAP’s hand become contaminated. Gloves should be discarded in the trash unless heavily contaminated with body fluids is likely to result in contamination of both hands. Double gloving is not necessary when performing activities as emptying a bedpan. 751- An adult woman who has recently diagnosed with type 2 diabetes mellitus (DM) is seen in the clinic for laboratory test. The client height is 5 feet and 2 inches (152.5 cm) and weight is 165 pounds (74.8kg). Her recent laboratory finding are described above. In planning nutrition teaching for this client, what diet modifications should the nurse recommend? Select all that apply. a- Reduce daily fat intake to 10% of total calories b- Increase dietary fiber such as whole grains c- Decrease processed carbohydrate in diet d- Eliminate alcohol intake except for special occasions. e- Restrict protein to 10% of total calories in diet. 752- An experiences UAP is hired to work with in an antepartal clinic. Which assignments is best for this new employee’s first day of work at the clinic? a- Accompany the health care provider during pelvic examinations b- Discuss good dietary choices with clients using a char provided by the clinic c- Take initial vital signs and urine checks for glucose along with the nurse. d- Schedule clients for their next antepartal visit to the clinic. Rationale: The nurse would have an opportunity to assess the UAP’s skills such as vital signs, glucose monitoring and communication by working with the UAP. A new employee would not be familiar with the location of supplies or the routines of the healthcare providers. B is responsibility of the RN or aa PN if standard plan is used. Evaluation of client’s condition and knowledge of clinic protocol are required when scheduling a client ‘s next antepartal visit, which would require judgment of the RN. 753- The home health nurse assesses a male client who is on the waiting list for a liver transplant. The client expresses fear that he will die before a liver becomes available. How should the nurse respond? a- Encourage the client to discuss how is he is copying with his fearfulness b- Offer to notify the healthcare provider about his feeling of fearfulness c- Explain that he will be moved up on the list if his condition worsens. d- Assess the client’s current mental status and over-all cognitive functions. 754- The healthcare provider prescribes a low dose heparin protocol at 18 units/kg/hr. for a client with a possible pulmonary embolism. This client weighs 144 pound. The available solution is labeled, Heparin Sodium 25,000 Units in 5% dextrose… 250ml. The nurse should program the pump to deliver how many ml/hr. (enter numeric value only. If round to the nearest whole number) a- 12. Rationale: Client’s weight 144 lbs./2.2 = 65 kgs 65 kg x 18 units/kg/hr.= 1170 units/hour. 25,000 units of heparin divided by 250ml of D5W = 100 units heparin per 1 ml of solution. Using the formula D/H x A=X 755- The nurse is planning care for a client at 30-weeks gestation who is experiencing…. Most important in preventing this fetus from developing respiratory distress syndrome… a- Butorphanol 1mg IV push q2h PRN pain b- Betamethasone (celestone) 12 mg deep IM c- Terbutaline (Brethine) 0.25 mg subcutaneously q15minutesx 3 d- Ampicillin 1 Gram IV push q8h. Rationale: RDS is common in preterm infants who have immature lungs. Has been found to de reduces is glucocorticoids (betamethasone….) hours before birth to a woman who is at less 3-weeks gestation…. 756- The nurse places one hand above the symphysis while massaging the fundus of a multiparous… 15 mints after delivering a 7 pound and 10 ounces’ infant. Which information should the nurse pr….. a- Clots may form inside a boggy uterus and need to be expelled b- The uterus should to be firm to prevent an intrauterine infection c- A firm uterus prevents the endometrial lining from being sloughed d- Both the lower uterine segment and the fundus must be massaged. Rationale: A boggy uterus indicates the uterus is not contracting adequately, the uterus is not contacting adequately, which means that the uterus…. Intrauterine bleeding that causes the uterus become a reservoir to hold blood clots. [Show More]

Last updated: 1 year ago

Preview 1 out of 67 pages

Reviews( 0 )

$18.00

Add to cart

Instant download

Can't find what you want? Try our AI powered Search

OR

GET ASSIGNMENT HELP
63
0

Document information


Connected school, study & course


About the document


Uploaded On

Apr 14, 2021

Number of pages

67

Written in

Seller


seller-icon
Grade_Bender

Member since 3 years

4 Documents Sold


Additional information

This document has been written for:

Uploaded

Apr 14, 2021

Downloads

 0

Views

 63

Document Keyword Tags

Recommended For You


$18.00
What is Browsegrades

In Browsegrades, a student can earn by offering help to other student. Students can help other students with materials by upploading their notes and earn money.

We are here to help

We're available through e-mail, Twitter, Facebook, and live chat.
 FAQ
 Questions? Leave a message!

Follow us on
 Twitter

Copyright © Browsegrades · High quality services·